Sie sind auf Seite 1von 233

Business Logistics/

Supply Chain
Management
Planning, Organizing, and Controlling the Supply Chain
Fifth Edition

Instructors Manual

Ronald H. Ballou
Weatherhead School of Management
Case Western Reserve University

CONTENTS
Preface
Chapter 1
2
3
4
5
6
7

10
11
12
13

14

15
16

Business Logistics/Supply ChainA Vital Subject


Logistics/Supply Chain Strategy and
Planning
The Logistics/Supply Chain Product...
Logistics/Supply Chain Customer Service..
Order Processing and Information Systems.
Transport Fundamentals..
Transport Decisions.
Fowler Distributing Company..
Metrohealth Medical Center.
Orion Foods, Inc...............
R & T Wholesalers...
Forecasting Supply Chain Requirements.
World Oil..
Metro Hospital .
Inventory Policy Decisions..
Complete Hardware Supply, Inc...
American Lighting Products.
American Red Cross: Blood Services..
Purchasing and Supply Scheduling Decisions.
Industrial Distributors, Inc
The Storage and Handling System...
Storage and Handling Decisions..
Facility Location Decisions.
Superior Medical Equipment
Company....
Ohio Auto & Drivers License Bureau.
Southern Brewery
The Logistics Planning Process...
Usemore Soap
Company...
Essen USA...
Logistics/Supply Chain Organization.
Logistics/Supply Chain
Control...

iii
1
2
4
9
13
14
17
35
41
48
52
65
84
88
94
121
124
131
134
144
147
148
162
186
190
198
204
208
217
229
230

ii

PREFACE
This instructor's guide provides answers to the more quantitatively oriented problems at
the end of the textbook chapters. If the questions or problems are for discussion or they
involve a substantial amount of individual judgment, they have not been included.
Solutions to the cases and exercises in the text are also included. These generally
require computer assistance for solution.
With the text, you are provided with a collection of software programs, called
LOGWARE, that assist in the solution of the problems, cases, and exercises in the text.
The LOGWARE software along with a users manual is available for downloading from
the Prentice Hall website or this book. The users manual is in Microsoft Word or
Acrobat .pdf formats. This software, along with the users manual, may be freely
reproduced and distributed to your classes without requiring permission from the
copyright holder. This permission is granted as long as the use of the software is for
educational purposes. If you encounter difficulty with the software, direct questions to
Professor Ronald H. Ballou
Weatherhead School of Management
Case Western Reserve University
Cleveland, Ohio 44106
Tel: (216) 368-3808
Fax: (216) 368-6250
E-mail: Ronald.Ballou@CASE.edu
Web site: www.prenhall.com/ballou

iii

CHAPTER 1
BUSINESS LOGISTICS/SUPPLY CHAINA VITAL SUBJECT
12
(a) This problem introduces the student to the evaluation of alternate channels of
production and distribution. To know whether domestic or foreign production is least
expensive, the total of production and distribution costs must be computed from the
source point to the marketplace. Two alternatives are suggested, and they can be
compared as follows.
Production at Houston:
Total cost = Production cost at Houston + Transportation and storage costs
= $8/shirt100,000 shirts + $5/cwt. 1,000 cwt.
= $805,000/year
Production at Taiwan:
Total cost = Production cost in Taiwan
+ Transportation and storage costs from Taiwan to Chicago
+ Import duty + Raw material transportation cost from Houston
to Taiwan
= $4/shirt100,000 shirts + $6/cwt. 1,000 cwt. + $0.5/shirt100,000 shirts
+ $2/cwt. 1,000 cwt.
= $458,000/year
Producing in Taiwan would appear to be the least expensive.
(b) Other factors to consider before a final decision is made might be:
(i) How reliable would international transportation be compared with domestic
transportation?
(ii) What is the business climate in Taiwan such that costs might change in favor of
Houston as a production point?
(iii) How likely is it that the needed transportation and storage will be available?
(iv) If the market were to expand, would there be adequate production capacity
available to support the increased demand?

CHAPTER 2
LOGISTICS/SUPPLY CHAIN STRATEGY AND PLANNING
13
The purpose of this exercise is to allow the student, in an elementary way, to examine the
tradeoffs between transportation and inventory-related costs when an incentive
transportation rate is offered. Whether the incentive rate should be implemented depends
on the shipment size corresponding to the minimum of the sum of transportation, inventory, and order processing costs. These costs are determined for various shipping
quantities that might be selected to cover the range of shipment sizes implied in the
problem. Table 2-1 gives a summary of the costs to Monarch for various shipment sizes.
From Monarch's point of view, the incentive rate would be beneficial. Shipment
sizes should be approximately doubled so that the 40,000 lb. minimum is achieved. It is
important to note that the individual cost elements are not necessarily at a minimum at
low shipment sizes, whereas order-processing costs are low at high shipment sizes. They
are in cost conflict with each other. Transportation costs are low at high shipment sizes,
but exact costs depend on the minimum volume for which the rate is quoted.
In preparation for a broader planning perspective to be considered later in the text, the
student might be asked what the place of the supplier is in this decision. How does he
affect the decision, and how is he affected by it? This will focus the student's attention
on the broader issues of the physical distribution channel.

TABLE 2-1

Evaluation of Alternative Shipment Sizes for the Monarch Electric Company


Current
Proposed
57 motors
114 motors
171 motors
228 motors
285 motors
or
or
or
or
or
10,000 lb.
20,000 lb.
30,000 lb.
40,000 lb.
50,000 lb.
Type of cost
Transportation
38,750
98,750
58,750
58,750
38,750
= $26,250
RD
= $78,750
= $43,750
= $43,750
= $26,250a
b
Inventory carrying
0.25200114/2
0.25200171/2
0.25200228/2
0.25200285/2
0.2520057/2
= $2,850
= $4,275
= $5,700
= $7,125
ICQ/2
= $1,425a
c
Order processing
5,00015/57
5,00015/114
5,00015/171
5,00015/228
5,00015/285
DS/Q
= $1,316
= $658
= $439
= $329
= $263a
Handling
0.308,750
0.308,750
0.308,750
0.308,750
0.308,750
HD
= $2,625
= $2,625
= $2,625
= $2,625
= $2,625
Total
$84,116
$49,883
$51,089
$36,263
$34,904a
a

Minimum values.
Students should be informed that average inventory can be approximated by one half the shipment size.
c
Demand D has been converted to units per year.
LEGEND
R = transportation rate, $/cwt.
D = annual demand, cwt.
I = inventory carrying cost, %/year.
C = cost of a motor, $/motor.
Q = shipment size in motors, where Q/2 represents the average number of motors maintained in inventory.
S = order processing costs, $/order.
H = handling costs, $/cwt.
b

CHAPTER 3
THE LOGISTICS/SUPPLY CHAIN PRODUCT
3
The 80-20 principle applies to sales and items where 80 percent of the dollar volume is
generated from 20 percent of the product items. While this ratio rarely holds exactly in
practice, the concept does. We can apply it to these data by ranking the products by
sales, and the percentage that the cumulative sales represent of the total. The following
table shows the calculations.

Product
code
08776
12121
10732
11693
10614
12077
07071
10542
06692
09721
14217
11007
Total

Dollar
sales
$71,000
63,000
56,000
51,000
46,000
27,000
22,000
18,000
14,000
10,000
9,000
4,000
$391,000

Cumulative
sales
$ 71,000
134,000
190,000
241,000
287,000
314,000
336,000
336,000
354,000
368,000
378,000
391,000

Cumulative
sales as
% of total
18.2
34.3
48.6
61.6
73.4
80.3
85.9
90.5
94.1
96.7
98.9
100.0

Cumulative
items as
% of total
8.3
16.7
25.0
33.3
41.7
50.0
58.3
66.7
75.0
83.3
91.7
100.0

The 80-20 rule cannot be applied exactly, since the cumulative percent of items does
not break at precisely 20 percent. However, we might decide that only products 08776
and 12121 should be ordered directly from vendors. The important principle derived
from the 80-20 rule is that not every item is of equal importance to the firm, and that different channels of distribution can be used to handle them. The 80-20 rule gives some
rational basis for deciding which products should be shipped directly from vendors and
which are more economically handled through a system of warehouses.
6
(a) Reading the ground transport rates for the appropriate zone as determined by zip code
and the weight of 27 lb. (rounding upward of 26.5 lb.) gives the following total cost
table for the four shipments.

To
zip code
11107
42117
74001
59615
a

Catalog
price
$99.95
99.95
99.95
99.95

UPS
zone
2
5
6
8

Transport
costa
$ 7.37
10.46
13.17
18.29

Total
cost
$107.32
110.41
113.12
118.24

Use 27 lb.

(b) The transport rate structure is reasonably fair, since ground rates generally follow
distance and size of shipment. These are the factors most directly affecting transport
costs. They are not fair in the sense that customers within a zone are all charged the
same rate, regardless of their distance from the shipment origin point. However, all
customers may benefit from lower overall rates due to this simplified zone-rate
structure.
10
(a) This is a delivered pricing scheme where the seller includes the transport charges in
the product price. The seller makes the transport arrangements.
(b) The seller prices the product at the origin, but prepays any freight charges; however,
the buyer owns the goods in transit.
(c) This is a delivered pricing scheme where the freight charges are included in the
product price, however the freight charges are then deducted from the invoice, and
the seller owns the goods in transit.
(d) The seller initially pays the freight charges, but they are then collected from the buyer
by adding them to the invoice. The buyer owns the goods in transit, since the pricing
is f.o.b. origin.
(e) The price is f.o.b. origin. The buyer pays the freight charges and owns the goods in
transit.
Regardless of the price policy, the customer will ultimately pay all costs. If a firm
does not consider outbound freight charges, the design of the distribution system will be
different than if it does. Since pricing policy is an arbitrary decision, it can be argued
that transport charges should be considered in decision making, whether the supplying
firm directly incurs them or not.
11
This shows how Pareto's law (80-20 principle) is useful in estimating inventory levels
when a portion of the product line is to be held in inventory. An empirical function that
approximates the 80-20 curve is used to estimate the level of sales for each product to be
held in inventory. According to Equation 3-2, the constant A is determined as follows.

X (1 Y ) 0.25(1.75)

0125
.
YX
0.75 0.25

The 80-20 type curve according to Equation 3-1 is:


Y

(1 A) (1 0125
. )X

A X
0125
.
X

This formula can be used to estimate the cumulative sales from the cumulative item
proportion. For example, item 1 is 0.05 of the total number of items (20) so that:

(1 0125
. )( 0.05)
0.321
0125
.
0.05

Of the $2,600,000 in total annual warehouse sales, item 1 should account for
0.3212,600,000 = $835,714.
By applying this formula to all items, the following inventory investment table can be
developed which shows sales by item. The average inventory investment by item is
found by dividing the turnover ratio into the item sales. The sum of the average
inventory value for each item gives a total projected inventory of $380,000.
Inventory Investment Table
Product
1
2
3
4
5
6
7
8
9
10
11
12
13
14
15
16
17
18
19
20

Cumulative
item proportion, X
0.05
0.10
0.15
0.20
0.25
0.30
0.35
0.40
0.45
0.50
0.55
0.60
0.65
0.70
0.75
0.80
0.85
0.90
0.95
1.00

Cumulative
sales, Y
$ 835,714
1,300,000
1,595,454
1,800,000
1,950,000
2,064,705
2,155,263
2,228,571
2,289,130
2,340,000
2,383,333
2,420,689
2,453,226
2,481,818
2,507,142
2,529,719
2,550,000
2,568,293
2,584,884
2,600,000

Projected
item sales
$ 835,714
464,286
295,454
204,546
150,000
114,706
90,558
73,308
60,559
50,870
43,333
37,356
32,537
28,592
25,324
22,587
20,271
18,293
16,591
15,116

Turnover
ratio
8
8
8
8
6
6
6
6
6
6
4
4
4
4
4
4
4
4
4
4
Total

Average
inventory
value
$104,464
58,036
36,932
25,568
25,000
19,118
15,093
12,218
10,093
8,478
10,833
9,339
8,134
7,148
6,331
5,647
5,068
4,473
4,148
3,779
$380,000

12
This problem involves the application of Equations 3-1 and 3-2. We can develop an 8020 curve based on 30 percent of the items accounting for 70 percent of sales. That is,

X (1 Y ) 0.30(1 0.70)

0.225
0.70 0.30
YX

Therefore, the sales estimating equation is:


Y

(1 0.225) X
0.225 X

By applying this estimating curve, we can find the sales of A and B items. For
example, 20 percent of the items, or 0.220 = 4 items, will be A items with a cumulative
proportion of sales of:
YA

(1 0.225)( 0.20)
0.5765
0.225 0.20

and 3,000,0000.5765 = 1,729,412.


The A+B item proportion will be:
YA B

(1 0.225)( 0.50)
0.8448
0.225 0.50

and 3,000,0000.8448 = 2,534,400. The product group B sales will A+B sales less A
sales, or 2,534,400 1,729,412 = $804,988.
The product group C will be the remaining sales, but these are not of particular
interest in this problem.
The average inventories for A and B products are found by dividing the estimated
sales by the turnover ratio. That is,
A:
B:

1,729,412/9
804,988/5
Total inventory

= 192,157
= 160,988
353,155 cases

The total cubic footage required for this inventory would be 353,1551.5 = 529,732
cu. ft. The total square footage for products A and B is divided by the stacking height.
That is, 529,731/16 = 33,108 sq. ft.

13
This problem is an application of Equations 3-1 and 3-2. We first determine the constant
A. That is,

X (1 Y ) 0.20(1 0.65)

0156
.
0.65 0.20
YX

and
0.75

(1 0156
. )X
0156
.
X

Solving algebraically for X, we have:


X

AxY
0156
. x 0.75

0.288
1 A Y 1 0156
.
0.75

That is, about 29% of the items (0.2885,000 = 1,440 items) produce 75% of the sales.
14
The price would be the sum of all costs plus an increment for profit to place the
automotive component in the hands of the customer.
This would be
25+10+5+8+5+transportation cost, or 53+T. Based on the varying transportation cost,
the following price schedule can be developed.

Quantity
1 to 1,000 units
1,001 to 2,000 units
>2,000 units
a

Price per unit


53+5=$58
53+4.00=57
53+3.00=56

Discount
0
1.7%a
3.5%

[(58 - 57)/58][100]=1.7%

CHAPTER 4
LOGISTICS/SUPPLY CHAIN CUSTOMER SERVICE
6
(a) This company is fortunate to be able to estimate the sales level that can be achieved at
various levels of distribution service. Because of this, the company should seek to
maximize the difference between sales and costs. These differences are summarized
as follows.

Percent of orders delivered


within 1 day
Contribution to 50 60 70 80 90 95 100
profit
-1.8 2.0 3.5 4.0 3.4 2.8 -2.0
The company should strive to make deliveries within 1 day 80 percent of the time for
a maximum contribution to profit.
(b) If a competing company sets its delivery time so that more than 80 percent of the
orders are delivered in 1 day and all other factors that attract customers are the same,
the company will lose customers to its competitor, as the sales curve will have shifted
downward. Cleanco should adjust its service level once again to the point where the
profit contribution is maximized. Of course, there is no guarantee that the previous
level of profits can be achieved unless the costs of supplying the service can
correspondingly be reduced.
7
(a) This problem solution requires some understanding of experimental design and
statistical inference, which are not specifically discussed in the text. Alert the
students to this.
The first task is to determine the increase in sales that can be attributed to the
change in the service policy. To determine if there is a significant change in the
control group, we set up the following hypothesis test.
z

X 2 X1
s22
s2
1
N 2 N1

224 185
612 79 2

102 102

39
.
394
36.48 6118
.

Now, referring to a normal distribution table in Appendix A of the text, there is a


significant difference at the 0.01 level in the sales associated with the control group.
That is, some factors other than the service policy alone are causing sales to increase.
Next, we analyze the test group in the same manner.

2,295 1,342
576 2 3352

56
56

953
10.7
5,924 2,004

This change is also significant at the 0.01 level.


The average increase in sales for the control group is 224/185 = 1.21, or 21%.
The average sales increase in the test group is 2295/1342 = 1.71, or 71%. If we
believe that 21% of the 71% increase in the test group is due to factors other than
service policy, then 71 21 = 50% was the true service effect. Therefore, for each
sales unit, an incremental increase in profit of (0.4095)(0.50) = $19 can be realized.
Since the cost of the service improvement is $2, the benefit exceeds the cost. The
service improvement should be continued.
Note: If the students are not well versed in statistical methodology, you may wish
to instruct them to consider the before and after differences in the mean values of
both groups as significant. The solution will be the same.
(b) The use of the before-after-with-control-group experimental design is a methodology
that has been used for some time, especially in marketing research studies. The
outstanding feature of the design is that the use of the control group helps to isolate
the effect of the single service variable. On the other hand, there are a number of
potential problems with the methodology:
The sales distributions may not be normal.
The time that it takes for diffusing the information that a service change has taken
place may distort the results.
The products in the control group may not be mutually exclusive from those in the
test group.
The method only shows the effect of a single step change in service and does not
develop a sales-service relationship.
It may not always be practical to introduce service changes into on-going
operations to test the effect.
8
(a) The optimum service level is set at that point where the change in gross profit equals
the change in cost.

The change in gross profit:


P = Trading margin Sales response rate Annual sales
= 1.000.0015100,000
= $150 per year per 1% change in the service level
The change in cost:
C = Annual carrying cost Standard product cost z
10

Demand standard deviation for order cycle


= 0.3010.00400z
Now, set P = C and solve for z.
150 = 1200z
z = 0.125
From the tabulated changes in service level with those changes in z, the service level
should be set between 96-97%.
(b) The weakest link in this analysis is estimating the effect that a change in service will
have on revenue. This implies that a sales-service relationship is known.
9
The methodology is essentially the same as that in question 7, except that we are asked to
find X instead of Y. That is,

P = 0.750.001580,000
= 90
and
C = 0.251,000500z
= 1250z
Then,
P= C
90 = 1250z
z = 0.072
From the normal distribution (see Appendix A), the z for an area under the curve of
93% is 1.48, and for 92%, z is 1.41. Since the difference of 1.48 1.41 = 0.07, we can
conclude that the in-stock probability should be set at 92-93%. Of course, the change in z
is found by taking the difference in z values for 1% differences in the area values under
the normal distribution curve for a wide range of area percentages.
10
Apply Taguchis concept of the loss function. First, estimate the loss per item if the
target level of service is not met. We know the profit per item as follows.

11

Sales price
Cost of item
Other costs
Profit per item

$5.95
-4.25
-0.30
$1.40

Since one-half of the sales are lost, the opportunity loss per item would be
Profit per item

Sales lost
$1.40 (1/2)(880)
Opportunity loss
$0.70/item
880
Current sales

Next, find k in the loss function.


L k ( y m) 2
0.70 k (10 5)2
0.70 k ( 25)
k 0.03

out-of-stock % at point where sales are lost


Target %

Finally, the point where the marginal supply cost equals the marginal sales loss is
( y 5)

B
0.10

1.67%
2k 2(0.03)

y 1.67 5 6.67%
The retailer should not allow the out-of-stock percentage to deviate more than 1.67%,
and should not allow the out-of-stock level to fall below 1.67 + 5 = 6.67%.

12

CHAPTER 5
ORDER PROCESSING AND
INFORMATION SYSTEMS

All questions in this chapter require individual judgment and response. No answers are
offered.

13

CHAPTER 6
TRANSPORT FUNDAMENTALS
14
The maximum that the power company can pay for coal at its power plant location in
Missouri is dictated by competition. Therefore, the landed cost at the power plant of coal
production costs plus transportation costs cannot exceed $20 per ton. Since western coal
costs $17 per ton at the mine, the maximum worth of transportation is $20 $17 = $3 per
ton. However, if the grade of coal is equal to the coal from the western mines, eastern
coal can be landed in Missouri for $18 per ton. In light of this competitive source,
transportation from the western mines is worth only $18 $17 = $1 per ton.
15
Prior to transport deregulation, it was illegal for a carrier to charge shippers less for the
longer haul than for the shorter haul under similar conditions when the shorter haul was
contained within the longer one. To be fair, the practice probably should be continued.
If competitive conditions do not permit an increase in the rate to Z, then all rates that
exceed $1 per cwt. on a line between X and Z should not exceed $1 per cwt. Therefore,
the rate to Z is blanketed back to Y so that the rate to Y is $1 per cwt. By blanketing the
rate to Z on intervening points, no intervening point is discriminated against in terms of
rates.
16
(a) From text Table 6-4, the item number for place mats is 4745-00. For 2,500 lb., the
classification is 100 since 2,500 lb. is less than the minimum weight of 20,000 lb. for
a truckload shipment. From text Table 6-5, the rate for a shipment 2,000 lb. is
8727/cwt. The shipping charges are $87.27 25 cwt. = $2,181.75.

(b) This is an LTL shipment with a classification of 100, item number 4980-00 in text
Table 6-4. From Table 6-5, the minimum charge is 9351 and the rate for a <500 lb.
shipment is 5401/cwt. Check the charges using the <500 lb. rate and compare it to
the minimum charge. That is,
$54.01 1.5 cwt. = $81.02
Since this is less than the minimum charge of $93.51, pay the minimum charge.
(c) From Table 6-4, the item number is 2055-00 with a classification of 55 for LTL and
37.5 for TL at a minimum weight of 36,000 lb. There are three possibilities that need
to be examined:
(1) Ship LTL at class 55 and 27,000 lb. shipment.
(2) Ship at class 55 and 30,000 lb. rate.
(3) Ship at class 37.5 and 36,000 lb. rate.

14

Try (1): Rate is $5.65/cwt. 5.65 270 = $1,525.50.


Try (2): Rate is $3.87/cwt. 3.87 300 = $1,161.00

Lowest cost

Try (3): Rate is $3.70/cwt. 3.70 360 = $1,332.00


(d) The shipment is a truckload classification (2070-00) of 65. The rate at 30,000 lb. is
$4.21/cwt. The charges are 4.21 300 = $1,263.00.
(e) Classification of this product is 55 (4860-00) for a truckload of 24,000 lb. Check the
break weight according to Equation 6-1.
Break weight =

3.87 30,000
20,549 lb.
5.65

Since current shipping weight of 24,000 lb. exceeds the break weight, ship as if 30,000
lb. Hence, 3.87 300 = $1,161.00. Now, discount the charges by 40 percent. That is,
$1,161 (1 0.40) = $696.60
21
The question involves evaluating two alternatives. The first is to compute the transport
charges as if there are three separate shipments. The next is to see if a stop-off privilege
offers any cost reduction. The comparison is shown below.
Separate shipments

Loading/unloading
22,000
3,000
15,000

Route
A to D
A to C
B to C

Rate, Stop-off
$/cwt. charge
Charges
$3.20
--$704.00
2.50
--75.00
1.50
--225.00
Total charges $1,004.00

With stop-off
Ship direct to B and split deliver thereafter.

Rate, Stop-off
Loading/unloading Route $/cwt. charge
Charges
25,000
A to B $1.20
$ 300.00
40,000
B to D 2.20
880.00
Stop-off @ C
$25.00
25.00
Stop-off @ D
25.00
25.00
Total charges $1,230.00

Direct shipment

15

Split deliver at all stops.


Rate, Stop-off
Loading/unloading Route $/cwt charge
Charges
40,000
A to D 3.20
1,280.00
Stop-off @ B
25.00
25.00
Stop-off @ C
25.00
25.00
Stop-off @ D
25.00
25.00
Total charges $1,335.00
Other combinations may be tried. In this case, there appears to be no advantage to using
the stop-off privilege.

16

CHAPTER 7
TRANSPORT DECISIONS
1
Selecting a mode of transportation requires balancing the direct cost of transportation
with the indirect costs of both vendor and buyer inventories plus the in-transit inventory
costs. The differences in transport mode performance affect these inventory levels, and,
therefore, the costs for maintaining them, as well as affect the time that the goods are in
transit. We wish to compare these four cost factors for each mode choice as shown in
Table 7-1 of the manual. The symbols used are:

R = transportation rate, $/unit


D = annual demand, units
C = item value at buyer's inventory, $
C' = item value at vendor's inventory, $
T = time in transit, days
Q = Shipping quantity, units
Rail has the lowest total cost.
TABLE 7-1

An Evaluation of the Transport Alternatives for the Wagner


Company

Cost type
Transport

Method
RD

Rail
2550,000
= $1,250,000
In-transit
0.2547550,000
inventorya
ICDt/365
(16/365)
= $260,274
Wagers
0.25475(10,000/2)
inventorya
ICQ/2
= $593,750
Electronics
0.25500(10,00/2)
inventory
ICQ/2
= $625,000
Total
$2,729,024
a
C refers to price less transport cost per unit.

Piggyback
4450,000
= $2,200,000
0.2545650,000
(10/365)
= $156,164
0.25456(7,000/2)
= $399,000
0.25500(7,000/2)
= $437,500
$3,192,664

Truck
8850,000
= $4,400,000
0.2541250,000
(4/365)
= $56,438
0.25412(5,000/2)
= $257,500
0.25500(5,000/2)
= $312,500
$5,026,438

2
As in question 1, this problem is one of balancing transport costs with the indirect costs
associated with inventories. However, in this case we must account for the variability in
transit time as it affects the warehouse inventories. We can develop the following
decision table.

Service type

17

Cost type

Method

Transport

RD

In-transit
inventory
Plant
inventory

ICDt/365

Warehouse
inventory
Total

ICQ*/2

ICQ*/2
+ ICr

A
129,600
= $115,200
0.20509,600
(4/365)
= $1,052
0.3050(321.8/2)
= $2,684
0.3062(321.3/2)
+ 0.306250.5
= $3,927
$122,863

B
11.809,600
=$114,048
0.20509,600
(5/365)
= $1,315
0.3050(357.8/2)
= $2,684
0.3061.80(321.8/2)
+ 0.3061.8060.6
= $4,107
$122,154

Recall that Q* 2 DS / IC 2(9,600 )(100 ) / 0.3(50 ) 357.8 cwt. for the plant, assuming
the order cost is the same at plant and warehouse. However, for the warehouse, we must
account for safety stock (r) and for the transportation cost in the value of the product.
Therefore,
For A:
Q * 2 DS / IC 2(9,600)(100) / 0.3( 62) 3213
. cwt.

and for z = 1.28 for an area under the normal distribution of 0.90, the safety stock is:
r zs LT ( d ) 1.28 1.5 (9,600 / 365) 50.5 cwt.

For B:
Q * 2(9,600 )(100 ) / 0.3( 6180
. ) 3218
. cwt.

and

r 1.28 1.8 (9,600 / 365) 60.6 cwt.


Service B appears to be slightly less expensive.
3
The shortest route method can be applied to this problem. The computational table is
shown in Table 7-2. The shortest route is defined by tracing the links from the
destination node. They are shown in Table 7-2 as A D F G for a total distance of
980 miles.
TABLE 7-2

Tabulation of Computational Steps for the Shortest Route Method


Applied to Transcontinental Trucking Company Problem

18

Solved nodes
directly
Its closest
connected to
connected
unsolved
unsolved
Step
nodes
node
1
A
B
A
D
2
A
D
B
C
3
B
C
D
C
D
F
4
C
E
C
F
D
F
5
C
F
E
G
D
F
6
E
G
F
G
a
Asterisk indicates the shortest route

Total time
involved
186 mi.
276
276
186+110= 296
186+110= 296
276+ 58= 334
276+300= 576
296+241= 537
296+350= 646
276+300= 576
296+350= 646
537+479=1016
276+300= 576
537+479=1016
576+404= 980

nth nearest
node
B

Its minimum
time
186 mi.

Its last
connectiona
AB

276

AD*

296

BC

537

CE

576

DF*

980

FG*

4
In this actual problem, the U.S. Army used the transportation method of linear
programming to solve its allocation problem. The problem can be set up in matrix form
as follows:

Origin
Destination
Letterkenny
Fort Hood
Fort Riley

Cleveland
150
150
325
50
275
100
375

South
Charleston
100
150
350

San
Jose
800

325

350

Demand
300

300
50

100
400

275

Fort Carson

100

300

100

100

450

250

Fort Benning

100

Supply

400

150

100
150

The cell values shown in bold represent the number of personnel carriers to be moved
between origin and destination points for minimum transportation costs of $153,750. An
alternative solution at the same cost would be:

19

Origin
Cleveland
S. Charleston
Cleveland
San Jose
Cleveland
San Jose
Cleveland

Destination
Letterkenny
Letterkenny
Fort Hood
Fort Hood
Fort Riley
Fort Carson
Fort Benning

Number of
carriers
150
150
50
50
100
100
100

5
This problem can be used effectively as an in-class exercise. Although the problem
might be solved using a combination of the shortest route method to find the optimum
path between stops and then a traveling salesman method to sequence the stops, it is
intended that students will use their cognitive skills to find a good solution. The class
should be divided into teams and given a limited amount of time to find a solution. They
should be provided with a transparency of the map and asked to draw their solution on it.
The instructor can then show the class each solution with the total distance achieved.
From the least-distance solutions, the instructor may ask the teams to explain the logic of
their solution process. Finally, the instructor may explore with the class how this and
similar problems might be treated with the aid of a computer.
Although the question asks the student to use cognitive skills to find a good route, a
route can be found with the aid of the ROUTER software in LOGWARE. The general
approach is to first find the route in ROUTER without regard to the rectilinear distances
of the road network. Because this may produce an infeasible solution, specific travel
distances are added to the database to represent actual distances traveled or to block
infeasible paths from occurring. A reasonable routing plan is shown in Figure 7-1 and
the ROUTER database that generates it is given in Figure 7-2. The total distance for the
route is 9.05 miles and at a speed of 20 miles per hour, the route time is approximately 30
minutes.

0.5

1.0

1.5

2.0

0
19

20

21

17

20

FIGURE 7-2 Input Data for ROUTER for School Bus Routing Problem
PARAMETERS AND LABELS
Problem label School Bus Routing Exercise
Grid corner with 0,0 coordinates (NW, SW, SE, or NE) - NW
DEPOT DATA
Depot description - Atlanta Located in zone - 0
Horizontal coordinate 0.14 Vertical coordinate 0.45
Earliest starting time (min) - 0 Latest return time (min) - 9999
Default vehicle speed (miles per hour) - 20
After how many clock hours will overtime begin - 9999
GENERAL DATA
Percent of vehicle in use before allowing pickups - 0
Horizontal scaling factor - 1 Vertical scaling factor - 1
Maximum TIME allowed on a route (hours) - 9999
Maximum DISTANCE allowed on a route (miles) - 9999
LOAD/UNLOAD TIME FORMULA
Fixed time per stop - 0
Variable time per stop by weight - 0 By cube - 0
BREAK TIMES
Duration of 1st break (minutes) - 0 To begin after - 9999
Duration of 2nd break (minutes) - 0 To begin after - 9999
Duration of 3rd break (minutes) - 0 To begin after - 9999
Duration of 4th break (minutes) - 0 To begin after - 9999

21

--STOP DATA
NO
1
2
3
4
5
6
7
8
9
10
11
12
13
14
15
16
17
18
19
20
21

STOP
LOAD VOL.
LOAD
DESCRIPTION TY WGHT CUBE HCRD VCRD ZN TIME BEG1 END1
Stop 1
D
1
0
0.14
0.80
0
0
0 9999
Stop 2
D
1
0
0.14
1.14
0
0
0 9999
Stop 3
D
1
0
0.14
1.31
0
0
0 9999
Stop 4
D
1
0
0.35
1.31
0
0
0 9999
Stop 5/22
D
1
0
0.52
0.61
0
0
0 9999
Stop 6
D
1
0
0.58
1.31
0
0
0 9999
Stop 7
D
1
0
0.80
1.31
0
0
0 9999
Stop 8
D
1
0
1.03
0.61
0
0
0 9999
Stop 9
D
1
0
1.03
0.96
0
0
0 9999
Stop 10
D
1
0
1.03
1.31
0
0
0 9999
Stop 11
D
1
0
1.36
1.31
0
0
0 9999
Stop 12
D
1
0
1.48
1.31
0
0
0 9999
Stop 13
D
1
0
1.80
1.31
0
0
0 9999
Stop 14
D
1
0
1.87
1.31
0
0
0 9999
Stop 15
D
1
0
1.84
0.61
0
0
0 9999
Stop 16
D
1
0
1.95
0.61
0
0
0 9999
Stop 17
D
1
0
1.29
0.10
0
0
0 9999
Stop 18
D
1
0
1.26
0.61
0
0
0 9999
Stop 19
D
1
0
1.15
0.10
0
0
0 9999
Stop 20
D
1
0
0.69
0.23
0
0
0 9999
Stop 21
D
1
0
0.14
0.26
0
0
0 9999

BEG2
9999
9999
9999
9999
9999
9999
9999
9999
9999
9999
9999
9999
9999
9999
9999
9999
9999
9999
9999
9999
9999

END2
9999
9999
9999
9999
9999
9999
9999
9999
9999
9999
9999
9999
9999
9999
9999
9999
9999
9999
9999
9999
9999

VEHICLE DATA
--CAPACITY--

NO.
1

VEHICLE
DESCRIPTION
Bus

TP
1

NO
1

WGHT
9999

CUBE
9999

--VEHICLE-FIXED
COST
0

PER MI
COST
0

--DRIVER-FIXED
COST
0

PER HR
COST
0

OVER
TIME
COST
0

SPECIFIED STOP-TO-STOP DISTANCES

NO
1
2
3
4
5
6
7
8
9
10
11
12
13
14
15
16
17
18
19

STOP
NO.
14
14
15
16
18
19
19
19
19
9
9
9
5/22
5/22
5/22
5/22
20
20
20

STOP
DESCRIPTION
Stop 14
Stop 14
Stop 15
Stop 16
Stop 18
Stop 19
Stop 19
Stop 19
Stop 19
Stop 9
Stop 9
Stop 9
Stops 5&22
Stops 5&22
Stops 5&22
Stops 5&22
Stop 20
Stop 20
Stop 20

STOP
NO.
16
15
17
17
9
8
20
5/22
18
20
19
21
1
21
20
9
21
0
5/22

STOP
DISTANCE
DESCRIPTION IN MILES
Stop 16
0.78
Stop 15
0.90
Stop 17
1.06
Stop 17
1.18
Stop 9
0.58
Stop 8
0.76
Stop 20
0.59
Stops5&22
1.14
Stop 18
0.53
Stop 20
1.08
Stop 19
1.11
Stop 21
1.69
Stop 1
0.56
Stop 21
1.05
Stop 20
1.14
Stop 9
0.97
Stop 21
0.84
School
1.03
Stops 5&22
0.55

22

20
21
22

17
0
2

Stop 17
School
Stop 2

0
School
5/22 Stops 5&22
5/22 Stops 5&22

2.43
1.37
1.03

6
Strategy 1 is to stay at motel M2 and serve the two routes on separate days. Using the
ROUTESEQ module in LOGWARE gives us the sequence of stops and the coordinate
distance. The routes originating at M2 would be:

Distancea
95.55 mi.
86.45
182.00 mi.

Route Stop sequence


1
8,6,1,4,2,3,5,7,9
2
10,13,14,17,18,16,12,15,11
a

Includes map scaling factor

The total cost of this strategy would be:


Motel 3 nights @ 49.00
Travel 182 miles @ $.30/mi.
Total

$147.00
54.60
$201.60

Strategy 2 is a mixed strategy involving staying at motels closest to the center of the
stops clusters. The route sequences from different motels are:
Route Stop sequence
Distance
1
4,2,3,5,7,9,8,6,2
98.50 mi.
2
18,17,13,14,10,11,15,12,16 80.30
178.80 mi.
The total cost of this strategy is:
Motel M1 1st night
$ 40.00
M1 2nd night
40.00
rd
M1 3 night
45.00
Travela 214.80 mi. @ 0.30/mi. 64.44
Total
$189.44
a

178.80 + 36 = 214.80

Strategy 2 appears to be most economical.


7
(a) Since distances are asymmetrical, we cannot use the geographically based traveling
salesman method in LOGWARE. Rather, we use a similar module in STORM that
allows such asymmetrical matrices, or the problem is small enough to be solved by
inspection. For this problem, the minimal cost stop sequence would be:

23

BakeryStop 5Stop 3Stop 4Stop 2Stop 1Bakery


with a tour time of 130 minutes.
(b) Loading/unloading times may be added to the travel times to a stop. The problem
may then be solved as in part a.
(c) The travel times between stop 3 and all other nodes are increased by 50%. The
remaining times are left unchanged. Optimizing on this matrix shows no change in
the stop sequence. However, the tour time increases to 147.50 minutes.
8
This may be solved by using the ROUTER module in LOGWARE. The screen set up for
this is as follows.

24

FIGURE 7-3 Input Data for ROUTER for Sima Donuts


--PARAMETERS AND LABELS
Problem label - Sima Donuts
Grid corner with 0,0 coordinates (NW, SW, SE, or NE) - NE
DEPOT DATA
Depot description - Atlanta Located in zone - 0
Horizontal coordinate - 2084 Vertical coordinate - 7260
Earliest starting time (min) - 180 Latest return time (min) - 9999
Default vehicle speed (miles per hour) - 45
After how many clock hours will overtime begin - 168
GENERAL DATA
Percent of vehicle in use before allowing pickups - 0
Horizontal scaling factor - 0.363 Vertical scaling factor - 0.363
Maximum TIME allowed on a route (hours) - 40
Maximum DISTANCE allowed on a route (miles) - 1400
LOAD/UNLOAD TIME FORMULA
Fixed time per stop - 0
Variable time per stop by weight - 0 By cube - 0
BREAK TIMES
Duration of 1st break (minutes) - 60 To begin after - 720
Duration of 2nd break (minutes) - 60 To begin after - 1200
Duration of 3rd break (minutes) - 60 To begin after - 2160
Duration of 4th break (minutes) - 60 To begin after - 2640
--STOP DATA
STOP
NO DESCRIPTION
1 Tampa FL
2 Clearwater FL
3 Daytona Beach F
4 Ft Lauderdale FL
5 N Miami FL
6 Oakland Park FL
7 Orlando FL
8 St Petersburg FL
9 Tallahassee FL
10 W Palm Beach F
11 Puerto Rico

TY
D
P
D
D
D
P
D
P
D
D
D

LOAD
WGHT
20
14
18
3
5
4
3
3
3
3
4

VOL.
CUBE
0
0
0
0
0
0
0
0
0
0
0

HCRD VCRD ZN
1147
8197
0
1206
8203
0
1052
7791
0
557
8282
0
527
8341 0
565
8273
0
1031
7954
0
1159
8224
0
1716
7877
0
607
8166
0
527
8351
0

LOAD
TIME
15
45
45
45
45
45
45
45
15
45
45

BEG1
360
360
360
180
360
180
180
180
600
360
360

END1 BEG2
1440
1800
1440
1800
1440
1800
1440
1800
1440 1800
1440
1800
1440
1800
1440
1800
1440
1800
1440
1800
1440
1800

END2
2880
2880
2880
2880
2880
2880
2880
2880
2880
2880
2880

--VEHICLE DATA
-CAPACITY--

NO.
1
2
3

VEHICLE
DESCRIPTION
Truck #1-20
Truck #2-25
Truck #3-30

TP
1
2
3

NO
3
1
1

WGHT
20
25
30

CUBE
9999
9999
9999

--VEHICLE-FIXED
COST
0
0
0

PER MI
COST
1.30
1.30
1.30

--DRIVER-FIXED
COST
0
0
0

PER HR
COST
0
0
0

OVER
TIME
COST
0
0
0

Making a run with ROUTER will give the route design.

25

Pickup

Pickup

FIGURE 7-4 Graphical Display of Route Design for Sima Donuts

The route design involves 3 routes for a total distance of 3,830 miles, a cost of $4,978.71,
and a total time of 100.4 hours. The route details are as follows:
Route #1 with 20-pallet truck
Depot
Start time 3:00AM of day 1
Daytona Beach Deliver 18 pallets
Clearwater
Pickup 14 pallets
Depot
Return time 5:48AM of day 2
Route #2 with 20-pallet truck
Depot
Start time 3:00AM of day 1
Orlando
Deliver 3 pallets
W Palm Beach
Deliver 3 pallets
Ft Lauderdale
Deliver 3 pallets
N Miami
Deliver 5 pallets
Miami-Puerto R. Deliver 4 pallets
Depot
Return time 4:43PM of day 2

26

Route #3 with 30-pallet truck


Depot
Start time 4:13AM of day 1
Tallahassee
Deliver 3 pallets
Tampa
Deliver 20 pallets
St Petersburg
Pickup 3 pallets
Oakland Park
Pickup 4 pallets
Depot
Return time 4:03PM of day 2
9
Given sailing times and dates when deliveries are to be made, loadings need to be
accomplished no later than the following dates:

To: A B C D
From: 1 16 40 1
2 69 25 5
The problem can be expressed as a transportation problem of linear programming. There
will be 6 initial states [(1,1), (2,5), (1,16), (2,25), (1,40), and (2,69)] and 6 terminal states
[(D,10), (C,15), (A,36), (B,39), (C,52), and (A,86)]. The linear program is structured as
shown in Figure 7-4.
Using a transportation solution method, we determine one of the optimum solutions.
There are several. The solution is read by starting with the slack on initial loading state
1. This tells us to next select the cell of terminal state 1. In turn, this defines initial state
3 and hence terminal state 3. And so it goes until we reach the terminal state slack
column. This procedure is repeated until all initial state slacks are exhausted. Our
solution
shows
two
routings.
The
first
is
(1,1)(D,10)(1,16)(A,36)(2,69)(A,86).
The
second
is
(2,5)(C,15)(2,25)(B,39)(1,40)(C,52). Two ships are needed.

27

1
1

Load
date
Discharge
date
100

100

XXa
100

C 15

2
69

XX

A 36

Rim restriction

XX

B 39

10

100

100

10

10

10

100

10

XX
XX

XX
100

XX
100

XX

XX
10

1
100

XX

XX
100

10

XX
100

XX
10

100

XX

XX
100

10

100

XX

A 86

XX
100

100

10

100

XX

C 52

XX
100

100

XX
100

100

XX
100

100

1
40

Slack

D 10

Slack
Rim restriction

Loading points and dates


1
2
16
25

2
5

XX
10

XX
10

10

1
1

XX inadmissible cells given a high cost

FIGURE 7-5 Transportation Matrix Setup and Solution for the Queens Lines
Tanker Scheduling Problem
10
This is a problem of freight consolidation brought about by holding orders so they can be
shipped with orders from subsequent periods. The penalty associated with holding the
orders is a lost sales cost.

(i) Orders shipped as received


Weight
Rate
a
Hays
10,000 0.0519
Manhattan
14,000 0.0519
Salina
13,000 0.0408
Great Bend
10,000 0.0498
Transportation
Lost sales
Total
a

=
=
=
=
=

Cost
$519.00
726.00
530.00
498.00
$2,274.00
.00
$2,274.00

Ship 8,000 lb. as if 10,000 lb.

Average period cost is $2,274.00

28

(ii) Consolidate first period orders with second period orders.


Weight
Rate
Cost
=
Hays
16,000 0.0519
=
$830.40
a
40,000 0.0222
=
888.00
Manhattan
Salina
26,000 0.0342
=
889.20
Great Bend
10,000 0.0498
=
498.00
Transportation
$3,105.60
Lost sales
1,050.00
Total
$4,155.60
a

Ship 28,000 lb. as if 40,000 lb.

The lost sales cost is 1,000 cases $1.05 = $1,050.00 to hold one group of orders for 2
weeks.
Average cost per period is $4,155.60/2 = $2,077.80.
(iii) Hold all orders until the third period.
Weight
Rate
Hays
24,000 0.0426
Manhattan
42,000 0.0222
40,000 0.0246
Salinaa
Great Bend
15,000 0.0498
Transportation
Lost sales
Total
a

=
=
=
=
=

Cost
$1,022.40
932.40
984.00
747.00
$3,685.80
3,150.00
$6,835.80

Ship as if 40,000 lb.

Lost sales
Hold 1st period orders for 2 periods 1,0001.05.2 = $ 2,100
Hold 2nd period sales for 1 period 1,0001.05 = 1,050
$ 3,150
Average period cost is $6,835.80/3 = $2,278.60
Summary
Ship immediately
$ 2,274.00
Hold orders 1 period
2,077.80
Hold orders 2 periods
2,278.60

Optimum

11
Routes are built by placing the trips end-to-end throughout the day from 4AM until
11PM, respecting the times that a warehouse can receive a shipment. This is a 19-hour
block of time per day, or there are 95 hours per week per truck in which a truck may

29

operate. If there were no delivery time restrictions on warehouses and trips could be
placed end-to-end for a truck without any slack at the end of the day, the absolute
minimum number of trucks can be found multiplying the number of trips by the route
time and then dividing the total by the 95 hours allowed per week. That is,
Warehouse
location
Flint
Alpena
Saginaw
Lansing
Mt. Pleasant
W. Branch
Pontiac
Traverse City
Petoskey

(1)
Number of
trips
43
5
8
21
12
5
43
6
5

(2)
Total time
per trip, hr.
1.25
10.50
2.25
3.75
5.50
6.00
2.75
10.50
11.75
Total

(3)=(1)(2)
Total time,
hr.
53.75
52.50
18.00
78.75
66.00
30.00
118.25
63.00
58.75
539.00

For 539 trip hours, 539/95 = 5.67 rounded to six trucks needed per week. Now, it is
necessary to adjust for the problem constraints. A good schedule can be found by
following a few simple rules that can be developed by examining the data. First, begin
the day with a trip where the driving time to a warehouse is just long enough for the truck
to arrive at the warehouse just after it opens. One-half the driving time should exceed
6:30 4:00 = 2:30, or 2 hr. Trips to Alpena, Traverse City, and Petoskey qualify.
Second, use the short trips at the end of the day to avoid slack time. Third, allocate the
trips to the days using the longest ones first. Make sure that the total trip time for a day
does not exceed 19 hours. For a minimum of six trucks, the following feasible schedule
can be developed by inspection.
Truck 1

Truck 2

Truck 3

Truck 4

Truck 5

Truck 6

Day 1
Petoskey 11.75
W Branch 6.00
Flint 1.25
Total =19.00 hr.
T. City 10.50
2 Lansing 7.50
Total =18.00 hr.
T. City 10.50
2 Lansing 7.50
Total =18.00 hr.
Alpena 10.50
Lansing 3.75
3 Flint 3.75
Total =18.00 hr.
M Pleasant 5.50
4 Pontiac 11.00
2 Flint 2.50
Total =19.00 hr.
M Pleasant 5.50
4 Pontiac 11.00
2 Flint 2.50
Total =19.00 hr.

Day 2
Petoskey 11.75
W Branch 6.00
Flint 1.25
Total =19.00 hr.
T. City 10.50
2 Lansing 7.50
Total =18.00 hr.
Alpena 10.50
2 Lansing 7.50
Total =18.00 hr.
M Pleasant 5.50
4 Pontiac 11.00
2 Flint 2.50
Total =19.00 hr.
M Pleasant 5.50
4 Pontiac 11.00
2 Flint 2.50
Total =19.00 hr.
M Pleasant 5.50
3 Pontiac 8.25
2 Flint 2.50
Total =16.25 hr.

Day 3
Petoskey 11.75
W Branch 6.00
Flint 1.25
Total =19.00
T. City 10.50
2 Lansing 7.50
Total =18.00
Alpena 10.50
2 Lansing 7.50
Total =18.00 hr.
M Pleasant 5.50
4 Pontiac 11.00
2 Flint 2.50
Total =19.00 hr.
M Pleasant 5.50
4 Pontiac 11.00
Flint 1.25
Total =17.75
M Pleasant 5.50
6 Saginaw 13.50

Day 4
Petoskey 11.75
5 Flint 6.25

Day 5
Petoskey 11.75
5 Flint 6.25

Total =18.00 hr.


T. City 10.50
2 Lansing 7.50
Total =18.00 hr.
Alpena 10.50
2 Lansing 7.50
Total =18.00 hr.
M Pleasant 5.50
4 Pontiac 11.00
2 Flint 2.50
Total =19.00 hr.
M Pleasant 5.50
4 Pontiac 11.00

Total =18.00 hr.


T. City 10.50
2 Lansing 7.50
Total =18.00 hr.
Alpena 10.50
2 Lansing 7.50
Total =18.00 hr.
M Pleasant 5.50
4 Pontiac 11.00
2 Flint 2.50
Total =19.00
M Pleasant 5.50
4 Pontiac 11.00

Total =16.50 hr.


W Branch 6.00
2 Saginaw 4.50

Total = 16.50
W Branch 6.00
10 Flint 12.50

Total =19.00 hr.

Total =10.50 hr.

Total =18.50 hr.

30

Although this schedule meets the requirements of the problem, it might be improved by
better balancing the workload across the trucks and the days.
12
(a) A sweep method solution is shown on the following figure. Five trucks are needed
with a total route distance of (30+29+39+44+19.5)10 = 1,615 miles.

20

18
Route #1
Load 19

Route #5
Load 9

2
5

4
3

14

Miles 12
x 10
10

Route #4
Load `8

2
3
Warehouse

5
4

6
4

16

Route #2
Load 20

Route #3
Load 17

2
4

0
0

10 12 14
Miles x 10

16

18

20

22

24

26

(b) The sweep method is a fast and relatively simple method for finding a solution to
rather complex vehicle routing problems. Solutions can be found graphically without
the aid of a computer. However, there are some limitations. Namely,

The method is heuristic and has an average error of about 10 to 15 percent. This
error is likely to be low if the problem contains many points and the weight of
each point is small relative to the capacity of the vehicle.
The method does not handle timing issues well, such as time windows.
Too many trucks may be used in the route design.

13
This problem may be solved with the aid of ROUTER in LOGWARE. The model input
data may be formatted as shown in Figure 7-6.

31

(a) The solution from ROUTER shows that four routes are needed with a minimum total
distance of 492 miles. The route design is shown graphically in Figure 7-7. A
summary for these routes is given in following partial output report.
Route
Route
no
1
2
3
4
Total

Run Stop
Brk Stem
time, time, time, time, time,
Start Return No of
Route
hr
hr
hr
hr
hr
time
time stops dist,Mi
1.2
1.0
.3
.0
.4 08:59AM 10:12AM
3
29
8.9
6.6
1.3
1.0
1.1 08:32AM 05:25PM
19
199
6.2
3.7
1.4
1.0
.9 08:42AM 02:54PM
14
112
7.5
5.1
1.5
1.0
1.4 08:30AM 04:02PM
12
152
23.8 16.4
4.4
3.0
3.8
48
492

Route
cost,$
.00
.00
.00
.00
.00

(b) Note that route #1 is short and that a driver and a station wagon would be used for a
route that takes 1.2 hours to complete. By attaching route #1 to route #3, the same
driver and station wagon may be used, and the constraints of the problems are still
met. The refilled station wagon can leave the depot by 3:30-3:45PM and still meet
the customers time windows and return to the depot by 6PM. Thus, only three
drivers and station wagons are actually needed for this problem.
FIGURE 7-6 Input Data for ROUTER for Medic Drugs
--PARAMETERS AND LABELS
Problem label - Medic Drugs
Grid corner with 0,0 coordinates (NW, SW, SE, or NE) - SW
DEPOT DATA
Depot description - Pharmacy Located in zone - 0
Horizontal coordinate - 13.7 Vertical coordinate - 21.2
Earliest starting time (min) - 480 Latest return time (min) - 9999
Default vehicle speed (miles per hour) - 30
After how many clock hours will overtime begin - 168
GENERAL DATA
Percent of vehicle in use before allowing pickups - 0
Horizontal scaling factor - 4.6 Vertical scaling factor - 4.6
Maximum TIME allowed on a route (hours) - 168
Maximum DISTANCE allowed on a route (miles) - 9999
LOAD/UNLOAD TIME FORMULA
Fixed time per stop - 0
Variable time per stop by weight - 0 By cube - 0
BREAK TIMES
Duration of 1st break (minutes) - 60 To begin after - 720
Duration of 2nd break (minutes) - 0 To begin after - 9999
Duration of 3rd break (minutes) - 0 To begin after - 9999
Duration of 4th break (minutes) - 0 To begin after - 9999
--STOP DATA
STOP
NO DESCRIPTION
1 Covington House
2 Cuyahoga Falls
3 Elyria

TY
D
D
D

LOAD
WGHT
1
9
1

VOL.
CUBE
0
0
0

HCRD VCRD ZN
23.40
12.90
0
13.40
13.40
0
6.30
16.80
0

LOAD
TIME
2
18
5

BEG1
540
540
540

END1 BEG2
1020
9999
1020
9999
1020
9999

END2
9999
9999
9999

32

46
47
48

Westbay
Westhaven
Broadfiels Mnr

D
D
D

6
2
6

0
0
0

8.40
8.50
18.20

18.00
18.10
22.90

0
0
0

10
5
2

630
540
540

690
1020
1020

9999
9999
9999

9999
9999
9999

--VEHICLE DATA
-CAPACITY--

NO.
1

VEHICLE
DESCRIPTION
Station wagon

TP
1

NO
50

WGHT
63

CUBE
9999

--VEHICLE-FIXED
COST
0

--DRIVER--

PER MI
COST
0

FIXED
COST
0

PER HR
COST
0

OVER
TIME
COST
0

FIGURE 7-7 Graphical


Solution for Medic Drugs

14
There is no exact answer to this problem nor is one intended. Several approaches might
be taken to this problem. We could apply the savings method or the sweep method to
solve the routing problem for each day of the week, given the current demand patterns.
However, we can see that there is much overlap in the locations of the customers by
delivery day of the week. We might encourage orders to be placed so that deliveries
form tight clusters by working with the sales department and the customers. Perhaps
some incentives could be provided to help discipline the order patterns. The orders
should form a general pattern as shown below. Currently, the volume for Thursday
exceeds the available truck capacity of 45 caskets. Maybe the farthest stops could be
handled by a for-hire service rather than acquiring another truck for such little usage.

33

Monday

Tuesday

Friday
Depot

Wednesday

Thursday

It appears that the truck capacity is about right, given that some slack capacity is
likely to be needed.
Once the pattern orders are established, either as currently given or as may be revised,
apply principles numbers 1, 3, 4, 5, and 7.

34

FOWLER DISTRIBUTING COMPANY


Teaching Note

Fowler Distributing Company involves a problem of routing and scheduling a fleet of


trucks to serve customers for beer and wine products on a daily basis. Determining an
efficient design for making daily deliveries when stop sequence, number of trucks and
their sizes, warehouse location, and time window restrictions are variables is the
objective of this exercise. For the most part, students should consider that they are to
prepare a consulting report to management about this problem. The several questions
provided at the end of the case study should help direct the analysis.
The ROUTER model is used to cost out and to optimize the various design scenarios.
Run results are tabulated in Table 1.
Q1. The current design may not be as efficient as it might be. Therefore, our first
task is establish a profile of the current design and then to plan the routes so as to
minimize the total miles driven and the number of trucks needed to serve the customers,
subject to the restrictions of truck capacity, time windows, total time on the route, etc.
Running the current route design in ROUTER establishes a benchmark as shown in
Figure 1. The design requires 334 miles of travel with 5 trucks. The daily routing cost is
$764.62.
Next, optimizing this problem, given no change in data or restrictions, gives the
revised benchmark design as shown in Figure 2. Now, costs are reduced to $731.31 per
day for a total routing distance of 311 miles. Six trucks are needed.
There appears to be a substantial change in the design of the routes that can achieve a
((334 311)/334)100 = 7% reduction in the total miles traveled to make the deliveries.
The saving in cost is ($764.62 731.31)250 = $8,327.50 per year. This is not a large
savings, but it can be achieved without changing the restriction on deliveries or incurring
additional investment.
Q2. Time window restrictions often force route design into stop sequences that are
not very economical, that is, routes cross themselves. This is the case for the revised
benchmark design as shown in Figure 2. To determine the impact of the time windows,
we can make an optimizing ROUTER run where there are no time window restrictions.
The daily time windows are all set 8:00AM to 5:00PM. The optimized routes are shown
in Figure 3. The route mileage is 270 miles per day for five trucks. The total cost is
$673.64 per day. From the revised benchmark, this is an additional cost reduction of
($731.31 673.64) 250 = $14,417.50 per year.

35

FIGURE 1 Current
Route Design

The questions for management are: How restrictive are the time windows? Can
deliveries be made outside of the account's "open hours", such as by giving the driver the
key to a safe storage area? Can management offer a small incentive to widen the time
window when it otherwise would not be convenient for the account? Relaxing such time
windows is often one of the important sources for cost savings in routing problems.
Q3. When there are no truck capacity restrictions on a routing problem, the most
efficient route design would be to use one large truck to serve all accounts. Therefore,
we would expect that trucks of larger capacity would reduce the total distance traveled.
A ROUTER optimizing run was made with 600 case capacity trucks to find out. All
other conditions were set at the current design.

36

FIGURE 2
Optimized
Current Route
Design

Compared with
the optimized current design, the potential savings is ($731.31 722.98) 250 =
$2,082.50 per year. Two 600-case trucks would be needed, along with 3 of the smaller
trucks. Although there is a positive savings associated with using larger trucks, the
savings seems quite small and perhaps not worth switching to some larger trucks at this
time. We should note that these savings are a result of comparing full costs, which
include truck depreciation. Although a present value analysis would be appropriate here,
we would need some additional information which might include (1) the proportion of
truck costs allocated to equipment depreciation, (2) the life of the trucks, (3) the
estimated salvage value of the trucks, and (4) and the required rate of return on
investments of this type.

37

FIGURE 3 Route Design with Relaxed


Time Window Constraints

Q4. From the incremental costs in the detailed report of the optimized current design,
we can see that account 14 costs $39.05 and account 19 costs $35.04 to make the
deliveries. Since the volume of account 19 is 90 cases and exceeds the 50 case capacity
of an outside transport service, account 19 cannot be considered for alternate delivery. If
it only costs $35.00 for an outside delivery service to handle account 14, then a cost
savings can be realized. Dropping this account and redesigning the routes shows that
only 5 trucks are needed for a route cost of $690.17 and route miles of 286. The total
cost for handling all accounts would be $690.17 + 35.00 = $725.17. Compared with the
optimized current design, this is an annual savings of ($731.31 725.17) 250 =
$1,535.00.
Economically, Roy should use the outside transportation. However, losing direct
control over the deliveries and possible adverse reactions from route salesmen may give
him second thoughts about it.
Q5. Two ROUTER runs were made here in order to determine the effect of a shorter
workday before overtime begins. If no overtime is allowed, then six trucks are required
for a total daily routing cost of $754.77. If some overtime is permitted, then the route
cost can be reduced to $733.28 with 6 trucks required. Compared with the optimized
current design, Roy Fowler would seem to be putting himself at a disadvantage by not
wanting to pay overtime.
Q6. Moving the warehouse to a more central location would have the appeal of being
closer to all stops and would result in shorter routes. Testing this shows that total route
distance can be reduced to 305 miles with a daily cost of $716.91. However, in order to
meet the time window and other constraints, an extra truck is needed. There is a potential
annual savings of ($731.31 716.91) 250 = $3,600.00. Since it costs $15,000 to make

38

the move, a simple return on investment would be (3,600/15,000) 100 = 24%. At this
ROI, the move should be seriously considered.
Q7. Routing and scheduling beer trucks from a central depot is quite similar to
delivery problems found in manufacturing, retail, and service industries. Several
examples are listed below.

Serving branch banks by making pickups and deliveries of canceled checks,


supplies, and other paper work from a central domicile point for the trucks
Making Federal Express pickups and deliveries around an airport
Dispatching school buses around a school for student pick up and drop off
Dispatching service personnel for various service companies such as electric,
water, gas, and telephone
Making retail deliveries from a warehouse
Making deliveries from a plant location to warehouses and then picking up
supplies from vendors on the return trip.
Making deliveries (and pickups) of medical records to doctor's offices and other
locations from a central record-keeping location such as a hospital

Summary
Roy Fowler should consider optimizing his route design. This can be done at no
investment to him, and he can gain about a 7% reduction in operating costs on the
average. A substantial benefit can be realized by widening the time windows. He should
especially see if those time windows that cause routes to overlap themselves can be
widened. The potential for doing this is up to $14,418 per year. He should explore the
possibility and reasonableness of serving accounts 14 and 19 by an outside transport
service. Finally, he should consider relaxing his rigid policy of not wanting to pay
overtime, especially if the union is successful in negotiating a 7-hour workday.
There seems to be little opportunity for reducing cost by increasing truck capacity or
relocating the warehouse. Although there are operating cost reductions available, they do
not seem sufficient to justify a change in truck size unless Roy can make good use of the
trucks in other ways.

39

TABLE 1 Results of Various ROUTER Runs


Run type
Miles
Cost
Trucks Comments
Current design
334
$764.62
5
Cost out current design
Improves on current
Optimized current
311
731.31
6
route design
No time windows
restrictions
270
673.64
5
Time windows opened
Trucks at 600-case
Two routes require larger
capacity
299
722.98
5
trucks
7 hr. work day - no
Route time allowed to be
overtime
332
754.77
6
no longer than 8.0 hours
7 hr. work day Route time allowed to
with overtime
311
733.28
6
exceed 8.0 hours
New warehouse
Warehouse moved to
location
305
716.91
7
more central location

40

METROHEALTH MEDICAL CENTER


Teaching Note
Strategy
MetroHealth Medical Centers mission is to provide transportation to and from its facility
for those patients that cannot afford the expense or are unable to use other modes of
transportation. MMC currently has a fleet of vans and drivers to serve this patient
population. In addition, some transportation can be provided as part of the hospitals
contract ambulance service. The primary service provided by Transportation Services is
to pick patients up and bring them to MMC for their scheduled appointments and then to
return them to their origin points. Careful management of the fleet is needed through
vehicle assignment to patients, routing the vans, and assigning some of the patients to the
ambulance service for transportation.
It is the purpose of this case to show how a basic routing and scheduling model can
be used to assist in answering the typical questions surrounding fleet utilization in a
service setting. The ROUTER module in LOGWARE is used as the analytical tool. This
case does require some insight on the part of the student to deal with the complexities of
this problem. Considering the amount of data provided and the richness of the issues
involved, this case is probably best assigned as a project rather than as a homework
exercise.
Answer to Questions
(1) Which vans from the current fleet should be used?
subcontracting be used?

To what extent should

This problem is complex, considering that it is quite dynamic. Although the patient pick
up list appears to be fixed for a particular day, changes take place in the form of
cancellations and occasional additions. Patient appointment times are met a high
percentage of the time, but not always. Patients to be returned to their origins may find
alternate means of transportation so that 100 percent of the patients picked up may not
have to be returned. For simplicity, it will be assumed that the patient pick up list is fixed
for a particular day and appointment times are rigid. In addition, the returns are not
directly considered in designing the routes. Rather, they are expected to be seeded into
the pick up routes. This need not be a serious limitation as long as van capacity (number
of pick-up patients to the number of available seats) is not highly utilized on a pickup
route and van utilization for pick ups drops as appointments are concentrated in the
morning hours and return times are shifted toward the afternoon hours, as shown in
Figure 1.

41

Appointment times

Return times

FIGURE 1 Appointment Time and Return Time Distributions

A typical days patient list can be submitted to ROUTER. Coordinate points for zip
code areas are scaled from the map in Figure 2 of the case study. Appointment times are
represented as time windows with a 2-hour gap for pick up. A large number of 15passenger and 6-passenger vans are assumed available to start the ROUTER model.
Since ROUTER is a basic vehicle routing and scheduling model, it simply assigns
patients to vans and gives a sequence to pick them up. It assumes that a single van leaves
and returns to MMC only once in a day. Analysis must account for this, since vans make
several out and back trips per day.
It is not possible to accurately recreate the current routing patterns for the vans from
the data given. However, optimized routing and scheduling can be found through the
application of ROUTER. First, the typical daily appointment list is solved with
ROUTER using a 2-hour time window where the ending window time is the appointment
time. An average speed of 27.5 miles per hour, variable costs of $0.11 per mile1, and a
large number of 15-passenger and 6-passenger vans are used. A complete database is
shown in the appendix of this note.
Using the appointment list for the typical day as representative with all 56 patients,
ROUTER shows that 6 vans are needed, where one is a 15-passenger type (route 1). This
is routing where no patient is transported by the contract transport service. The routing
represents a total of 329 miles driven at a variable cost of 329 mi. $0.11/mi. = $36.19.
The routes are placed end-to-end to cover the full day and then ranked, first by the
number of patients transported throughout the day on a van and second by the total miles
driven, as shown in Table 1. A reasonable procedure for allocating routes to vans is to
assign routes with the largest number of patients first to the largest vans. This is because
1

Per mile variable cost is determined as repair cost/30,000 mi. per yr. = $1,000/30,000 = $0.03 per mi. plus
fuel cost at $1.00 per gallon/average miles per gallon = 1/13 = $0.08 per mi. for a total of 0.08 + 0.03 =
$0.11 per mile.

42

the ambulance service charges on a per patient basis, and these routes would be most
costly to subcontract. If the number of patients is tied for a van, rank first the van with
the shortest total route distance.
TABLE 1 Route Design for Representative Patient Appointments with
No Transport Subcontracting
Van
no.
1
1
1
1
1
2
2
3
3
4
4
5
6
1

Start from
MMC
6:07AM
6:52AM
8:55AM
11:04AM
11:41AM
7:24AM
10:59AM
7:41AM
10:45AM
7:37AM
11:52AM
7:24AM
7:43AM

Return to
MMC
6:28AM
8:28AM
10:30AM
11:31AM
1:27PM
9:05AM
11:57AM
9:16AM
12:27PM
9:12AM
1:34PM
8:44AM
9:03AM

Patients on
route
1
81
4
1
5
5
5
5
5
5
5
4
3
56

Route distance
7 mi.
22
33
9
35
32
13
29
33
30
33
25
28
329 mi.

Route time
0.3 hr.
1.6
1.6
0.4
1.8
1.7
1.0
1.6
1.7
1.6
1.7
1.3
1.6
17.6 hr.

A 15-passenger van needed.

The cost of this initial route configuration can be summarized as follows:


Annual cost of 1 15-passenger van
Annual cost of 5 6-passenger vans
Annual repair cost of 6 vans
Annual cost for 6 drivers
Variable cost
Total cost

$ 5,750 1
28,500 2
6,000
141,000
8,686 3
$189,936

$23,000/4 yr. = $5,750


($19,000/4 yr.) 6 = $28,500
3
329 mi. x $0.11/mi. 240 days/yr. = $8,686
2

The question now is whether substituting contract carriage for some of the routes will
significantly lower vehicle and driver costs while only slightly increasing variable costs.
Since the routes are ranked in Table 1, a reasonable rule for dropping routes would be to
start from the bottom of the route list in Table 1 and work upward. This will determine
the number of vans needed and the patients to be transported by each mode. These
results are shown in Table 2. An unlimited number of subcontracting trips is assumed.

TABLE 2 Optimal Number of Vans

43

Vans
6
5
4
3
2
1
0
1

Driver
$141,000
117,500
94,000
70,500
47,000
23,500
0

Vehicle
$34,250
29,500
20,000
15,250
10,500
5,750
0

Variable
(gas +
repair)
$14,686
12,946
11,286
8,623
5,986
3,798
0

Subcontracting
$
0
6,235 1
14,549
35,333
56,117
76,901
116,390

Total
$189,936
166,181
139,835
129,706
119,603
109,949
116,390

Annual subcontracting cost is $8.66 per round trip 240 days per year 3 patients = $6,235.20

(2) How many subcontracted trips should MMC negotiate and at what price?
One 15-passenger van appears to be the optimal number, but this requires 37 patients
20 days per month = 740 subcontracting trips per month. This is more than the 500
allowed. If the 500-trip limit (500/20 days per mo. = 25 patients per day) is to be
respected, approximately one additional van will be needed. On the other hand, the cost
of the additional van is the annual cost of a van + the annual drivers salary + the gas and
repair cost = $19,000/4 + 23,500 + 1,1882 = $29,438/yr. to transport 10 patients per day.
At this rate, MMC could afford to pay $29,438/240/10 = $12.27 per round trip (find 10
patients for van 2 in Table 1). Possibly the subcontractor would be willing to offer this
additional transport for a price between the $8.66 and $12.27 per trip. It would be in
Macs interest to do this. Renegotiating all trips at the 750-trip level is another
possibility. MMC can afford to pay a subcontracting cost of up to $189,936 - 31,0483 =
$158,888/yr. This would require transporting 37 patients per day at a maximal average
trip cost of $158,888/240/37 = $17.89. Since $17.89 is the optimal worth of
subcontracting, MMC probably can negotiate a rate much less than this while still
encouraging the subcontractor to provide service at the higher level.
(3) MMC is considering using all 6-passenger vans. Would this be a good decision?
Developing daily routes and assigning them to 6-passenger vans is the same as for Table
1. This initial allocation without subcontracting is shown in Table 3. Considering the
subcontracting trip limit of 500 patients per month (25 patients per day), two vans are
needed in the MMC fleet. This number of vans is determined by counting the patients
from the bottom of Table 3 until approximately 25 patients are found, considering
increments of full vans and noting the number of vans remaining at the top of the list.
(Assume 27, an extra 2 patients can be accommodated by the subcontractor at no
additional cost.) The variable cost for two vans is shown below.

Variable cost type


2
3

Two
6-passenger

One 15- & one


6-passenger van

45 miles x $0.11/mile 240 days/yr. = $1,188.


Driver + vehicle +variable costs for one van, or 19,000/4 + 23,500 + 2,798 = $31,048.

44

vans
Annual cost of 2 drivers
Vehicle mileage + repair cost
Subcontracting cost
Total
1
2

47,000
5,9601
56,1172
$109,077

47,000
5,986
56,117
$109,103

150 mi. $0.11/mi. 240 days/yr. + 2,000 = $3,960.


27 patients $8.66/trip 240 days/yr. = $56,117.

In contrast, the 15-passenger and 6-passenger van configuration compared with the two
6-passenger van configuration saves $109,103 109,077 = $26 per year. The initial
outlay is less for the smaller van by $4,000. From purely an economic standpoint, using
just 6-passenger vans would appear to be the best choice, however there may be customer
service reasons for having the larger van available to meet peak demand needs.
TABLE 3 Route Design for 6-Passenger Vans Only
Van
no.
1
1
1
1
1
2
2
2
3
3
4
4
5
6

Start from
MMC
6:07AM
6:52AM
7:37AM
11:04AM
11:52AM
7:22AM
8:55AM
10:59AM
7:41AM
10:45AM
7:24AM
11:41AM
7:24AM
7:43AM

Return to
MMC
6:28AM
7:37AM
9:12AM
11:31AM
1:34PM
8:19AM
10:30AM
11:57AM
9:16AM
12:27PM
9:05AM
1:27PM
8:44AM
9:03AM

Patients on
route
1
3
5
1
5
5
4
5
5
5
5
5
4
3
56

Route distance
7 mi.
12
30
9
33
13
33
13
29
33
32
35
25
28
332 mi.

Route time
0.3 hr.
0.8
1.6
0.4
1.7
1.0
1.6
1.0
1.6
1.7
1.7
1.8
1.3
1.3
17.8 hr.

45

APPENDIX A Database for ROUTER


--PARAMETERS AND LABELS-Problem label - METROHEALTH MEDICAL HOSPITAL
Grid corner with 0,0 coordinates (NW, SW, SE, or NE) - SW
DEPOT DATA
Depot description - Medical Center
Located in zone - 0
Horizontal coordinate 7.40
Vertical coordinate 6.13
Earliest starting time (min) - 360
Latest return time (min) - 960
Default vehicle speed (miles per hour) 28
After how many clock hours will overtime begin 68.0
GENERAL DATA
Percent of vehicle in use before allowing pickups - 100
Horizontal scaling factor 3.102
Vertical scaling factor 2.820
Maximum TIME allowed on a route (hours) 2.0
Maximum DISTANCE allowed on a route (miles) - 9999.0
LOAD/UNLOAD TIME FORMULA
Fixed time per stop 6.00
Variable time per stop by weight 0.00 By cube 0.00
BREAK TIMES
Duration of 1st break (minutes) 0 To begin after - 9999
Duration of 2nd break (minutes) 0 To begin after - 9999
Duration of 3rd break (minutes) 0 To begin after - 9999
Duration of 4th break (minutes) 0 To begin after - 9999
--STOP DATA-NO.
1
2
3
4
5
6
7
8
9
10
11
12
13
14
15
16
17
18
19
20
21
22
23
24
25
26
27
28
29
30
31
32
33
34
35
36
37
38
39
40
41
42
43
44

STOP
DESCRIPTION---- TY
Baker
P
Boyd
P
Carver
P
Ivey
P
Rashed
P
Walsh
P
Johnson
P
Burgess
P
Delgado
P
Fairrow
P
Middlebrooks
P
Suech
P
Lawson
P
Reed
P
Bongiovanni
P
Miller
P
Talley
P
Williams
P
Dumas
P
Taylor
P
Barker
P
Lhota
P
Manco
P
Webb
P
Wilson
P
Arrington
P
Staunton
P
Wall
P
Williams
P
Caruso
P
West
P
Amaro
P
Brown
P
Ciesicki
P
Pinkevich
P
Staufer
P
Winterich
P
Brown
P
Ball
P
Lanza
P
Mayernik
P
Suech
P
Heffner
P
Jarrell
P

LOAD VOLUME
WGHT
CUBE
1
0
1
0
1
0
1
0
1
0
1
0
1
0
1
0
1
0
1
0
1
0
1
0
1
0
1
0
1
0
1
0
1
0
1
0
1
0
1
0
1
0
1
0
1
0
1
0
1
0
1
0
1
0
1
0
1
0
1
0
1
0
1
0
1
0
1
0
1
0
1
0
1
0
1
0
1
0
1
0
1
0
1
0
1
0
1
0

-COORDINATESLOAD ---TIME WINDOWS---HCRD


VCRD ZN TIME BEG1 END1 BEG2 END2
6.50
8.90 0
0 390 510 9999 9999
6.20
9.00 0
0 390 510 9999 9999
11.00
5.00 0
0 420 540 9999 9999
10.00
6.00 0
0 420 540 9999 9999
10.00
10.00 0
0 420 540 9999 9999
3.50
5.00 0
0 420 540 9999 9999
8.00
5.50 0
0 450 570 9999 9999
9.00
7.50 0
0 465 585 9999 9999
8.20
7.20 0
0 465 585 9999 9999
8.60
7.80 0
0 480 600 9999 9999
8.00
5.40 0
0 480 600 9999 9999
5.00
6.50 0
0 480 600 9999 9999
6.30
8.80 0
0 510 630 9999 9999
9.05
7.30 0
0 510 630 9999 9999
9.00
5.80 0
0 525 645 9999 9999
7.80
5.40 0
0 540 660 9999 9999
6.50
10.20 0
0 570 690 9999 9999
7.50
7.00 0
0 585 705 9999 9999
7.20
5.90 0
0 630 750 9999 9999
9.80
7.00 0
0 645 765 9999 9999
7.20
6.00 0
0 660 780 9999 9999
8.00
6.20 0
0 660 780 9999 9999
10.10
9.00 0
0 660 780 9999 9999
9.70
7.00 0
0 660 780 9999 9999
9.00
5.80 0
0 660 780 9999 9999
10.20
5.90 0
0 720 840 9999 9999
6.40
8.80 0
0 720 840 9999 9999
9.50
5.10 0
0 720 840 9999 9999
8.20
7.20 0
0 720 840 9999 9999
7.00
5.00 0
0 375 495 9999 9999
5.60
6.00 0
0 390 510 9999 9999
6.00
6.60 0
0 420 540 9999 9999
7.20
4.20 0
0 420 540 9999 9999
6.50
3.00 0
0 420 540 9999 9999
6.60
5.10 0
0 420 540 9999 9999
5.80
6.00 0
0 420 540 9999 9999
7.30
4.80 0
0 420 540 9999 9999
4.00
5.00 0
0 435 555 9999 9999
6.20
6.30 0
0 450 610 9999 9999
6.20
5.90 0
0 450 610 9999 9999
7.10
7.00 0
0 450 610 9999 9999
4.00
6.50 0
0 465 625 9999 9999
5.20
4.50 0
0 480 640 9999 9999
4.20
6.80 0
0 480 640 9999 9999

46

45
46
47
48
49
50
51
52
53
54
55
56

Piatak
Swaysland
Baer
Wills
Fauber
Mullins
Pack
Westerfield
Lisiewski
McPherson
Mykytuk
Gutschmidt

VEHICLE
NO. DESCRIPTION---1 15-pasngr vehcl
2 6-pasngr vehcl

P
P
P
P
P
P
P
P
P
P
P
P

TP
1
2

1
1
1
1
1
1
1
1
1
1
1
1

0
0
0
0
0
0
0
0
0
0
0
0

7.00
6.10
3.80
4.70
5.20
7.20
6.20
5.90
5.00
4.20
6.00
5.50

3.50
6.70
4.90
6.10
6.80
6.30
5.00
6.30
5.50
6.20
6.50
5.90

0
0
0
0
0
0
0
0
0
0
0
0

0
0
0
0
0
0
0
0
0
0
0
0

480
480
540
540
660
660
660
675
690
720
750
780

640
640
700
700
780
780
780
795
810
840
870
900

9999
9999
9999
9999
9999
9999
9999
9999
9999
9999
9999
9999

9999
9999
9999
9999
9999
9999
9999
9999
9999
9999
9999
9999

--VEHICLE DATA----VEHICLE--- ----DRIVER---OVER


--CAPACITY--FIXED PER MI
FIXED PER HR
TIME
NO WEIGHT --CUBE ---COST --COST ---COST --COST --COST
20
15
99
0.00
0.11
0.00
0.00
0.00
20
6
99
0.00
0.11
0.00
0.00
0.00

47

ORION FOODS, INC.


Teaching Note
Strategy
The purpose of this case study is to allow students to analyze a distribution problem
where determining the optimal paths through a network is central to the problem solution.
The shortest route methodology applies, and the ROUTE module in the LOGWARE
software can effectively be used. This module permits students to quickly find the
shortest distance paths from any starting point to all other points in the network. A
prepared data file of the network is available for use under the ROUTE module.
The case has several dimensions that allow it to be used as a homework assignment, a
short case study project, or as a basis for classroom discussion. It is a simple problem in
network design highlighting transportation routing issues, however additional factors
such as inventory consolidation and investment costs allow for an enriched analysis.
Note: A database for this problem is available in the ROUTE module of the
LOGWARE software.
Answers to Questions
(1) Can Anita improve upon the current distribution operations?

It should be obvious to Anita that the current distribution system may not be performing
at optimum. Allowing carriers to decide the routes to use when they are being paid on a
mileage basis is like asking a fox to watch the chicken coop. She really cannot expect
that carriers will be motivated to seek out optimal routing patterns. Therefore, she should
determine the best routes between regional and field warehouses and insist that carriers
invoice according to the mileages along these specified routes.
Using the map provided in Figure 1 of the case study and the current assignments of
field warehouses to regional warehouses, she can develop a database for the ROUTE
module in LOGWARE. The database is shown in the Supplement to this note. Running
ROUTE will give the optimal routes from which she can develop transportation costs, as
shown in Table 1 below. Compared with the current cost level, a savings of
$652,274 630,140 = $22,134 per year
can be realized. This savings does not require any investment; however, it will be
magnified by the growth in demand in the next five years.

48

TABLE 1 Optimal Transportation Costs Under Current


Distribution System Design
Regional
warehouse
Fresno
Fresno
Fresno
Fresno
Burns
Burns
Burns
Totals

Field
warehouse
Los Angeles
Phoenix
Salt Lake City
San Francisco
Portland
Butte
Seattle

Optimal
route
miles
219
588
815
183
293
676
467
3,241

Average
number of
tripsa
366.7
200.0
116.7
280.0
143.3
16.7
186.7

Transport
cost, $
104,399b
152,880
123,644
66,612
54,583
14,676
113,346
630,140

Determined from the warehouse throughput divided by the average shipment


size. e.g., 110,000 cwt./300 cwt. = 366.7 trips.
b
$1.30 per mi. 219 mi. 366.7 trips = $104,399.

(2) Is there any benefit to expanding the warehouse at Burns, OR?


Finding the optimal distances when serving all field warehouses from Burns or Fresno
gives a slightly different allocation of field warehouses to regional warehouses than is
currently the case. That is,

Field warehouse
Los Angeles
Phoenix
Salt Lake City
San Francisco
Portland
Butte
Seattle

If served
from Burns
806 mi.
973
536*
555
293*
676*
467*

If served
from Fresno
219 mi.*
588*
815
183*
757
1,120
925

Indicates optimal assignment.

This shows that Salt Lake City would be better served out of Burns rather than Fresno.
Burns currently is near its capacity limit, so to assign Salt Lake City's volume to it
would require expansion. In the short term, 35,000/8 = 4,375 cwt. of inventory capacity
is needed. However, (43,000 + 5,000 + 56,000)/8 = 13,000 cwt. of the 15,000 cwt. of
available capacity is currently being used. At minimum, an additional increment of
capacity is required at a cost of $300,000. Reassignment of Salt Lake City to Burns
would save 815 536 = 279 miles per trip. On 116.7 trips per year, the annual savings
would be 1.30 279 116.7 = $42,327. The simple return on investment (ROI) would
be:
ROI

$42,327
100 141%
.
$300,000

49

If the anticipated growth in demand is realized, the number trips to Salt Lake City would
increase to 56,000/300 = 186.7. The projected savings in the fifth year would be 1.30
279 186.7 = $67,716. The average annual savings would be (42,327 + 67,716)/2 =
$55,022. The average annual ROI is:
ROI

$55,022
100 18.3%
$300,000

Anita must now compare this return to other worthy investments in the firm to see if this
opportunity is worth the risk.
(3) Is there any merit to consolidating the regional warehousing operation at Reno, NV?
If Reno were to replace the Burns and Fresno warehouses, an initial cost of $2,000,000
would be incurred to establish the new location and shut down the existing warehouses.
Against this cost would be a savings of 40 percent of the inventory in the two regional
warehouses. That is, the total inventory is 393,000/8 = 49,125 cwt. now and 528,000/8 =
66,000 cwt. in five years. The inventory cost savings now would be 0.40 0.35 60
49,125 = $412,650 and in five years 0.40 0.35 60 66,000 = $554,400. However,
transportation costs will increase compared with the two-warehouse distribution system.
If Reno is used, the transportation costs would be:
Optimal
Field warehouse
route
served from Reno
miles
Los Angeles
472
Phoenix
732
Salt Lake City
520
San Francisco
228
Portland
542
Butte
823
Seattle
716
Totals
4,033
a
472 336.7 1.30 = $225,007

Current
no. of
trips
366.7
200.0
116.7
280.0
143.3
16.7
186.7

Current
transport
cost, $
225,007a
190,320
78,889
82,992
100,969
17,867
173,780
869,824

5th-year
no. of
trips
440.0
280.0
186.7
350.0
190.0
50.0
263.3

5th-year
transport
costs, $
269,984
266,448
126,209
103,740
133,874
53,495
245,080
1,198,830

We cannot make a fair comparison with the current system design since there is
inadequate capacity at Burns to handle the growth in volume. Two additional units of
capacity will be needed for a total of $600,000. Then, the net investment attributable to
Reno is 2,000,000 600,000 = $1,400,000. The revised transportation cost for the fifth
year is as follows.

50

Regional
warehouse
Fresno
Fresno
Burns
Fresno
Burns
Burns
Burns
Totals

Field
warehouse
Los Angeles
Phoenix
Salt Lake City
San Francisco
Portland
Butte
Seattle

Optimal
route
miles
219
588
536
183
293
676
467
2,962

5th-year
number of
trips
440.0
280.0
186.7
350.0
190.0
50.0
263.3

Transport
cost, $
125,268
214,032
130,092
83,265
72,371
43,940
159,849
828,817

Now,

Transport costs
Transport costs
Net increase
Less inventory
savings
Net cost savings

Current year
$587,813
869,824
$282,011

5th year
$828,817
1,198,830
$370,013

(412,650)
$130,639

(554,400)
$184,387

Burns/Fresno
Reno

The average annual savings for Reno now is (130,639 + 184,387)/2 = $157,513. The
relevant return on investment is
ROI

$157,513
100 1125%
.
$1,400,000

This is probably not a sufficient return to justify the warehouse at Reno. Rather, if Orion
wishes to serve the increasing demandthere is no requirement to do sothen the better
strategy would be to expand Burns by 20,000 cwt. and serve Salt Lake City from this
location.
An interesting question to pose to students is: What does it mean to only serve
demand up to the limits of capacity? Orion could serve only the more profitable demand
and avoid the risks of expansion.

51

R & T WHOLESALERS
Teaching Note4

The objective of this assignment is to minimize the total monthly delivery costs for R&T
Wholesalers, a company distributing general products throughout India. The focus is on
one warehouse acting as a truck depot that delivers merchandise to retailers located in
surrounding towns. Delivery expenses are minimized through the optimal utilization of
the trucks, crews, and related expenses. The constraints and other considerations listed
below were used when designing the solution methodology and identifying the optimal
solution.
Constraints
Operating Schedule
- Trucks make deliveries every day of the week except Saturday or Sunday
- Normal operation is for trucks to be loaded overnight and leave from the
warehouse in the morning
- Trucks make deliveries within towns from 9 a.m. to 6 p.m.
- Earliest start time for trucks is 12 a.m. in the morning of delivery
- Trucks returned to the depot require 2 hours for reloading and subsequent sameday delivery
Visits Per Month
- Every town has a known number of visits per month
Truck Capacity
- T407 trucks have a capacity of Rs500,000
- T310 trucks have a capacity of Rs350,000
Truck Operating Costs
- Trucks operate at an average speed of 40 km/hr
- T407 trucks have an operating cost of Rs13,500
- T310 trucks have an operating cost of Rs7,000
- T407 trucks have a running cost of Rs5 per kilometer
- T310 trucks have a running cost of Rs3 per kilometer
- Each truck has a crew of two, a driver and a helper
- The driver is paid Rs2,200 per month
- The helper is paid Rs1,400 per month
- Each crewmember receives Rs60 per day for meals and other expenses while on
the road
Working Schedule
- Flexibly planned breaks for crewmembers are at approximately 6 a.m., 12 p.m.
and 6 p.m.
- Breakfast and lunchtime breaks are 30 minutes each and dinner is 60 minutes

The solution to this exercise is provided by Nutthapol Dussadeenoad, Inderjot Gandhi, Earle Keith, Lisa
Kuta, Jan Shahan, and
Piyanuch Vichitakul who were students in the MBA program of the Weatherhead School of Management.

52

- Crewmembers are allowed at least an 8-hour overnight break before starting a


route on the following day
- No overtime is paid and the company policy is to return the crews to the
warehouse each day rather than plan for overnight layovers
Other Considerations
Assumptions
- Delivery demand is fixed by quantity and location, and city demand cannot be
subdivided
Opportunities
- Subcontracting is available at an estimated rate of Rs15 per km one way to the
town.
Methodology
The methodological options leading to a good solution are outlined in Figure 1.
START

1. Route 43 cities
altogether

Cost = Rs116,678

2. Route all 4-visit cities and


outsource all 2-visit cities

Cost = Rs135,254

3. Route 2 sets of data: all 4-visit cities in


one seat and 2-visit cities in another set

4. Route 2 sets of data: mix between


4-visit cities and 2-visit cities

Cost = Rs109,092

Cost = Rs78,328

5. Outsource all cities whose demand is > Rs350,000 and route 2


sets of data: mix between 4-visit cities and 2-visit cities

Cost = Rs65,780

Satisfy?
Y
STOP

FIGURE 1 Computational Options to Find a Good Solution

53

Detail on each methodological step is offered as follows.


1. Route all 43 cities altogether
Weeks 1&3: visit all cities = 14 routes/week
Weeks 2&4: visit only 4-visit cities = 7 routes/week
Results:
- No. of truck used: T401 = 2, T301 = 2
- Cost = Rs116,678
2. Route all 4-visit cities, outsource all 2-visit cities
All week = 7 routes/week
Weeks 1 & 3: visit half of 2-visit cities.
Weeks 2 & 4: visit another half of 2-visit cities.
Results:
- No. of truck used: T401 = 1, T301 = 1
- Cost = Rs135,254
3. Route 2 set of data, separate between 4-visit cities and 2-visit cities
4-visit cities: all weeks = 9 routes/week
2-visit cities:
- Week 1&3: visit half of 2-visit cities = 4 routes/ week
- Week 2&4 :visit another half of 2-visit cities = 3 routes/week
Result
- No of truck used: T401=1, T301 =2
- Cost = Rs109,092
4. Route 2 sets of data, mix between 4-visit cities and 2-visit cities
4-visit cities + half of 2-visit cities: visit in weeks 1&3 = 10 routes/week
4-visit cities + another half of 2-visit cities: visit in week 2&4 = 9 routes/week
Note: Arrange 2-visit cities into 2 sets by following the below steps
- Separate those 2-visit cities into 2 sets by area, east/west.
- Sort cities in each set by sales per visit.
- Set all cities in odd row as set 1, and all cities in even row as set 2
- The logic behind this method is that each set should be balanced in terms of
demand and area.
Result
- No of truck used: T401 = 1, T301 = 1
- Cost = Rs78,328
5. Route 2 sets of data, mix 4-visit cities and 2-visit cities, and outsource all cities
whose demand greater than Rs350,000
4-visit cities + half of 2-visit cities: visit in weeks 1&3 = 10 routes/week
4-visit cities + another half of 2-visit cities: visit in weeks 2&4 = 10 routes/week
Note:

54

1) Arrange 2-visit cities into 2 sets by following the below steps


- Separate those 2-visit cities into 2 sets by sales per visits, low demand and
high demand
- Sort cities in each set by distance from the central warehouse
- Set all cities in odd row as set 1, and all cities in even row as set 2
- The logic behind this method is each set should be balanced in terms of
demand and distance from the central warehouse. Using distance should
reflect the better route than roughly separating cities by east/west area like in
method 4.
2) Outsource cities whose demand is greater than 350,000 so that only a T310
truck is required
- Guntur and Rajahmundry are the only 2 cities whose demand is greater than
350,000. Both of them are weekly visit cities, therefore, outsource to these
two cities every week.
Result
- No. of truck used: T301 = 2
- Outsource 2 cities for all weeks: Guntur and Rajahmundry
- Cost = Rs65,780, which is the best solution
Best Solution
The best solution per method 5 completes all deliveries for a total monthly cost of
Rs65,780 using only 2 of T301 trucks together with 2 sets of crew members. The cost
calculation is shown in Figure 2.
The daily schedule shown in Figure 3 represents the number of trucks needed, the
truck routes with stop sequence, the schedule of truck usage through the month, and the
schedule for using the crews. The Gantt chart of truck deliveries throughout the month is
shown in Figure 4.
See the router solution reports generated from LOGWARE, ROUTER module, for
individual route detail. The reports divide the month into weeks 1 & 3 (Figure 5) and
weeks 2 & 4 (Figure 6).

55

FIGURE 2 Cost Calculation Sheet for Best Solution


Total monthly cost
2
2
2

fixed cost

allowance

Week 1&3
Week 2&4

Running cost

Common Carrier

Total

of
of
of

truck 2 (T310)
driver
helper
4
2
2

people
people
people

Week 1 & 3
Week 2 & 4
To Guntur
To Rajahmundry

=
=
=
x
x
x
=
=
=
=

2 weeks
2 weeks
2 weeks

x
x
x

3,146 km
2,414 km
35
76

km
km

2
2
2
5 days
5 days
4 days

x
x
x

x
x
x

7,000
2,200
1,400
Rs 60
Rs 60
Rs 60

=
=
=

14,000
4,400
2,800

=
=
=

2,400
1,200
960

x
x

3 Rs/km
3 Rs/km

x
x

2 weeks
2 weeks

=
=

18,876
14,484

x
x

15 Rs/km
15 Rs/km

x
x

4 weeks
4 weeks

=
=

2,100
4,560

65,780

56

FIGURE 3 Daily Schedule

Daily Schedule Summary


WEEK 1 & 3

Truck #
DAY 1

II

DAY 2

II

DAY 3

II

DAY 4

II

Route
1 Depot
Podili
Kondulur
Tanguturu
Depot
25 Depot
Jangareddygudem
Kakinada
Depot

Start Time

Crew #
Driver I, Helper I

7:04 PM
5.46 AM

405
339

Driver II, Helper II

5:45 PM
6:39 AM

7 Depot
Tadikonda
Tenali
Chirala
Vuyyuru
Depot

7:59 AM

8 Depot
Sattenapalle
Ongole
Depot

Route
Miles

4:27 AM

5 Depot
Chilakalurupet
Narasaraopet
Macheria
Depot

13 Depot
Kaikalur
Bhimavaram
Tadepallegudem
Depot

End Time

Driver I, Helper I

8:18 PM

306
Driver II, Helper II

8:47 PM

232

6:47 AM

Driver I, Helper I

7:07 PM

253

7:01 AM

Driver II, Helper II

8:12 PM

19 Depot
Narasapur
Mandapeta
Depot

5:19 AM

11 Depot
Pamarru
Machilipatnam
Palakolu
Depot

7:19 AM

307
Driver I, Helper I

7:03 PM

349
Driver II, Helper II

5:19 PM

240

57

DAY 5

24 Depot
Eluru
Chintalapuidi
Depot

II

2 Depot
Kani Giri
Bestavaipetta
Giddalur
Markapur
Depot

6:56 AM

Driver I, Helper I

3:11 PM

190

3:44 AM

Driver II, Helper II

11:21 PM

525

End Time

Route
Miles

Guntur
Rajahmundry

Outsource

WEEK 2 & 4

DAY 1

Truck #

Route

Start Time

3 Depot
Ongole
Depot

5:01 AM

II

DAY 2

II

Driver I, Helper I
3:28 PM

18 Depot
Jaggayyapeta
Eluru
Depot

6:34 AM

13 Depot
Vuyyuru
Machilipatnam
Gudivada
Depot

7:41 AM

19 Depot
Hunuman Junction
Chirala
Bapatia
Repalie
Depot

8:26 AM

Crew#

278
Driver II, Helper II

5:00 PM

276
Driver I, Helper I

3:30 PM

153
Driver II, Helper II

7:49 PM

175

58

DAY 3

II

DAY 4

II

DAY 5

II
Outsource

14 Depot
Tenali
Palakolu
Depot

7:35 AM

24 Depot
Tanuku
Nidadvole
Kovvur
Depot

5:12 AM

16 Depot
Nuzvid
Tadepallegudem
Bhimavaram
Depot

7:27 AM

29 Depot
Piduguralia
Addanki
Vinukonda
Narasaraopet
Depot

6:23 AM

23 Depot
Amaiapuram
Kakinada
Depot

4:23 AM

Driver I, Helper I

4:40 PM

243
Driver II, Helper II

5:04 PM

315
Driver I, Helper I

5:51 PM

236

Driver II, Helper II

7:49 PM

318
Driver I, Helper I

7:52 PM

420

Open
Guntur
Rajahmundry

Note: Truck # represents the number of type T301 truck.

59

FIGURE 4 Gantt Chart of Truck Deliveries Throughout the Month

60

FIGURE 5 ROUTER Solution Report, Weeks 1 and 3


Label- Untitled
Date- 11/23/2002
Time- 2:47:17 AM
*** SUMMARY REPORT ***
TIME/DISTANCE/COST INFORMATION

Route
no
1
2

Route
Run Stop
Brk Stem
time, time, time, time, time,
Start Return No of
Route
hr
hr
hr
hr
hr
time
time stops dist,Mi
14.6 10.1
2.5
2.0
7.8 04:27AM 07:04PM
3
405
19.6 13.1
4.5
2.0
9.4 03:44AM 11:21PM
4
525

Route
cost,$
1215.00
1575.00

13.6

7.6

4.0

2.0

5.4 06:39AM 08:18PM

306

918.00

7
8

12.8
13.2

5.8
7.7

5.0
3.5

2.0
2.0

1.4 07:59AM 08:47PM


5.0 07:01AM 08:12PM

4
2

232
307

696.00
921.00

11

10.0

6.0

3.0

1.0

3.3 07:19AM 05:19PM

240

720.00

13

12.3

6.3

4.0

2.0

4.5 06:47AM 07:07PM

253

759.00

19

13.7

8.7

3.0

2.0

7.2 05:19AM 07:03PM

349

1047.00

24
25

8.2
12.0

4.8
8.5

2.5
2.5

1.0
1.0

3.6 06:56AM 03:11PM


7.7 05:46AM 05:45PM

2
2

190
339

570.00
1017.00

VEHICLE INFORMATION
Route Veh Weight Delvry Pickup Weight
Cube Delvry Pickup
no typ capcty weight weight
util capcty
cube
cube
1
1
350
180
0 51.4%
9999
0
0
2
1
350
134
0 38.3%
9999
0
0

Cube
util
.0%
.0%

Vehicle
description
T310
T310

350

342

97.7%

9999

.0% T310

7
8

1
1

350
350

337
350

0 96.3%
0 100.0%

9999
9999

0
0

0
0

.0% T310
.0% T310

11

350

350

0 100.0%

9999

.0% T310

13

350

326

93.1%

9999

.0% T310

19

350

330

94.3%

9999

.0% T310

24
25

1
1

350
350

266
296

0
0

76.0%
84.6%

9999
9999

0
0

0
0

.0% T310
.0% T310

*** DETAIL REPORT ON ROUTE NUMBER 1 ***


A T310 leaves at 4:27AM on day 1 from the depot at Vijayawada

Stop
No description

Stop
Drive Distance Time
Arrive
Depart
time to stop to stop wind
time Day
time Day Min
Min
Miles met?
-->Break 30 minutes

61

2 Podili
6 Kondulur

09:00AM
1 09:30AM
11:28AM
1 12:28PM
-->Break 30 minutes
01:19PM
1 02:19PM
-->Break 60 minutes
07:04PM
1 -------

1 Tanguturu
Depot
Stop
No description

Stop volume
Weight
Cube

2 Podili
6 Kondulur
1 Tanguturu
Totals Weight: Del = 180

24
90
66
Pickups

Route time:
Driving
Load/unload
Break
Total

10.1 hr
2.5
2.0
14.6 hr

Max allowed

21.0 hr

Route costs:
Driver (reg time)
Driver (over time)
Vehicle (mileage)
Fixed
Total

0
0
0
=

1
1

30
60

243.0
118.5

162
79

YES
YES

60

21.0

14

YES

--

---

225.0

150

Inc cost to serve stop


In $ In $/unit

Capacity in use
Weight
Cube
51.4%
.0%
183.00
7.6
44.6
.0
51.00
.6
18.9
.0
-48.00
-.7
.0
.0
0 Cube: Del = 0 Pickups = 0

Distance:
To 1st stop
From last stop
On route
Total
Max allowed

162 mi
150
93
405 mi
9999 mi

$.00
.00
1215.00
.00
$1215.00

*** DETAIL REPORT ON ROUTE NUMBER 2 ***


A T310 leaves at 3:44AM on day 1 from the depot at Vijayawada

Stop
No description
5 Kani Giri
9 Bestavaipetta
7 Giddalur
4 Markapur
Depot
Stop
No description

Arrive
time
-->Break
09:00AM
-->Break
01:21PM
02:41PM
05:12PM
-->Break
11:21PM

Stop
Drive Distance Time
Depart
time to stop to stop wind
Day
time Day Min
Min
Miles met?
30 minutes
1 11:30AM
1 150
286.5
191 YES
30 minutes
1 01:51PM
1
30
81.0
54 YES
1 03:41PM
1
60
49.5
33 YES
1 05:42PM
1
30
91.5
61 YES
60 minutes
1 ------- -- --279.0
186

Stop volume
Weight
Cube

5 Kani Giri
9 Bestavaipetta
7 Giddalur
4 Markapur
Totals Weight: Del = 134

24
25
25
60
Pickups

Route time:
Driving
Load/unload
Break
Total

13.1 hr
4.5
2.0
19.6 hr

Max allowed

24.0 hr

0
0
0
0
=

Inc cost to serve stop


In $ In $/unit

Capacity in use
Weight
Cube
38.3%
.0%
87.00
3.6
31.4
.0
15.00
.6
24.3
.0
210.00
8.4
17.1
.0
-21.00
-.4
.0
.0
0 Cube: Del = 0 Pickups = 0

Distance:
To 1st stop
From last stop
On route
Total
Max allowed

191 mi
186
148
525 mi
9999 mi

62

Route costs:
Driver (reg time)
Driver (over time)
Vehicle (mileage)
Fixed
Total

$.00
.00
1575.00
.00
$1575.00

*** DETAIL REPORT ON ROUTE NUMBER 5 ***


A T310 leaves at 6:39AM on day 1 from the depot at Vijayawada

Stop
No description

Arrive
time
-->Break
11 Chilakalurupet 09:00AM
12 Narasarapet
10:31AM
-->Break
42 Macheria
01:46PM
-->Break
Depot
08:18PM
Stop
No description

Stop
Drive Distance Time
Depart
time to stop to stop wind
Day
time Day Min
Min
Miles met?
30 minutes
1 10:00AM
1
60
111.0
74 YES
1 11:31AM
1
60
31.5
21 YES
30 minutes
1 03:46PM
1 120
105.0
70 YES
60 minutes
1 ------- -- --211.5
141

Stop volume
Weight
Cube

Inc cost to serve stop


In $ In $/unit

Capacity in use
Weight
Cube
97.7%
.0%
57.00
.6
71.4
.0
.00
.0
42.9
.0
405.00
2.7
.0
.0
0 Cube: Del = 0 Pickups = 0

11 Chilakalurupet
92
12 Narasarapet
100
42 Macheria
150
Totals Weight: Del = 342 Pickups

0
0
0
=

Route time:
Driving
Load/unload
Break
Total

7.6 hr
4.0
2.0
13.6 hr

Distance:
To 1st stop
From last stop
On route
Total

Max allowed

21.0 hr

Route costs:
Driver (reg time)
Driver (over time)
Vehicle (mileage)
Fixed
Total

Max allowed

74 mi
141
91
306 mi
9999 mi

$.00
.00
918.00
.00
$918.00

*** DETAIL REPORT ON ROUTE NUMBER 7 ***


A T310 leaves at 7:59AM on day 1 from the depot at Vijayawada

Stop
No description
14 Tadikonda
19 Tenali
8 Chirala
18 Vuyyuru
Depot
Stop
No description

Arrive
time
-->Break
09:00AM
10:58AM
-->Break
02:00PM
05:57PM
-->Break
08:47PM

Stop
Drive Distance Time
Depart
time to stop to stop wind
Day
time Day Min
Min
Miles met?
30 minutes
1 10:00AM
1
60
31.5
21 YES
1 11:58AM
1
60
58.5
39 YES
30 minutes
1 04:00PM
1 120
91.5
61 YES
1 06:57PM
1
60
117.0
78 YES
60 minutes
1 ------- -- --49.5
33

Stop volume
Weight
Cube

Inc cost to serve stop


In $ In $/unit

Capacity in use
Weight
Cube

63

14 Tadikonda
60
19 Tenali
140
8 Chirala
98
18 Vuyyuru
39
Totals Weight: Del = 337 Pickups

96.3%
0
72.00
1.2
79.1
0
33.00
.2
39.1
0
219.00
2.2
11.1
0
66.00
1.7
.0
= 0 Cube: Del = 0 Pickups = 0

Route time:
Driving
Load/unload
Break
Total

5.8 hr
5.0
2.0
12.8 hr

Distance:
To 1st stop
From last stop
On route
Total

Max allowed

21.0 hr

Route costs:
Driver (reg time)
Driver (over time)
Vehicle (mileage)
Fixed
Total

Max allowed

.0%
.0
.0
.0
.0

21 mi
33
178
232 mi
9999 mi

$.00
.00
696.00
.00
$696.00

*** DETAIL REPORT ON ROUTE NUMBER 8 ***


A T310 leaves at 7:01AM on day 1 from the depot at Vijayawada

Stop
No description
15 Sattenapalle
3 Ongole
Depot

Arrive
time
-->Break
09:00AM
-->Break
01:13PM
-->Break
08:12PM

Stop
No description

Stop
Drive Distance Time
Depart
time to stop to stop wind
Day
time Day Min
Min
Miles met?
30 minutes
1 10:00AM
1
60
88.5
59 YES
30 minutes
1 03:43PM
1 150
163.5
109 YES
60 minutes
1 ------- -- --208.5
139

Stop volume
Weight
Cube

Capacity in use
Weight
Cube
100.0%
.0%
15 Sattenapalle
45
0
87.00
1.9
87.1
.0
3 Ongole
305
0
567.00
1.9
.0
.0
Totals Weight: Del = 350 Pickups = 0 Cube: Del = 0 Pickups = 0
Route time:
Driving
Load/unload
Break
Total

7.7 hr
3.5
2.0
13.2 hr

Max allowed

21.0 hr

Route costs:
Driver (reg time)
Driver (over time)
Vehicle (mileage)
Fixed
Total

Inc cost to serve stop


In $ In $/unit

Distance:
To 1st stop
From last stop
On route
Total
Max allowed

59 mi
139
109
307 mi
9999 mi

$.00
.00
921.00
.00
$921.00

*** DETAIL REPORT ON ROUTE NUMBER 11 ***


A T310 leaves at 7:19AM on day 1 from the depot at Vijayawada
Stop

Drive Distance Time

64

Stop
No description
20 Pamarru
22 Machilipatnam
38 Palakolu
Depot

Arrive
Depart
time to stop
time Day
time Day Min
Min
-->Break 30 minutes
09:00AM
1 10:00AM
1
60
70.5
10:36AM
1 11:36AM
1
60
36.0
-->Break 30 minutes
02:12PM
1 03:12PM
1
60
126.0
05:19PM
1 ------- -- --127.5

Stop
No description

Stop volume
Weight
Cube

47
24

YES
YES

84
85

YES

Inc cost to serve stop


In $ In $/unit

Capacity in use
Weight
Cube
100.0%
.0%
-9.00
-.1
82.3
.0
126.00
1.2
51.4
.0
285.00
1.6
.0
.0
0 Cube: Del = 0 Pickups = 0

20 Pamarru
62
22 Machilipatnam
108
38 Palakolu
180
Totals Weight: Del = 350 Pickups

0
0
0
=

Route time:
Driving
Load/unload
Break
Total

6.0 hr
3.0
1.0
10.0 hr

Distance:
To 1st stop
From last stop
On route
Total

Max allowed

21.0 hr

Route costs:
Driver (reg time)
Driver (over time)
Vehicle (mileage)
Fixed
Total

to stop wind
Miles met?

Max allowed

47 mi
85
108
240 mi
9999 mi

$.00
.00
720.00
.00
$720.00

*** DETAIL REPORT ON ROUTE NUMBER 13 ***


A T310 leaves at 6:47AM on day 1 from the depot at Vijayawada

Stop
No description

Arrive
time
-->Break
23 Kaikalur
09:00AM
39 Bhimavaram
10:54AM
-->Break
36 Tadepallegudem 01:48PM
-->Break
Depot
07:07PM
Stop
No description

Stop
Drive Distance Time
Depart
time to stop to stop wind
Day
time Day Min
Min
Miles met?
30 minutes
1 10:00AM
1
60
102.0
68 YES
1 12:24PM
1
90
54.0
36 YES
30 minutes
1 03:18PM
1
90
54.0
36 YES
60 minutes
1 ------- -- --169.5
113

Stop volume
Weight
Cube

Inc cost to serve stop


In $ In $/unit

Capacity in use
Weight
Cube
93.1%
.0%
-12.00
-.2
79.4
.0
-39.00
-.3
37.1
.0
123.00
.9
.0
.0
0 Cube: Del = 0 Pickups = 0

23 Kaikalur
48
39 Bhimavaram
148
36 Tadepallegudem
130
Totals Weight: Del = 326 Pickups

0
0
0
=

Route time:
Driving
Load/unload
Break
Total

6.3 hr
4.0
2.0
12.3 hr

Distance:
To 1st stop
From last stop
On route
Total

Max allowed

21.0 hr

Max allowed

68 mi
113
72
253 mi
9999 mi

65

Route costs:
Driver (reg time)
Driver (over time)
Vehicle (mileage)
Fixed
Total

$.00
.00
759.00
.00
$759.00

*** DETAIL REPORT ON ROUTE NUMBER 19 ***


A T310 leaves at 5:19AM on day 1 from the depot at Vijayawada

Stop
No description
30 Narasapur
29 Mandapeta

Depot

Arrive
time
-->Break
09:00AM
11:30AM
-->Break
-->Break
07:03PM

Stop
No description

Stop
Drive Distance Time
Depart
time to stop to stop wind
Day
time Day Min
Min
Miles met?
30 minutes
1 10:00AM
1
60
190.5
127 YES
1 01:30PM
1 120
90.0
60 YES
30 minutes
60 minutes
1 ------- -- --243.0
162

Stop volume
Weight
Cube

Capacity in use
Weight
Cube
94.3%
.0%
30 Narasapur
160
0
75.00
.5
48.6
.0
29 Mandapeta
170
0
285.00
1.7
.0
.0
Totals Weight: Del = 330 Pickups = 0 Cube: Del = 0 Pickups = 0
Route time:
Driving
Load/unload
Break
Total

8.7 hr
3.0
2.0
13.7 hr

Max allowed

21.0 hr

Route costs:
Driver (reg time)
Driver (over time)
Vehicle (mileage)
Fixed
Total

Inc cost to serve stop


In $ In $/unit

Distance:
To 1st stop
From last stop
On route
Total
Max allowed

127 mi
162
60
349 mi
9999 mi

$.00
.00
1047.00
.00
$1047.00

*** DETAIL REPORT ON ROUTE NUMBER 24 ***


A T310 leaves at 6:56AM on day 1 from the depot at Vijayawada

Stop
No description
37 Eluru
41 Chintalapuidi
Depot
Stop
No description

Stop
Drive Distance Time
Arrive
Depart
time to stop to stop wind
time Day
time Day Min
Min
Miles met?
-->Break 30 minutes
09:00AM
1 11:00AM
1 120
94.5
63 YES
-->Break 30 minutes
12:41PM
1 01:11PM
1
30
70.5
47 YES
03:11PM
1 ------- -- --120.0
80
Stop volume
Weight
Cube

Inc cost to serve stop


In $ In $/unit

Capacity in use
Weight
Cube
76.0%
.0%
37 Eluru
198
0
90.00
.5
19.4
.0
41 Chintalapuidi
68
0
192.00
2.8
.0
.0
Totals Weight: Del = 266 Pickups = 0 Cube: Del = 0 Pickups = 0

66

Route time:
Driving
Load/unload
Break
Total

4.8 hr
2.5
1.0
8.2 hr

Max allowed

21.0 hr

Route costs:
Driver (reg time)
Driver (over time)
Vehicle (mileage)
Fixed
Total

Distance:
To 1st stop
From last stop
On route
Total
Max allowed

63 mi
80
47
190 mi
9999 mi

$.00
.00
570.00
.00
$570.00

*** DETAIL REPORT ON ROUTE NUMBER 25 ***


A T310 leaves at 5:46AM on day 1 from the depot at Vijayawada
Stop
Drive Distance Time
Arrive
Depart
time to stop to stop wind
time Day
time Day Min
Min
Miles met?
-->Break 30 minutes
40 Jangareddygudem 09:00AM
1 09:30AM
1
30
163.5
109 YES
32 Kakinada
10:15AM
1 12:15PM
1 120
45.0
30 YES
-->Break 30 minutes
Depot
05:45PM
1 ------- -- --300.0
200
Stop
No description

Stop
No description

Stop volume
Weight
Cube

Capacity in use
Weight
Cube
84.6%
.0%
40 Jangareddygudem
68
0
-183.00
-2.7
65.1
.0
32 Kakinada
228
0
363.00
1.6
.0
.0
Totals Weight: Del = 296 Pickups = 0 Cube: Del = 0 Pickups = 0
Route time:
Driving
Load/unload
Break
Total

8.5 hr
2.5
1.0
12.0 hr

Max allowed

21.0 hr

Route costs:
Driver (reg time)
Driver (over time)
Vehicle (mileage)
Fixed
Total

Inc cost to serve stop


In $ In $/unit

Distance:
To 1st stop
From last stop
On route
Total
Max allowed

109 mi
200
30
339 mi
9999 mi

$.00
.00
1017.00
.00
$1017.00

67

FIGURE 6 ROUTER Solution Report, Weeks 2 and 4


Label- Untitled
Date- 11/23/2002
Time- 4:21:32 AM
*** SUMMARY REPORT ***
TIME/DISTANCE/COST INFORMATION

Route
no
3

Route
Run Stop
Brk Stem
time, time, time, time, time,
Start Return No of
Route
hr
hr
hr
hr
hr
time
time stops dist,Mi
10.4
7.0
2.5
1.0
7.0 05:01AM 03:28PM
1
278

Route
cost,$
834.00

13
14

7.8
9.1

3.8
6.1

3.0
2.0

1.0
1.0

2.0 07:41AM 03:30PM


3.0 07:35AM 04:40PM

3
2

153
243

459.00
729.00

16

10.4

5.9

3.5

1.0

3.8 07:27AM 05:51PM

236

708.00

18
19

4.8
11.4

3.8
4.4

.5
5.0

.5
2.0

3.8 06:34AM 5:00PM


1.7 08:26AM 07:49PM

2
4

276
175

828.00
525.00

23
24

15.5
11.9

10.5
7.9

3.0
3.0

2.0
1.0

9.1 04:23AM 07:52PM


7.0 05:12AM 05:04PM

2
3

420
315

1260.00
945.00

29

13.4

8.0

3.5

2.0

4.0 06:23AM 07:49PM

318

954.00

VEHICLE INFORMATION
Route Veh Weight Delvry Pickup Weight
Cube Delvry Pickup
no typ capcty weight weight
util capcty
cube
cube

Cube Vehicle
util description

350

305

87.1%

9999

.0% T310

13
14

1
1

350
350

327
320

0
0

93.4%
91.4%

9999
9999

0
0

0
0

.0% T310
.0% T310

16

350

315

90.0%

9999

.0% T310

18
19

1
1

350
350

235
280

0
0

67.1%
80.0%

9999
9999

0
0

0
0

.0% T310
.0% T310

23
24

1
1

350
350

318
229

0
0

90.9%
65.4%

9999
9999

0
0

0
0

.0% T310
.0% T310

29

350

305

87.1%

9999

.0% T310

*** DETAIL REPORT ON ROUTE NUMBER 3 ***


A T310 leaves at 5:01AM on day 1 from the depot at Vijayawada

Stop
No description
3 Ongole
Depot
Stop
No description

Stop
Drive Distance Time
Arrive
Depart
time to stop to stop wind
time Day
time Day Min
Min
Miles met?
-->Break 30 minutes
09:00AM
1 11:30AM
1 150
208.5
139 YES
-->Break 30 minutes
03:28PM
1 ------- -- --208.5
139
Stop volume
Weight
Cube

Inc cost to serve stop


In $ In $/unit

Capacity in use
Weight
Cube
87.1%
.0%

68

3 Ongole
305
0
834.00
2.7
Totals Weight: Del = 305 Pickups = 0 Cube: Del = 0 Pickups = 0
Route time:
Driving
Load/unload
Break
Total

7.0 hr
2.5
1.0
10.4 hr

Max allowed

21.0 hr

Route costs:
Driver (reg time)
Driver (over time)
Vehicle (mileage)
Fixed
Total

Distance:
To 1st stop
From last stop
On route
Total
Max allowed

.0

.0

139 mi
139
0
278 mi
9999 mi

$.00
.00
834.00
.00
$834.00

*** DETAIL REPORT ON ROUTE NUMBER 13 ***


A T310 leaves at 7:41AM on day 1 from the depot at Vijayawada

Stop
No description
18 Vuyyuru
22 Machilipatnam
26 Gudivada
Depot

Stop
Drive Distance Time
Arrive
Depart
time to stop to stop wind
time Day
time Day Min
Min
Miles met?
-->Break 30 minutes
09:00AM
1 10:00AM
1
60
49.5
33 YES
10:58AM
1 11:58AM
1
60
58.5
39 YES
-->Break 30 minutes
01:20PM
1 02:20PM
1
60
51.0
34 YES
03:30PM
1 ------- -- --70.5
47

Stop
No description

Stop volume
Weight
Cube

Inc cost to serve stop


In $ In $/unit

Capacity in use
Weight
Cube
93.4%
.0%
-6.00
-.2
82.3
.0
153.00
1.4
51.4
.0
21.00
.1
.0
.0
0 Cube: Del = 0 Pickups = 0

18 Vuyyuru
39
22 Machilipatnam
108
26 Gudivada
180
Totals Weight: Del = 327 Pickups

0
0
0
=

Route time:
Driving
Load/unload
Break
Total

3.8 hr
3.0
1.0
7.8 hr

Distance:
To 1st stop
From last stop
On route
Total

Max allowed

21.0 hr

Route costs:
Driver (reg time)
Driver (over time)
Vehicle (mileage)
Fixed
Total

Max allowed

33 mi
47
73
153 mi
9999 mi

$.00
.00
459.00
.00
$459.00

*** DETAIL REPORT ON ROUTE NUMBER 14 ***


A T310 leaves at 7:35AM on day 1 from the depot at Vijayawada

Stop
No description

Stop
Drive Distance Time
Arrive
Depart
time to stop to stop wind
time Day
time Day Min
Min
Miles met?
-->Break 30 minutes

69

19 Tenali

09:00AM
1 10:00AM
-->Break 30 minutes
01:33PM
1 02:33PM
04:40PM
1 -------

38 Palakolu
Depot
Stop
No description

Stop volume
Weight
Cube

60

54.0

36

YES

1
--

60
---

183.0
127.5

122
85

YES

Capacity in use
Weight
Cube
91.4%
.0%
19 Tenali
140
0
219.00
1.6
51.4
.0
38 Palakolu
180
0
513.00
2.8
.0
.0
Totals Weight: Del = 320 Pickups = 0 Cube: Del = 0 Pickups = 0
Route time:
Driving
Load/unload
Break
Total

6.1 hr
2.0
1.0
9.1 hr

Max allowed

21.0 hr

Route costs:
Driver (reg time)
Driver (over time)
Vehicle (mileage)
Fixed
Total

Inc cost to serve stop


In $ In $/unit

Distance:
To 1st stop
From last stop
On route
Total
Max allowed

36 mi
85
122
243 mi
9999 mi

$.00
.00
729.00
.00
$729.00

*** DETAIL REPORT ON ROUTE NUMBER 16 ***


A T310 leaves at 7:27AM on day 1 from the depot at Vijayawada
Stop
Drive Distance Time
Arrive
Depart
time to stop to stop wind
time Day
time Day Min
Min
Miles met?
-->Break 30 minutes
21 Nuzvid
09:00AM
1 09:30AM
1
30
63.0
42 YES
36 Tadepallegudem 10:45AM
1 12:15PM
1
90
75.0
50 YES
-->Break 30 minutes
39 Bhimavaram
01:39PM
1 03:09PM
1
90
54.0
36 YES
Depot
05:51PM
1 ------- -- --162.0
108
Stop
No description

Stop
No description

Stop volume
Weight
Cube

Inc cost to serve stop


In $ In $/unit

Capacity in use
Weight
Cube
90.0%
.0%
-63.00
-1.7
79.4
.0
-66.00
-.5
42.3
.0
93.00
.6
.0
.0
0 Cube: Del = 0 Pickups = 0

21 Nuzvid
37
36 Tadepallegudem
130
39 Bhimavaram
148
Totals Weight: Del = 315 Pickups

0
0
0
=

Route time:
Driving
Load/unload
Break
Total

5.9 hr
3.5
1.0
10.4 hr

Distance:
To 1st stop
From last stop
On route
Total

Max allowed

21.0 hr

Route costs:
Driver (reg time)
Driver (over time)
Vehicle (mileage)
Fixed
Total

Max allowed

42 mi
108
86
236 mi
9999 mi

$.00
.00
708.00
.00
$708.00

70

*** DETAIL REPORT ON ROUTE NUMBER 18 ***


A T310 leaves at 6:34AM on day 1 from the depot at Vijayawada

Stop
No description
24 Jaggayyapeta
37 Eluru
Depot

Stop
Drive Distance Time
Arrive
Depart
time to stop to stop wind
time Day
time Day Min
Min
Miles met?
-->Break 30 minutes
09:00AM
1 09:30AM
1
30
115.5
77 YES
-->Break 30 minutes
13:24AM
1 3:24PM
1 120
94.5
63 YES
5:00PM
1 ------- -- --115.5
77

Stop
No description

Stop volume
Weight
Cube

Capacity in use
Weight
Cube
10.6%
.0%
24 Jaggayyapeta
37
0
462.00
12.5
.0
.0
Totals Weight: Del = 37 Pickups = 0 Cube: Del = 0 Pickups = 0
Route time:
Driving
Load/unload
Break
Total

3.8 hr
.5
.5
4.8 hr

Max allowed

21.0 hr

Route costs:
Driver (reg time)
Driver (over time)
Vehicle (mileage)
Fixed
Total

Inc cost to serve stop


In $ In $/unit

Distance:
To 1st stop
From last stop
On route
Total
Max allowed

77 mi
63
136
276 mi
9999 mi

$.00
.00
828.00
.00
$828.00

*** DETAIL REPORT ON ROUTE NUMBER 19 ***


A T310 leaves at 8:26AM on day 1 from the depot at Vijayawada

Stop
No description
25 Hanuman
8 Chirala
27 Bapatia
16 Repalie
Depot

Arrive
time
-->Break
Junctio 09:00AM
11:11AM
-->Break
02:01PM
04:08PM
-->Break
07:49PM

Stop
No description

Stop volume
Weight
Cube

25 Hanuman Junctio
8 Chirala
27 Bapatia
16 Repalie
Totals Weight: Del = 280
Route time:
Driving
Load/unload

Stop
Drive Distance Time
Depart
time to stop to stop wind
Day
time Day Min
Min
Miles met?
30 minutes
1 10:00AM
1
60
3.0
2 YES
1 01:11PM
1 120
72.0
48 YES
30 minutes
1 03:01PM
1
60
19.5
13 YES
1 05:08PM
1
60
67.5
45 YES
60 minutes
1 ------- -- --100.5
67

50
98
82
50
Pickups

4.4 hr
5.0

0
0
0
0
=

Inc cost to serve stop


In $ In $/unit

Capacity in use
Weight
Cube
80.0%
.0%
-117.00
-2.3
65.7
.0
-57.00
-.6
37.7
.0
.00
.0
14.3
.0
108.00
2.2
.0
.0
0 Cube: Del = 0 Pickups = 0

Distance:
To 1st stop
From last stop

2 mi
67

71

Break
Total

2.0
11.4 hr

On route
Total

Max allowed

21.0 hr

Max allowed

Route costs:
Driver (reg time)
Driver (over time)
Vehicle (mileage)
Fixed
Total

106
175 mi
9999 mi

$.00
.00
525.00
.00
$525.00

*** DETAIL REPORT ON ROUTE NUMBER 23 ***


A T310 leaves at 4:23AM on day 1 from the depot at Vijayawada

Stop
No description
31 Amaiapuram
32 Kakinada

Depot

Arrive
time
-->Break
09:00AM
11:22AM
-->Break
-->Break
07:52PM

Stop
No description

Stop
Drive Distance Time
Depart
time to stop to stop wind
Day
time Day Min
Min
Miles met?
30 minutes
1 10:00AM
1
60
247.5
165 YES
1 01:22PM
1 120
82.5
55 YES
30 minutes
60 minutes
1 ------- -- --300.0
200

Stop volume
Weight
Cube

Capacity in use
Weight
Cube
90.9%
.0%
31 Amaiapuram
90
0
60.00
.7
65.1
.0
32 Kakinada
228
0
270.00
1.2
.0
.0
Totals Weight: Del = 318 Pickups = 0 Cube: Del = 0 Pickups = 0
Route time:
Driving
Load/unload
Break
Total

10.5 hr
3.0
2.0
15.5 hr

Max allowed

21.0 hr

Route costs:
Driver (reg time)
Driver (over time)
Vehicle (mileage)
Fixed
Total

Inc cost to serve stop


In $ In $/unit

Distance:
To 1st stop
From last stop
On route
Total
Max allowed

165 mi
200
55
420 mi
9999 mi

$.00
.00
1260.00
.00
$1260.00

*** DETAIL REPORT ON ROUTE NUMBER 24 ***


A T310 leaves at 5:12AM on day 1 from the depot at Vijayawada

Stop
No description
34 Tanuku
35 Nidadvole
33 Kovvur
Depot

Stop
Drive Distance Time
Arrive
Depart
time to stop to stop wind
time Day
time Day Min
Min
Miles met?
-->Break 30 minutes
09:00AM
1 10:00AM
1
60
198.0
132 YES
10:33AM
1 11:33AM
1
60
33.0
22 YES
11:55AM
1 12:55PM
1
60
22.5
15 YES
-->Break 30 minutes
05:04PM
1 ------- -- --219.0
146

72

Stop
No description

Stop volume
Weight
Cube

Inc cost to serve stop


In $ In $/unit

Capacity in use
Weight
Cube
65.4%
.0%
60.00
.4
27.1
.0
-9.00
-.2
12.9
.0
81.00
1.8
.0
.0
0 Cube: Del = 0 Pickups = 0

34 Tanuku
134
35 Nidadvole
50
33 Kovvur
45
Totals Weight: Del = 229 Pickups

0
0
0
=

Route time:
Driving
Load/unload
Break
Total

7.9 hr
3.0
1.0
11.9 hr

Distance:
To 1st stop
From last stop
On route
Total

Max allowed

21.0 hr

Route costs:
Driver (reg time)
Driver (over time)
Vehicle (mileage)
Fixed
Total

Max allowed

132 mi
146
37
315 mi
9999 mi

$.00
.00
945.00
.00
$945.00

*** DETAIL REPORT ON ROUTE NUMBER 29 ***


A T310 leaves at 6:23AM on day 1 from the depot at Vijayawada

Stop
No description
43 Piduguralia
10 Addanki
13 Vinukonda
12 Narasarapet
Depot

Arrive
time
-->Break
09:00AM
11:57AM
-->Break
01:52PM
03:56PM
-->Break
07:49PM

Stop
No description

Stop
Drive Distance Time
Depart
time to stop to stop wind
Day
time Day Min
Min
Miles met?
30 minutes
1 10:00AM
1
60
127.5
85 YES
1 12:27PM
1
30
117.0
78 YES
30 minutes
1 02:52PM
1
60
55.5
37 YES
1 04:56PM
1
60
63.0
42 YES
60 minutes
1 ------- -- --114.0
76

Stop volume
Weight
Cube

Inc cost to serve stop


In $ In $/unit

Capacity in use
Weight
Cube
87.1%
.0%
147.00
1.8
64.3
.0
150.00
2.5
47.1
.0
54.00
.8
28.6
.0
.00
.0
.0
.0
0 Cube: Del = 0 Pickups = 0

43 Piduguralia
80
10 Addanki
60
13 Vinukonda
65
12 Narasarapet
100
Totals Weight: Del = 305 Pickups

0
0
0
0
=

Route time:
Driving
Load/unload
Break
Total

8.0 hr
3.5
2.0
13.4 hr

Distance:
To 1st stop
From last stop
On route
Total

Max allowed

21.0 hr

Route costs:
Driver (reg time)
Driver (over time)
Vehicle (mileage)
Fixed
Total

Max allowed

85 mi
76
157
318 mi
9999 mi

$.00
.00
954.00
.00
$954.00

73

CHAPTER 8
FORECASTING SUPPLY CHAIN REQUIREMENTS
4
(a) The answer to this question is aided by using the FORECAST module in
LOGWARE. A sample calculation is shown as carried out by FORECAST. The
results are then summarized from FORECAST output. An example calculation for an
= 0.1 is shown. Other values would be used, ranging 0.01 to 1.0.
We first calculate a starting forecast by averaging the first four weekly
requirements. That is,

[2,056 + 2,349 + 1,895 + 1,514]/4 = 1,953.50


Now we backcast this value and start the forecast at time 0. Thus, the forecasts and
the associated errors would be:

F1
F2
F3
F4
F5
F6
F7
F8
F9
F10
F11

=
=
=
=
=
=
=
=
=
=
=

.1(2056)
.1(2349)
.1(1895)
.1(1514)
.1(1194)
.1(2268)
.1(2653)
.1(2039)
.1(2399)
.1(2508)

+
+
+
+
+
+
+
+
+
+

.9(1953.50)
.9(1963.75)
.9(2002.20)
.9(1991.48)
.9(1943.73)
.9(1868.76)
.9(1908.00)
.9(1982.50)
.9(1988.15)
.9(2029.24)

Forecast
Error
1953.50
=
1963.75
=
2002.28
=
1991.48
=
1943.73 -749.73
=
1868.76
399.24
=
1908.00
745.00
=
1982.50
56.50
=
1988.15
410.85
=
2029.24
478.76
=
2077.12
Total squared error

Squared
error

562,095.07
159,392.58
555,025.00
3,192.25
168,797.72
229,211.14
1,677,713.76

The standard error of the forecast is:


SF

Total squared error


1,677,713.76

528.79
6
N

Note: FORECAST does not use N-1 in the denominator.


Repeating this type of analysis, the following table can be developed. The results
from FORECAST are shown.

74


.01
.05
.1
.2
.5
1.0

SF
528.72
528.42
528.46
528.89
535.55
566.07

The that minimizes S F is 0.05.


(b) The forecast errors are computed in part a.
(c) If we assume that the errors are normally distributed around the forecast, we can then
construct a 95% confidence band on the forecast. That is, if Y is the actual volume in
period 11, then the range of the forecast (F11 = 2,017.81 for = 0.05) will be:
Y = F11 + z S F
= 2,017.81 + 1.96528.42
Then,
982.11 Y 3,053.51
All values are in thousands.
5
(a) & (b) The solution to this problem was aided by the use of the exponential
smoothing module in FORECAST. Using the first four week's data to initialize the
level/trend version of the exponential smoothing model and setting and equal to
0.2, the forecast for next week is F11 = 2,024.47, with a standard error of the forecast
of S F = 171.28.

(c) Assuming that the forecast errors are normally distributed around F11, a 95%
statistical confidence band can be constructed. The confidence band is:
Y = F11 + z S F
= 2,024.47 + 1.96171.28
where z = 1.96 for 2.5% of the area under the two tails of a normal distribution. The
range of the actual weekly volume is expected to be:
1,688.76 Y 2,360.18
6

75

(a) The data may be restated as shown below.

Sales, S
27,000
70,000
41,000
13,000
30,000
73,000
48,000
15,000
34,000
82,000
51,000
16,000
500,000

t
1
2
3
4
5
6
7
8
9
10
11
12
78

St
27,000
140,000
123,000
52,000
150,000
438,000
336,000
120,000
306,000
820,000
561,000
192,000
3,265,000

t2
1
4
9
16
25
36
49
64
81
100
121
144
650

Trend
value,a
St
41,087
41,192
41,298
41,403
41,508
41,613
41,719
41,824
41,929
42,035
42,140
42,245

Seasonal
indexb
0.66
1.70
0.99
0.32
0.72
1.75
1.15
0.36
0.81
1.95
1.21
0.38

Computed from the linear trend line. For example, for period 1,
S1 = 40,981.6 + 105.31 = 41,087.
b
The ratio of the actual sales S to the trend line value St.
For example, for period 1, the seasonal index is 27,000/41,087 = 0.66.

Given the values from the above table and that t = 78/12 = 6.5, N = 12, and S =
500,000/12 = 41,666, the coefficients in the regression trend line would be:
b

S t N S t 3,265,000 12 41,666 6.5 105.3


650 12 6.5
t N t
2

and
a S b t 41,666 105.3 6.5 40,9816
.
Therefore, the trend value St for any period t would be:
St = 40,981.6 + 105.3t
(b) The seasonal factors are determined by the ratio of the actual sales in a period to the
trend value for that period. For example, the seasonal factor for period 12 (4th
quarter of last year) would be 16,000/42,245 = 0.38. This and the seasonal factors for
all past quarters are shown in the previous table.
(c) The forecasts using the seasonal factors from the last 4 quarters are as follows.
Seasonal
76

t
13
14
15
16

St
42,351
42,456
42,561
42,666

factors
0.81
1.95
1.21
0.38

Forecast
34,304
82,789
51,499
16,213

7
An exponential smoothing model is used to generate a forecast for period 13 (January of
next year). The sales for January through April are used to initialize the model, and an
= 0.2 is used as the smoothing constant. The FORECAST module is used to generate the
forecast. The results are summarized as follows.

Forecast, F13
Forecast error, SE

Region 1
219.73
26.89

Region 2
407.04
25.50

Region 3
303.30
17.54

Combined
938.26
61.41

Note that the sum of the forecasts by region nearly equals the forecast of the combined
usage. However, whether a by-region forecast is better than an overall forecast that is
disaggregated by region depends on the forecast error. The standard error of the forecast
is the best indicator. A comparison of a bottoms-up forecast developed from regional
forecasts to that of a forecast from combined data can be based on the law of variances.
That is, if the usage rates within the regions are independent of each other, the estimate of
the total error can be built from the individual regions and compared to that of the
combined usage data. The total forecast error (variance) from the individual regions ST2
might be estimated as the weighted average of the variances as follows.

ST2

F
F1 2 F2 2
S E1
S E2 3 S E23
FC
FC
FC

where
Fi = forecasts of each region
FC = forecast based on combined data
S E2 = variance of the forecast in each region
i

2
T

= total variance of the forecast based on regional data

Therefore,
219.73
407.04
30330
.
26.89 2
25.50 2
17.54 2
930.07
930.07
930.07
0.236 723.07 0.438 650.25 0.326 307.65
555.74

S T2

Then,

77

S T 555.74 2357
.
Since ST < SC, it appears that a bottom-up, or regional, forecast will have a lower error
than a top-down forecast.
9
(a) See the plot in Figure 8-1. It shows that there is a seasonal component with a very
slight trend to the data as well as some random, or unexplained, variation.
300

Average monthly unit prices

250

200

150

100

50

Jly

Oct

Apr

Jan

Oct

Jly

Apr

Jan

Oct

Jly

Apr

Jan

Oct

Jly

Apr

Jan

Oct

Jly

Apr

Jan

Time, months

FIGURE 8-1 Plot of time series data for Problem 9

(b) A time series model typically will involve only two components: trend and
seasonality. Using 2 years of data should be sufficient to establish an accurate trend
line and the seasonal indices. We can develop the following table for computing a
regression line and seasonal indices.

78

Prices,
Pt
211
210
214
208
276
269
265
253
244
202
221
210
215
225
230
214
276
261
250
248
229
221
209
214
5,575

Time,
t
1
2
3
4
5
6
7
8
9
10
11
12
13
14
15
16
17
18
19
20
21
22
23
24
300

Pt
211
420
642
832
1380
1614
1855
2024
2196
2020
2431
2520
2795
3150
3450
3424
4692
4698
4750
4960
4809
4862
4807
5136
69,678

t2
1
4
9
16
25
36
49
64
81
100
121
144
169
196
225
256
289
324
361
400
441
484
529
576
4,900

Trend,a
Tt
232.4
232.4
232.4
232.4
232.2
232.3
232.3
232.3
232.3
232.3
232.3
232.2
232.3
232.2
232.3
232.3
232.2
232.2
232.2
232.2
232.2
232.2
232.2
232.2

Seasonal
indexb
0.91
0.90
0.92
0.90
1.19
1.16
1.14
1.09
1.05
0.87
0.95
0.90
0.93
0.97
0.99
0.92
1.19
1.12
1.08
1.07
0.99
0.95
0.90
0.92

St

0.92
0.93
0.96
0.91
1.19
1.14
1.11
1.08
1.02
0.91
0.92
0.91

Computed from the trend regression line. For example, the period 1 trend is T1 =
232.39 - 0.0081 = 232.4.
b
The seasonal index is the ratio of the actual price to the trend for the same period.
For example, the period 1 seasonal index is 211/232 = 0.91.

We also have N = 24, t = 300/24 = 12.5, and P = 5575/24 = 232.29.


Now,
b

P t N P t 69,678 24( 232.29)(12.5) 0.008


4,900 24(12.5)
t N t
2

and
a P t b t 232.29 ( 0.008)(12.5) 232.39
Therefore, the trend equation is:

79

Tt 232.29 0.008 t
Note that the trend is negative for the last two years of data, even though the 5-year
trend would appear to be positive.
Now, computing the trend value Tt for each value of t gives the results as shown
in the previous table. The seasonal index is a result of dividing Pt by Tt for each
period t. The indices are averaged for corresponding periods that are one year apart.
Forecasting into the 5th year shows the potential error in the method. That is, for
January of the 5th year, the forecast is Ft = TtSt-12, or F25 = [232.39 0.00825][0.92]
= 213.6. Repeating for each month, we have:

t
25
26
27
28
29
30
31
32
33
34
35
36
a

Actual
price
210
223
204
244
274
246
237
267
212
211
188
188

Forecast Forecast
price
error
213.6
- 3.6
215.6
7.1
222.9
-18.9
211.3
32.7
276.3
- 2.3
264.6
-18.6
257.7
-20.7
250.7
16.3
236.8
-24.8
211.2
- 0.2
213.5
-25.5
211.2
-23.2
Total squared error

Squared
error
13.0
50.4
357.2
1069.3
5.3
345.9
428.5
265.7
615.0
0.0
51.0
538.2
3,739.5

Revised
seasonala
0.91

The seasonal index for period 25 is .90. The average of the seasonal index for period 25 12 = 13,
and this period is (0.92 + 0.90)/2 = 0.91.

The standard error of the forecast is S F 3,739.5 / (12 2) 19.34 . Now, the forecast
for period 37 would be:
F37 ( 232.39 0.008 37)( 0.91) 21121
.
(c) Using the exponential smoothing module in the FORECAST software, the forecast
for the coming period is F = 201.26, with SF = 17.27. The smoothing constants given
in the problem are the "best" that FORECAST could find.
(d) Each model should be combined according to its ability to forecast accurately. We
can give each a weight in proportion to its forecast error, or standard error of the
forecast (SF). Hence, the following table can be developed.

80

(1)
Model type
Regression
Exp. smooth.
Total

Forecast error
19.34
17.27
36.61

(2) = (1)/36.61
Proportion of
total error
0.528
0.472
1.000

(3)=1/(2)
Inverse of error
proportion
1.894
2.119
4.013

(4)=(3)/4.013
Model weights
0.472
0.528
1.000

Therefore, each of the model results is weighted according to the model weights.
The weighted forecast for the upcoming January would be:
(1)
Model type
Regression
Exp. smooth.

(2)

Forecast Model weight


211.21
0.472
201.26
0.528
Weighted forecast

(3)=(1)(2)
Weighted
proportion
99.69
106.27
205.96

In a similar fashion, we can weight the forecast error variances to come up with a
weighted forecast error standard deviation SFw. That is,
S Fw 0.472 19.34 2 0.528 17.27 2 18.28
A 95% confidence band using the combined results might be constructed as:
Y = 205.96 z18.28
where z is 1.96 for 95% of the area under the normal distribution.
Y = 205.96 1.9618.28
Hence, we can be 95% sure that the actual price Y will be within the following range:
170.13 Y 241.79
10
The plot of the sales data is shown in Figure 8-2. The plot reveals a high degree of
seasonality with a noticeable downward trend. A level-trend-seasonal model seems
reasonable.

(b) Using the search capability within the FORECAST software, a Level-Trend-Seasonal
form of the exponential smoothing model was found to give the lowest forecast error.
A 14-period initialization and 6 periods to compute error statistics were used. The
respective smoothing constants were = 0.01, = 0.08, and = 0.60. This produced

81

a forecast for the upcoming period (January 2004) of F = 6,327.60 and a standard
error of the forecast of SF = 1,120.81.
30000

Aggregate sales in 000s

25000

20000

15000

10000

Oct

Jly

Apr

Jan

Oct

Jly

Apr

Jan

Oct

Jly

Apr

Jan

Oct

Jly

Apr

Jan

Oct

Jly

Apr

Jan

5000

Time, months

FIGURE 8-2 Plot of Time Series Data for Hudson Paper Company

(c) Assuming that the forecast errors are normally distributed around the forecast, a 95%
confidence band on the forecast is given by:
Y = F + zSF
Y = 6,327.60 1.961,120.81
where z = 1.96 for 95% of the area under the normal distribution curve. Therefore,
we can be 95% sure that the actual sales Y should fall within the following limits:
4,130.8 Y 8,524.4
11
(a) For A569, the BIAS = 165,698 and the RMSE = 126,567 when using the 3-month
moving average. However, if a level only exponential smoothing model with an =
0.10, the BIAS drops to 9,556 and the RMSE is 118,689. The model fits the data
better and there is a slight improvement in the forecasting accuracy.
For A366, the BIAS = 18,231 and the RMSE = 144,973 when using the 3-month
moving average. A level-trend-seasonal model offers the best fit, but it is suspect
since the data show a high degree of random variability rather than seasonality.
Overall, a simple level-only model is probably better in practice. The model has an
= 0.08, a BIAS = 3,227, and a RMSE = 136,256. This is an improvement over the
3-month moving average.

82

(b) Using the level-only models, the forecast for October for A569 = 193,230 and for
A366 = 603,671.
(c) The 3-sigma (99.7%) confidence band on the forecasts would be:
For A569, Y = 193,230 3(118,689), or 0 Y 549,297.
For A366, Y = 603,671 3(136,256), or 194,903 Y 1,012,439.
The actual October usage falls within the 3-sigma confidence bands for each of
these products. The difference of the actual from the forecast for each product is
attributable to the substantial variability in the data, which is characteristic of
purchasing in the steel processing industry.

83

WORLD OIL
Teaching Note
Strategy
The purpose of this case study is to allow students to develop an appropriate forecasting
model for some time series data. Discussion may begin with the nature of this
productone with which most students should be very familiar. Based on the many
available forecasting approaches, students should be encouraged to select several for
consideration. In this note, both exponential smoothing and time series decomposition
are evaluated. Both are appropriate here because (1) they can project from historical time
series data, (2) they can handle seasonality, which appears to be present in the data, (3)
there is enough data to construct and test the models, and (4) the forecast is for a short
period into the future.
Assistance with the computational aspects of this problem is available with the use of
the FORECAST module in the LOGWARE software.
Answers to Questions
(1) Develop a forecasting procedure for this service station. Why did you select your
method?

Both exponential smoothing and time series decomposition forecasting methods are
tested using the FORECAST module in LOGWARE. For exponential smoothing, an
initialization period of one seasonal cycle (52 weeks) plus two weeks are used for a total
of 54 weeks, a minimum requirement in FORECAST. The last 30 weeks of data is used
for computing the error statistics. This number of periods is arbitrary, but seems
reasonably large so as to give stable statistical values. We wish to minimize the forecast
error over time, and FORECAST computes both MAD and RMSE statistics that can be
used to make comparisons among model types. Testing the various exponential
smoothing model types and the time series gives the following statistics.
Smoothing
constants

Model type
Level only....
Level-trend...
Level-seasonal
Level-trendseasonal......
TS decomp.....

.4
.2
.3

.5

.01 .2

1.0

MAD
37.82
45.85
38.97

BIAS
-5.27
7.13
11.30

RMSE
67.61
67.80
45.71

.4

30.27
59.46

-6.05
37.18

44.17
71.85

Forecast
week 6 of this
year
817.35
860.26
648.75
770.74
731.33

The MAD and RMSE statistics show how well the forecast has been able to track
historical fuel usage rates. They are an indication of the accuracy of the forecasting
process in the future on the average. We favor forecasting methods that can minimize
these statistics. In this case, the Level-Trend-Seasonal version of the exponential

84

smoothing model seems to do this best. Both MAD and RMSE are the lowest for this
model type among the alternatives.
Further evidence of the performance of a forecasting method is obtained from a plot
of the forecast against the actual usage rates. This is shown in Figure 1. Note that the
Level-Trend-Seasonal model tracks the usage rates quite well, especially in the more
recent weeks. The modeling process has likely stabilized in the last 30 weeks of the data
and is now tracking quite well.

FIGURE 1

Fit of Level-Trend-Seasonal Exponential Smoothing Model to Fuel


Usage Data on Mondays of the Week

(2) How should the periods of promotions, holidays, or other periods where usage rates
deviate from normal patterns, be handled in the forecast?
If the deviations occur at the same time within the seasonal cycle and with the same
relative intensity, no special procedures are required. The adaptive characteristic of the
exponential smoothing process will automatically incorporate these deviations into the
forecast. However, when the deviations are not regular, as promotions may be timed
irregularly, they may best be handled as outliers in the time series and eliminated from
the time series. The model may be fit without the outliers, and then the effect of them
treated as modifications to the forecast. These modifications can be handled manually.

85

(3) Forecast next Monday's fuel usage and indicate the probable accuracy of the
forecast.
From the Level-Trend-Seasonal exponential smoothing model developed in question 1,
where the smoothing constants are = 0.01, = 0.2, and = 0.4, the forecast for Monday
of week 6 would be 771 gallons. However, this forecast only represents the average fuel
usage.
Determining the accuracy of the forecast requires that the forecast track the mean of
the actual usage, i.e., a bias of 0, and that the forecast errors be normally distributed.
While the BIAS (sum of the forecast errors over the last 30 weeks) is not exactly 0, and
will not likely ever be so, it is low (-6.05), such that we will assume good tracking by the
forecast model. A histogram of the forecast errors can reveal whether they follow the
familiar bell-shaped pattern. Such a histogram is given below. We can conclude that
while the errors are not precisely normally distributed, we cannot reject the idea that they
did not come from a normally distributed population. A goodness-of-fit test could be
used to check this assumption. Although this test is not performed here, it is quite
forgiving, such that the normal distribution of errors assumption is not likely to be
rejected where the data show a reasonably normal distribution pattern. The distribution
here qualifies.
We can now proceed with developing a 95% confidence band around the forecast.
The forecast of the actual fuel usage rate Y will be:
F z ( F ) Y F z ( F )

where F is the standard error of the forecast. F is the forecast, and z is the number of
standard deviations for 95% of the area under a normal distribution. FORECAST
computes the root mean squared error (RMSE) as:
N

(A

RMSE =

Ft ) 2

t =1

86

HISTOGRAM FOR FORECAST ERROR OF LAST 30 WEEKS


Class Width = 20.0000
Number of Classes = 10
MID CLASS
< -80.0000
-70.0000
-50.0000
-30.0000
-10.0000
10.0000
30.0000
50.0000
70.0000
90.0000
110.0000
>= 120.0000

0%
50%
100%
+----+----+----+----+----+----+----+----+----+----+
|
|
|******
|
|***
|
|********
|
|********
|
|*****
|
|********
|
|******
|
|*
|
|*
|
|
|
|
|
+----+----+----+----+----+----+----+----+----+----+

Since RMSE is uncorrected for degrees of freedom lost, we apply a correction factor
(CF) as a multiplier to RMSE to get the unbiased estimate of the standard error of the
forecast ( F ):
CF =

N
N-n

where n is the number of degrees of freedom lost in the model building process. We
estimate n to be the number of smoothing constants in the model, or three in this case.
Hence,

F RMSE CF
30
30 3
. 1054
.
4417
.
4417

46.56

Now, with z@95% = 1.96 from a normal distribution table, we can be 95% confident that
the true 87-octane fuel usage Y on Monday of week 6 will be:
771 1.96(46.56) < Y < 771 + 1.96(46.56)
680 < Y < 862 gallons

87

METRO HOSPITAL

You are the materials manager at Metro Hospital. Approximately one year ago, the
hospital began stocking a new drug (Ziloene) that helps the healing process for wounds
and sutures. It is your responsibility to forecast and order the monthly supply of Ziloene.
The goal is to minimize the combined cost of overstocking and understocking the drug.
Orders are placed and received at the beginning of the month and demand occurs
throughout the month. The following demand and cost data have been compiled.
Costs. If more is ordered than is demanded, a monthly holding cost of $1.00 per case
is incurred. If less is ordered than is demanded, a $2.00 per case lost sales cost is
incurred. The drug has a short shelf life, and any overstocked product at the end of the
month is worthless and no longer available to meet demand.
Demand. The demand for the twelve months of last year was:
Month
Cases

1
43

2
36

Last year's demand


3
4
5
6
7
8
24 69 34 75 90 67

9
59

10
51

11
77

12
50

You believe this demand to be representative of Metro's normal usage pattern.

FIGURE 1 Plot of last year's monthly demand in cases

88

Decision Worksheet
Month
1 (13)
2 (14)
3 (15)
4 (16)
5 (17)
6 (18)
7 (19)
8 (20)
9 (21)
10 (22)
11 (23)
12 (24)
Total

Month
1 (25)
2 (26)
3 (27)
4 (28)
5 (29)
6 (30)
7 (31)
8 (32)
9 (33)
10 (34)
11 (35)
12 (36)
Total

Cases
ordered

Actual
demand

Over @
$1/case

Short @
$2/case

Cost, $

Cases
ordered

Actual
demand

Over @
$1/case

Short @
$2/case

Cost, $

89

METRO HOSPITAL
Exercise Note
Purpose
Metro Hospital is an in-class exercise designed to illustrate the relationship between good
forecasting and the control of inventory related costs. It shows that accurate forecasting
is a primary factor in minimizing inventory costs. Participants in this exercise use a
variety of methods, often intuition, to forecast demand and to come up with a purchase
quantity. Their performance is measured as over- or understock costs. Using a simple
exponential smoothing forecasting model and an understanding of the standard deviation
of the forecast, an effective purchase plan can be constructed. This process results in
costs that are significantly lower than the majority of the participants are able to achieve
using intuitive methods.
Administration
The descriptive material and the decision worksheet are to be distributed to the class at
the time that the exercise is conducted. To hand out the material ahead of time may take
away much of the drama from the exercise. About one half hour should be scheduled for
running the exercise.
The instructor asks the class to make a decision regarding the size of the order to be
placed in the upcoming period and to record it on the worksheet. The participants are
then informed of the demand for that period from Table 1 after the simulated time of one
month has passed. Given that they now know the actual demand for the period, the
participants are asked to record their costs and then to place an order for the next period.
The pattern is repeated for at least twelve months, a full seasonal cycle. The participants
are asked to sum their costs and to report them to the exercise leader. They are displayed
in a public place, such as a chalkboard, for all to see. Then, the exercise leader
announces his or her cost level that was achieved using a disciplined approach using a
simple forecasting procedure and some basic statistics.
TABLE 1 Actual Demand for Period 13 Through 36
Period
13 14 15 16 17 18 19 20
Demand
47 70 55 38 90 24 65 65

21
23

22
55

23
85

24
66

Period
Demand

33
45

34
70

35
50

36
56

25
53

26
64

27
61

28
63

29
65

30
38

31
80

32
88

Quantitative Analysis
The demand series was generated using a normal distribution with a reasonably high
variance and a very slight upward trend. To illustrate the use of a quantitative approach
to forecasting, an exponential smoothing model was selected, although other methods
such as time series decomposition would also be appropriate. The twelve historical data
points were submitted to the FORECAST module in LOGWARE. A 3-month
initialization period and a 3-month time period for computing error statistics were
chosen. The smoothing constants for the level, level-trend, and level-trend-seasonal
models were examined. Based on the root mean squared error (RMSE), the best model

90

was the level-trend-seasonal (RMSE = 13.59), but the level model with = 0.19 and
RMSE = 13.89 performed very well and is used here. The model is:
Ft 1 0.19 At 0.81Ft

where
Ft 1 forecast for next period t 1
At actual demand for current period t
Ft forecast for current period t
LOGWARE gives a forecast value of 58.1 and this is used as the forecast value for
period 13. Applying this simple, level only model to the second year demand as it is
revealed in each period gives the following forecast values.
TABLE 2 Simple Exponential
Smoothing Forecast Values
for the Next Year
Actual
Period
demand
Forecast
13
47
58.1
14
70
56.0
15
55
58.7
16
38
58.0
17
90
54.2
18
24
61.0
19
65
54.0
20
65
56.1
21
23
57.8
22
55
51.2
23
85
51.9
24
66
54.6

Now we must determine the order quantity. It can be calculated from


QF
z(RMSE)
t 1

Recall the RMSE was 13.89 for this model. To be precise, we calculate z by trial and
error. The following order quantity and cost computations can be made for a z value of
0.8 (Table 3).

91

TABLE 3

Period
13
14
15
16
17
18
19
20
21
22
23
24
*

Purchase Order Quantity and Associated Inventory Costs


Actual
Order
Units
Units
demand
Forecast
quantity
over
short
47
58.1
69*
22
70
56.0
67
3
55
58.7
70
15
38
58.0
69
31
90
54.2
65
25
24
61.0
72
48
65
54.0
65
65
56.1
67
2
23
57.8
69
46
55
51.2
62
7
85
51.9
63
22
66
54.6
66

Cost, $
22
6
15
31
50
48
0
2
46
7
44
0
271

Q = 58.1 + 0.8(13.89) = 69.21, or 69

We see from the following graph (Figure 1) that z = 0.8 is optimal.


310
305
300

Cost, $

295
290
285
280
275
270
265
0

0.2

0.4

0.6

0.8

1.2

1.4

FIGURE 1 Plot of Total Annual Costs Against the Factor z

Figure 2 graphically shows the good purchase pattern of Table 3.

92

100
Order
quantity

90
80

Cases

70
60
50
40

Demand
Forecast

30
20
10
0
1 2 3 4 5 6 7 8 9 10 11 12 13 14 15 16 17 18 19 20 21 22 23 24
Time period
FIGURE 2 Plot of Forecast and Purchase Order Quantity on Product Demand
Summary
The exercise leader should discuss that one of the problems with intuitively forecasting
demand is overreacting to randomness in the demand pattern. This has the effect of
causing extreme over and short costs in inventories. A model for short term forecasting
that is integrated into the purchasing and inventory control process can help to avoid
these extremes and give lower costs. Several forecasting models may perform well, such
as exponential smoothing, a simple moving average, a regression model, or a times series
decomposition model. One of the most practical for inventory control purposes is the
exponential smoothing model. The results from a simple, level only model were
illustrated above using the same information that was available to the participants.
Recognizing that it is less costly to order too much than to order too little, the
purchase quantity should exceed the forecast by some margin. The astute participant will
likely approximate the standard deviation of demand from the range of the demand
values, that is, = (Max - Min)/6. Then, one or two might be used to add a margin of
safety to the forecast and size of the purchase order. This simple approximation
procedure can lead to reasonable results.

93

CHAPTER 9
INVENTORY POLICY DECISIONS
1
The probability of finding all items in stock is the product of the individual probabilities.
That is,

(0.95)(0.93) (0.87) (0.85) (0.94) (0.90) = 0.55


2
(a) The order fill rate is the weighted average of filling the item mix on an order. We can
setup the following table.

(1)

(2)
Frequency
Order Item mix probabilities
of order
0.20
1
.95.95.95.90.90 = .69
2
0.15
.95.95.95 = .86
3
0.05
.95.95.90.90 = .73
4
0.15
.95.95.95.95.95.90.90 = .62
5
0.30
.95.95.90.90.90.90 = .59
6
0.15
.95.95.95.95.95 = .77
Order fill rate

(3)=(1)(2)
Marginal
probability
0.139
0.129
0.037
0.094
0.178
0.116
0.693

Since 69.3% < 92%, the target order fill rate is not met.
(b) The item service levels that will give an order fill rate of 92% must be found by trial
and error. Although there are many combinations of item service levels that can
achieve the desired service level, a service level of 99% for items A, B, C, D, E, and
F, and 97% to 98% for the remaining items would be about right. The order fill rates
can be found as follows.
(1)
Order
1
2
3
4
5
6

Item mix probabilities


(.99)3(.975)2 = .922
(.99)3 = .970
(.99)2(.975)2 = .932
(.99)5(.975)2 = .904
(.99)2(.975)4 = .886
(.99)5 = .951

(2)
Frequency
of order
0.20
0.15
0.05
0.15
0.30
0.15
Order fill rate

(3)=(1) (2)
Marginal
probability
0.184
0.146
0.047
0.136
0.266
0.143
0.922

94

This is a problem of push inventory control. The question is one of finding how many of
120,000 sets to allocate to each warehouse. We begin by estimating the total
requirements for each warehouse. That is,
Total requirements = Forecast + zForecast error
From Appendix A, we can find the values for z corresponding to the service level at each
warehouse. Therefore, we have:

Warehouse
1
2
3
4
Total

(1)
Demand
forecast, sets
10,000
15,000
35,000
25,000
85,000

(2)
Forecast
error, sets
1,000
1,200
2,000
3,000

(3)
Values
for z
1.28
1.04
1.18
1.41

(4)=(1)+(2)(3)
Total requirements, sets
11,280
16,248
37,360
29,230
94,118

We can find the net requirements for each warehouse as the difference between the
total requirements and the quantity on hand. The following table can be constructed.

Warehouse
1
2
3
4

(1)
Total
requirements
11,280
16,248
37,360
29,230
94,118

(2)

(3)=(1)(2)

(4)

(5)=(3)(4)

On hand
quantity
700
0
2,500
1,800

Net requirements
10,580
16,248
34,860
27,430
89,118

Proration of
excess
3,633
5,450
12,716
9,083
30,882

Allocation
14,213
21,698
47,576
36,513
120,000

There is 120,000 89,118 = 30,882 sets to be prorated. This is done by assuming that
the demand rate is best expressed by the forecast and proportioning the excess in relation
to each warehouse's forecast to the total forecast quantity. That is, for warehouse 1, the
proration is (10,000/85,000)30,882 = 3,633 sets. Prorations to the other warehouses are
carried out in a similar manner. The allocation to each warehouse is the sum of its net
requirements plus a proration of the excess, as shown in the above table.
4
(a) The reorder point system is defined by the order quantity and the reorder point
quantity. Since the demand is known for sure, the optimum order quantity is:
Q * 2 DS / IC 2(3,200 )(35) / ( 015
. )(55) 164.78, or 165 cases

The reorder point quantity is:

95

. 92 units
ROP d LT (3,200 / 52) 15
(b) The total annual relevant cost of this design is:
TC D S / Q I C Q * / 2
. )(55)(164.78 ) / 2
(3,200)(35) / 164.78 ( 015
679.69 679.97
$1,359.66
(c) The revised reorder point quantity would be:

ROP (3,200 / 52) 3 185 units .


The ROP is greater than Q*. It is possible under these circumstances the reorder
quantity may not bring the stock level above the ROP quantity. In deciding whether
the ROP has been reached, we add any quantities on order or in transit to the quantity
on hand as the effective quantity in inventory. Of course, we start with an adequate
in-stock quantity that is at least equal to the ROP quantity.
5
(a) The economic order quantity formula can be used here. That is,
Q * 2 DS / IC 2(300)(8,500 ) / ( 010
. )(8,500 ) 77.5, or 78 students

(b) The number of times that the course should be offered is:
. or about 4 times per year
N * D / Q * 300 / 77.5 39,
6
This is a single-period inventory control problem. We have:

Revenue = $350/unit
Profit = $350 $250 = $100/unit
Loss = 0.2250 = $50/unit
Therefore,
CPn

100
0.667
100 50

Developing a table of cumulative frequencies, we have:

96

Quantity
50
55
60
65
70
75

Frequency
0.10
0.20
0.20
0.30
0.15
0.05
1.00

Cumulative
frequency
0.10
0.30
0.50
0.80 Q*
0.95
1.00

CPn lies between quantities of 60 and 65. We round up and select 65 as the optimal
purchase order size.
7
This question can be treated as a single-order problem. We have:

Revenue = 1 + 0.01 = $1.01/$


Cost/Loss = 0.10(2/365) = $0.00055/$ which is the interest expense for 2 days
Profit = 1.01 1.00055 = $0.00945/$
and
CPn

0.00945
0.945
0.00945 0.00055

For an area under the normal curve of 0.945 (see Appendix A), z = 1.60.
The planned number of withdrawals is:
Q* = D + z D = 120 + 1.60(20) = 152.00
The amount of money to stock in the teller machine over 2 days would be:
Money = Q*75 = 152.0075 = $11,400
8
This is a single-period inventory control problem.

(a) We have:
Profit = 400 320
Loss = 320 300
Then,

97

CPn

400 320
0.80
( 400 320) (320 300)

We now need to find the sales that correspond to a cumulative frequency of 0.80. In the
following table:

Sales
500
750
1,000
1,250
1,500

Frequency
0.2
0.2
0.3
0.2
0.1
1.0

Cumulative
frequency
0.2
0.4
0.7
0.9 Q*
1.0

Q* lies between 1,000 and 1,200 in the cumulative frequency table. We choose to
roundup to Q* = 1,250 units.
(b) Carrying the excess inventory to next year,
CPn

80
0.556
80 ( 0.2 320)

where the loss is the cost of holding a unit until the next year. The Q* now lies between
750 and 1,000 units. We choose 1,000 units. Holding the excess units means a potential
loss of 0.2320 = $64/unit, whereas discounting the excess units represents a loss of only
320 300 = $20/unit. Therefore, Cabot will need fewer units if they are held over in
inventory.
9
(a) The optimum order quantity is:
Q * 2 DS / IC 2(1,250)(52)( 40) /(0.3)(56) 556 cases

and the reorder point quantity is:


ROP d LT z sd'
where
sd' sd LT 475 2.5 751
and z P0.80 = 0.84.

98

Now,

ROP (1,250)( 2.5) ( 0.84)( 751) 3,756 cases


Policy: When the amount of inventory on hand plus any quantities on order or in
transit falls below ROP, reorder an amount Q*.
(b) For the periodic review system, we first estimate the order review time:
T * Q * / d 556 / 1,250 0.44 weeks
The max level is:
M * d (T * LT ) z sd'
where sd' now is:
sd' sd T * LT 475 0.44 2.5 814 cases
Hence,
M * 1,250( 0.44 2.5) 0.84(814 ) 4,359 cases
Policy: Find the amount of stock on hand every 0.44 weeks and place a reorder for
the amount equal to the difference between the quantity on hand plus on order and the
max level (M*) of 4,359 cases.
(c) The total annual relevant cost for these policies is:
TC DS / Q ICQ / 2 ICzsd' kDs d' E ( z ) / Q

For the reorder point system:


TCQ = 1250(52)(40)/556 + .3(56)(556)/2
+ .3(56)(.84)(751) + 10(1250)(52)(751)(.1120)/556
= 4,676.26 + 4,670.40 + 10,598.11 + 98,332.37
= $118,277.14
For the periodic review system:
TCP = 1250(52)(40)/556 + .3(56)(556)/2
99

+ .3(56)(.84)(814) + 10(1250)(52)(814)(.1120)/556
= 4,676.26 + 4,670.40 + 11,487.17 + 106,581.29
= $127,415.12
(d) The actual service level achieved is given by:
SL 1

sd' E( z )
Q

For the reorder point system:


SLQ 1

751( 01120
.
)
1 015
.
556

or demand is met 85% of the time.


For the periodic review system:
SLP 1

814( 01120
.
)
1 016
.
556

or demand is met 84% of the time.


(e) This requires an iterative approach as follows:
Compute Q 2 DS / IC

Compute P 1 QIC / Dk , then z, then E (z)

Compute Q 2 D( S ksd' E( z ) ) / IC
Go back and stop when there is no change
in either P or Q
After the initial value of Q = 556.3, the process can be summarized in tabular form.

100

Step
1
2
3
4
5
6

Q
778.4
860.0
889.9
899.6
902.8
902.8

P
0.9856
0.9799
0.9778
0.9777
0.9767
0.9767

z
2.19
2.06
2.01
2.00
1.99
1.99

E(z)
0.0050
0.0072
0.0083
0.0085
0.0087
0.0087

Now, for P = 0.9767, z = 1.99


ROP = 1,250(2.5) + 1.99(751) = 4,620 cases
and the total relevant cost is:
TCQ DS/Q ICQ/2 ICzs'd kDs'd E(z) /Q
65,000(40) / 902.8 0.3(56)(902.8) / 2
0.3(56)(1.99)(751)
10(65,000)(751)(0.0087) / 902.8
$40,275

This is considerably less than the $118,277.14 for the preset P at 0.80.
If you solve this problem using INPOL, you will get a slightly different answer. That
is, Q* = 858. This simply is because z is carried to two significant digits rather than
the 4 significant digits used in the above calculations.
10
Refer to the solution of problem 10-9 for the general approach.

(a) Q* = 556.3 cases


and
2
ROP d LT z LT sd2 d 2 sLT

1,250( 2.5) 0.84 2.5 4752 1,250 2 0.52


3125
0.84(977.08 )
,
3,946 cases

(b) An approximation for T* = Q*/d, or


T* = 556/1,250 = 0.44 weeks
and approximating sd' as

101

2
sd' (T * LT )sd2 d 2 sLT

( 0.44 2.5)( 4752 ) 1,250 2 ( 0.5) 2


1,027 cases

So,
Max d (T * LT ) z sd'
1,250( 0.44 2.5) 0.84(1,027)
4,537 cases

(c) According to INPOL,


TC = 4,686 + 4,686 + 128,195 + 13,862 = $151,429
Q

TC = 4,686 + 4,686 + 134,751 + 14,571 = $158,694


P

(d) According to INPOL,


SL = 80.28%
Q

SL = 79.27%
P

(e) According to INPOL,


Q* = 930 cases, ROP = 5,128 cases,
TC = $49,532, SLQ = 99.22%
Q

T* = 0.76 weeks, MAX = 6,257 cases


TC = $52,894, SLP = 99.18%
P

11
(a) The production run quantity is:

Q *p

2 DS
p

IC
pd

2(100)( 250)( 250)


300

1,000 units
0.25(75)
300 100

(b) The production run cycle is:


Q *p / p 1,000 / 300 3.33 days

102

(c) The number of production runs is:


D / Q *p 100( 250 ) / 1,000 25 runs per year

12
(a) The order quantity is:
Q * 2 DS / IC 2( 2,000 )( 250 )(100
. ) / ( 0.30)(35) 309 valves

and the reorder point quantity is:

ROP d LT z sd LT
but sd 0 . Therefore,

ROP d LT ( 2,000 / 8)(1) 250 valves


(b) Boxes are set up that contain 309 valvesthe optimum order quantity. When an
order arrives from a supplier, 250 valves are set aside in a separate box and are
treated as the backup stock. The residual 309 250 = 59 valves are used on the
production line. When the 59 valves at the production line are used up, the backup
box containing 250 valves is brought to the production line and the empty box is sent
to the supplier for refilling. When the order arrives one hour later, there will be 0
valves remaining at the production line. Then, 250 valves are set aside and 59 are
sent to the production line. The cycle is then repeated.
This problem is similar to the KANBAN system. Lead times are very short so
that lead times are virtually certain. Demand is certain, since it is fixed by the
production schedule. Boxes or cards are used to assure movement of the most
economic quantity. KANBAN is essentially classic economic reorder point inventory
control under certainty.
13
(a) The economical quantity of cars to be called for at a time is found by the economic
order quantity formula:
Q * 2 DS / IC 2( 40 )(52 )(500 ) / ( 0.25)(90,000)(30) / 2,000 78.5, or 79 cars

(b) This is the reorder point quantity:

ROP d LT z sd LT
where z = 1.28 from Appendix A for an area under the curve equal to 0.90.
Therefore,

103

ROP 40(1) 128


. (333
. ) 1
44.3 cars, or 44.3(90,000 / 2,000)
= 1,994 tons of soda ash
14
(a) This is a reorder point design under conditions of uncertainty for both demand and
lead-time. We assume that the probability of an out of stock is given. Therefore, the
order quantity is:
Q * 2 DS / IC 2(50)(365)(50) / ( 0.30)( 45) 367.7 units

and

ROP d LT z sd'
where

z = 1.04 (see Appendix A) for the area under the curve equal to 0.85 and
2
sd' sd2 LT d 2 sLT
152 ( 7) (50 2 )( 2 2 ) 107.6 units

Therefore,

ROP 50( 7) 104


. (107.6) 4619
. units
(b) This is the periodic review system design under uncertainty. The complexity requires
us to make some approximations here. The time interval for review of the stock level
is:

T * Q * / d 367.7 / 50 7.35 days


The MAX level is:

MAX d (T * LT ) z sd'
where z = 1.04 and sd' is approximated as:
2
sd' (T * LT )( sd2 ) d 2 ( sLT
)

( 7.35 7)(152 ) 50 2 ( 2 2 )
115.0 units

104

Therefore,

MAX = 50(7.35 + 7) + 1.04(115.0)


= 837.1 units
(c) Since the service level is specified, the probability is not set at the optimum level.
Knowing the out-of-stock cost allows us to find the most appropriate service level.
Since this is an iterative process, we use INPOL to carry out the calculations.
The optimized service level yields a reorder point design of

Q* = 410 units and ROP = 571 units


and the total relevant cost drops from $12,642 in part a to $8,489. The demand in
stock in part a was 97.74%, and it now increases to 99.81%.
15
(a) Find the common review time:

T * 2(O s I ) / I Ci Di
2(100 0) /[(0.3 / 52)( 2.25 2,000 1.90 500) ]
2.5 weeks
Then,
M A* d A (T * LT ) z A sd A T * LT

where z = 1.28 for P = 0.90


A

M A* 2,000( 2.5 1.5) 1.28(100) 2.5 1.5 8,256 units


and

M B* 500( 2.5 15
. ) 0.842( 70) 2.5 15
. 2,118 units
where zB = 0.842 for P = 0.80.
The control system works as follows: the stock levels of both items are reviewed
every 2.5 weeks. The reorder size for A is the difference between the amount on hand
(includes on-order) and 8,256 units. The reorder size for B is the difference between
the amount on hand (includes on-order) and 2,118 units.
(b) The average amount in inventory is expected to be:

105

AIL d T * / 2 z sd T * LT
For A:
AILA 2,000( 2.5) / 2 128
. (100) 2.5 15
. 2,756 units
For B:
AILB 500( 2.5) / 2 0.842( 70) 2.5 15
. 743 units
(c) The service level is given by:
SL 1 sd' E( z ) / d T *

For A:
SL A 1 100 2.5 15
. ( 0.0475) / 2,000( 2.5) 0.998
For B:
SLB 1 70 2.5 15
. ( 01120
.
) / 500( 2.5) 0.987
(d) We set T* = 4 and cycle through the previous calculations. Thus, we have:
M A* 11,301 units

M B* 2,888 units

AILA = 4,301

AILB = 1,138

SLA = 0.999

SLB =0 .991

16
This problem is one of comparing the combined cost of transportation and in-transit
inventory. In tabular form, we have the following annual costs:

106

Cost type
Transportation

Formula
RD

In-transit
inventory

ICDT/365

Rail
Truck
6(40,000)(1.25)
11(40,000)(1.25)
= $300,000
= $550,000
0.25( 250)( 40,000)( 21) 0.25( 250)( 40,000)(7)
365
365
= $143,836
= $47,945
$597,945
$443,836

Total
Select rail.

17
The two transport options from the consolidation point are diagrammed in Figure 9-1.
Whether to choose one mode other the other depends more than transportation costs
alone. Because the transport modes differ in the time in transit, the cost of the money
tied up in the goods while in transit must be considered in the choice decision. This inICDt
transit inventory cost is estimated from
. The following design matrix can be
365
developed.

Cost type
Transportation
In-transit inventory
*

Method
RD
ICDt/365
Total

Air
$180,800
3,447*
$184,247

Ocean
$98,800
34,467
$133,267

ICDt/365 = 0.17(185)(20,000)(2)/365= 3,447

Ocean appears to be the lowest cost option even when a substantial in-transit inventory
cost is included. The ocean option assumes that the trucking cost to move the goods from
the consolidation point to the Port of Baltimore is included in the ocean carrier rate.
FIGURE 9-1 The
Consolidation
Operation for a
Hydraulic
Equipment
Manufacturer

Multiple sourcing points

Consolidation
point

Baltimore

20 days
2 days
Sao
Paolo

107

18
The demand pattern is definitely lumpy, since s = 327 > d = 169. To develop the minmax system of inventory control, we first find Q*. That is,
d

Q * 2 DS / IC 2(169 )(12 )(10) / 0.20( 0.96 0.048) 448.5 units

The ROP is
ROP d LT z sd' ED
where
z = 1.04 from Appendix A,
ED = 8 unitsthe average daily demand rate,
and
2
sd' sd2 LT d 2 sLT

327 2 ( 4) 169 2 ( 0.8 2 )


667.8 units

So,
ROP = 169(4) + 1.04(667.8) + 8
= 1,378.5 units
The max level is:
M* = ROP + Q* ED
= 1,378.5 + 448.5 8
= 1,819 units
19
(a) The basic relationship is:

IT I i n

108

We know that I = $5,000,000. If there are 10 warehouses, the amount of inventory in


a single one would be:
T

I1 I T / 10 5,000,000 / 3162
.
1,581139
,
The inventory in all 10 warehouses would be $1,581,13910 = $15,811,390.
(b) The inventory in a single warehouse would be:
I T 1,000,000 9 3,000,000
In each of 3 warehouses, we would have:
I 3,000,000 / 3 $1,732,051
and in all 3 warehouses, we would have $1,732,0513 = $5,196,152.
20
(a) The turnover ratio is the annual demand (throughput) divided by the average
inventory level. These ratios for each warehouse and for the total system are shown
in the table below.

Warehouse
21
24
20
13
2
11
4
1
23
9
18
12
15
14
6
7
22
8
17
16

Annual
warehouse
thruput
2,586,217
4,230,491
6,403,349
6,812,207
16,174,988
16,483,970
17,102,486
21,136,032
22,617,380
24,745,328
25,832,337
26,368,290
28,356,369
28,368,270
40,884,400
43,105,917
44,503,623
47,136,632
47,412,142
48,697,015

Average
inventory
level
504,355
796,669
1,009,402
1,241,921
2,196,364
1,991,016
2,085,246
2,217,790
3,001,390
2,641,138
3,599,421
2,719,330
4,166,288
3,473,799
5,293,539
6,542,079
2,580,183
5,722,640
5,412,573
5,449,058

Turnover
ratio
5.13
5.31 Avg. = 5.59
6.34
5.49
7.36
8.28
8.20
9.53
7.54
9.37
7.18
9.70
6.81
8.17
7.72
6.59
17.25
8.24
8.76
8.94
109

10
19
3
5

57,789,509
75,266,622
78,559,012
88,226,672
818,799,258

6,403,076
7,523,846
9,510,027
11,443,489
97,524,639

9.03
10.00
8.26
7.71
8.40

Avg. = 8.66

The overall turnover ratio is 8.40. Ranking the warehouses by throughput and
averaging turnover ratios for the top 3 and the bottom 3 warehouses shows that the
lowest volume warehouses have a lower turnover ratio (5.59) than the highest volume
warehouses (8.66). There are several reasons why this may be so:

The larger warehouses contain the higher-volume items such as the A items in the
line. These may carry less safety stock compared with the sales volume.
Conversely, the low-volume warehouses may have more dead stock in them.

There may be start-up (fixed) stock in the warehouses, needed to open them, that
becomes less dominant with greater throughput.

(b) A plot of the inventory-throughput data is shown in Figure 10-1. A linear regression
line is also shown fitted to the data. The equation for this line is:
Inventory = 200,168 + 0.1132Throughput
12

Average inventory level, $ (Millions)

10

8
Estimating line

0
0

20

40

60

80

100

Annual warehouse thruput, $ (Millions)

FIGURE 10-1 Plot of Inventory and Warehouse Thruput for California Fruit
Growers Association

110

(c) The total throughput for the three warehouses is:


Warehouse
1
12
23
Total

Throughput
$21,136,032
26,368,290
22,617,380
$70,121,702

Using this total volume and reading the inventory level from Fig. 10-1 or using the
regression equation, we have:
Inventory = 200,168 + .01132(70,121,702)
= $8,137,945
(d) Warehouse 5 has a throughput of $88,226,672. Splitting this throughput by 30% and
70%, we have:
0.3088,226,672 = 26,468,002
0.7088,226,672 = 61,758,670
88,226,672
Estimating the inventory for each of the new warehouses using the regression
equation, we have:
Inventory = 200,168 + 0.113226,468,002 = $3,196,346
and
Inventory = 200,168 + 0 .113261,758,670 = $7,191,249
for at total inventory in the two warehouses of $10,387,595
21
The order quantity for each item when there is no restriction on inventory investment is:

Q * 2 DS / IC
We first find the unrestricted order quantities.
Q A* 2(51,000)(10) / 0.25(17
. ) 1,527 units
QB* 2( 25,000 )(10) / 0.25(3.25) 784 units
QC* 2(9,000)(10 ) / 0.25( 2.50) 537 units

111

The total inventory investment for these items is:


IV C A ( Q A / 2) CB ( QB / 2) CC ( QC / 2)
. (1,527 / 2) 3.25( 784 / 2) 2.50(537 / 2)
175
.
$3,28138
Since the total investment limit is exceeded, we need to revise the order
quantities. For each product:
Q * 2 DS / [C( I )]

For product A:
Q A* 2(51,000)(10) / [175
. ( 0.25 )]

For product B:
QB* 2( 25,000 )(10) / [3.25( 0.25 )]

For product C:
QC* 2(9,000)(10 ) / [2.50( 0.25 )]

Now, the investment limit must be respected so that:

3,000 C A ( QA / 2) CB ( QB / 2) CC ( QC / 2)
Expanding we have:
3,000 175
.
2(51,000 )(10) / [175
. ( 0.25 )]
3.25 2( 25,000 )(10) / [3.25( 0.25 )]
2.50 2(9,000 )(10 ) / [2.50( 0.25 )]

We now need to find an value by trial and error that will satisfy this equation. We
can set up a table of trial values.

112

Investment in

A
1,262.44
1,240.48
1,229.92
1,221.67
1,219.63
1,129.16

0.03
0.04
0.045
0.049
0.05
0.10

B
1,204.53
1,183.58
1,173.51
1,165.63
1,163.69
1,077.36

C
633.87
622.84
617.54
613.40
612.37
566.95

Total
inventory
value, $
3,100.84
3,046.90
3,020.97
3,000.70
2,995.69
2,773.47

When the term I+ is the same for all products, as in this case, may be found
directly from Equation 10-30.
We can substitute the value for = 0.049 into the equation for Q* and solve.
Hence, we have:
Q A* 2(51000)(10) / [175
. ( 0.25 0.049)] 1,396 units
QB* 2( 25,000 )(10) / [3.25( 0.25 0.049)] 717 units
QC* 2(9,000)(10) / [2.50( 0.25 0.049 )] 491 units

Checking:
1.75(1,396)/2 + 3.25(717)/2 + 2.50(491)/2 = $3,000
22
We first check to see whether truck capacity will be exceeded. Since three items are to
be placed on the truck at the same time, the items are jointly ordered. The interval for
ordering follows Equation 10-22, or:
2( O S i )

T*

I Ci Di

2( 60 0 )
0.25[50(100 )(52) 30(300)(52) 25( 200)(52)]

120
0.022 years, or 1.144 weeks
0.25(988,000 )

Now, from

DT
i

wi Truck capacity

[100(70) + 300(60) + 200(25)][1.144] = 34,320 lb.

113

The truck capacity of 30,000 lb. has been exceeded, and the order quantity or the order
interval must be reduced. Given the revised Equation 10-30, the increment to add to I
can be found. That is,

2O
Truck capacity

Di wi

C D
i

2( 60 )
2

30,000

50(10)(52) 30(30)(52) 25( 20)(52)


[100(52 )( 70 ) 300(52)( 60) 200(52)(10)]
120
2

30,000

(988,000)
2,340,000

0.25

0.73895 0.25 0.48895

Revise T*, the order interval by:


T*

2( O S i )

( I ) Ci Di

2( 60 0)
( 0.25 0.48895)[50(100)(52 ) 30(300)(52 ) 25( 200)(52)]

120
0.01282 years, or 0.6667 weeks
0.73895(988,000)

Once again, we check that the truck capacity has not been exceeded.
[100(70) + 300(60) + 200(25)][0.66667] = 30,000 lb.
Therefore, place an order every 4.7, or approximately 5 days.
23
The average inventory for each item is given by:

Q*
AIL
z sd'
2
2 DS
. z@ 95% = 1.65 from the normal
IC
distribution in Appendix A. The results of these computations can be tabulated.

where sd' sd LT and Q* is found by Q *

114

0.25

sd'
Q*

AIL

A
7.75
188.38
106.98

B
15.49
238.28
144.70

C
19.36
421.23
242.56

D
11.62
361.98
200.16

E
27.11
565.14
327.30

Summing the AIL for each product gives a total inventory of 1,022 cases.
24
The peak quantity of an item to appear on a shelf can be approximated as the order
quantity plus safety stock, or

Q z sd' 250 boxes


where z@93% = 1.48 from Appendix A and sd' sd LT 19 1 19 boxes. The
economic order quantity is
Q*

2 DS

IC

2(123 52)(125
. )
255.42 boxes
019
. (129
. )

Checking to see if the shelf space limit will be exceeded by this order quantity
255.42 + 1.48(19) = 283.54 boxes
The quantity is greater than the 250 allowed. Subtracting the safety stock from the limit
gives 250 28 = 222 boxes. The order quantity should be limited to this amount.
25
The plot of average inventory to period facility throughput (shipments) gives an overall
indication of how the company is managing collectively its inventory for all stocked
items. We can see that the relationship is linear with a zero intercept. This suggests that
the company is establishing its inventory levels directly to the level of demand
(throughput). An inventory policy, such as stocking to a number of weeks of demand,
may be in effect.
Overall, the inventory policy seems to be well executed in that the regression line fits
the point for each warehouse quite well. The terminal with an inventory level of $6,000
seems to be an outlier and it should be investigated. If its low turnover ratio were
brought in line with the other terminals, an inventory reduction from $6,000 to $4,000 on
the average could be achieved.
The stock-to-demand inventory policy should be challenged. An appropriate
inventory policy should show some economies of scale, i.e., the inventory turnover ratio
should increase as terminal throughput increases. Whereas the current policy is of the
form I 0.012 D , a better policy would be I kD 0.7 , where D represents terminal
throughput and I is the average inventory level. The coefficient 0.012 for the current
policy is found as the ratio of 6,000/500,000 = 0.012 for the last data point in the plot.

115

The k value for the improved policy needs to be estimated. From the cluster of the lowest
throughput facilities, the average inventory level is approximately $2,000 with an
average throughput of about $180,000. Therefore, from
I kD 0.7
2,000 k (180,000) 0.7
2,000 k ( 4,771.894)
2,000
k
4,771.894
k 0.419
Reading values from the plot, the following table can be developed showing the
inventory reduction that might be expected from revised inventory policy. (Note: If the
inventory-throughput values cannot be adequately read from the plot, the values in the
following table may be provided to the students.)

Terminal
1
2
3
4
5
6
7
8
9
Totals

Actual
Inventory, $
2,000
1,950
2,000
2,050
3,900
6,000
4,500
4,300
5,500
32,200

Shipments, $
150,000
195,000
200,000
200,000
320,000
330,000
390,000
410,000
500,000
2,695,000

Estimated inventory, $
I 0.012 D
1,800
2,340
2,400
2,400
3,840
3,960
4,680
4,920
6,000
32,340

Revised inventory, $
I 0.419 D 0.7
1,760
2,115
2,152
2,152
2,991
3,056
3,435
3,558
4,088
25,307

Revising the inventory control policy has the potential of reducing inventory from the
32,340 25,307
x100 21.7% .
linear policy by
32,340
26
We can use the decision curves of Figure 9-23 in the text answer this question since it
applies to a fill rate of 95% and an = 0.7. First, determine K for an inventory
throughput curve for the item, which is

D1 (117 x12) 0.3

1.466
6
TO

Next,

116

tD10.7
12(117 x12) 0.3

0.90
ICK
0.20( 400)(1.466)

and with z 1.96 from Appendix A


Y

zs LT
1.96(15) 2

0.18
a
KD
(1.466)(117 x12) 0.7

The demand ratio r is 42/177 = 0.36. The intersection of r and X lies below the curve Y
(use curve Y = 0.25), so do not cross fill.
27
Regular stock
For two warehouses, estimate the regular stock for the three products.

Product A

Product B

2dS
Q
RS IC
2
2
2(3,000)( 25)
0.02(15)
RS A1
354 units
2
2(5,000)( 25)
0.02(15)
RS A2
457 units
2

RS B1
RS B 2

Product C

RS C1
RS C 2

2(8,000)( 25)
0.02(30)
408 units
2
2(9,500)( 25)
0.02(30)
445 units
2

2(12,500)( 25)
0.02( 25)
559 units
2
2(15,000)( 25)
0.02( 25)
612 units
2

Regular system inventory for two warehouses is RS2W = 354 + 457 + 408 + 445 + 559 +
612 = 2,835.

117

Regular stock for a central warehouse

RS A
RS B
RS C

2(8,000)( 25)
0.02(15)
577 units
2
2(17,500)( 25)
0.02(30)
604 units
2
2( 27,500)( 25)
0.02( 25)
829 units
2

Total central warehouse regular stock is RS1W =577 + 604 + 828 = 2,009 units.
Safety Stock
Product A
SS zsd LT
SS A1 1.65(500) 0.75 714 units
SS A2 1.65(700) 0.75 1,000 units

where z@0.95 = 1.65 from Appendix A


Product B
SS B1 1.65( 250) 0.75 357 units
SS B 2 1.65(335) 0.75 479 units
Product C
SS zsd LT
SS C1 1.65(3,500) 0.75 5,001 units
SS C 2 1.65( 2,500) 0.75 3,572 units

System safety stock is SS2W = 714 + 1,000 + 357 + 479 + 5,001 + 3,572 = 11,123 units
For each product, the estimated standard deviation of demand on the central warehouse is
s A s12 s22 500 2 700 2 860 units
s B 250 2 3352 418 units
s B 3,500 2 2,500 2 4,301 units
The safety stock is

118

SS zs LT
SS A 1.65(860) .75 1,229 units
SS B 1.65( 418) .75 597 units
SS C 1.65( 4,301) .75 6,146 units
Total safety stock in the central warehouse SS1W = 1,229 + 597 + 6,146 = 7,972 units.
Total inventory with two warehouses RS2W + SS2W = 2,835 + 11,123 = 13,958 units and
for a central warehouse RS1W + SS1W = 2,009 + 7,972 = 9,981 units. Centralizing
inventories reduces them by 13,958 9,981 = 3,977 units.
28
The solution to this multi-echelon inventory control problem is approached by using the
base-stock control system method. The idea is that inventory at any echelon is to plan its
inventory position plus the inventory from all downstream echelons.
First, compute the average inventory levels for each customer. This requires finding
Q and the safety stock. Q is found from the EOQ formula.

For customer 1

Q1

2( 425 x12)(50)
270 units
0.2(35)

AIL1

Q1
270
zsd1 LT1
1.65(65) 0.5 211 units
2
2

where z@0.95 =1.65 from Appendix A


For customer 2
Q2

2(333x12)(50)
239 units
0.2(35)

AIL2

Q2
239
zsd 2 LT2
1.65(52) 0.5 180 units
2
2

For customer 3
Q3

2( 276 x12)(50)
218 units
0.2(35)

AIL3

Q3
218
zs d 3 LT3
1.65( 43) 0.5 159 units
2
2

119

Total customer echelon inventory is AILC = 211 + 180 + 159 = 550 units
For the distributors echelon
QD 2,000 units as given
AILD

QD
2,000
zsd D LTD
1.28(94) 1.0 1,120 units
2
2

where z@0.90 =1.28 from Appendix A


The expected inventory that the distributor will hold is the distributor echelon inventory
less the combined inventory for the customers, or 1,120 550 = 570 units.

120

COMPLETE HARDWARE SUPPLY, INC.


Teaching Note
Strategy
Complete Hardware Supply is an exercise involving the control of inventoried items
collectively. Data for a random sample of 30 items from the company's total of 500 items
held in inventory are given. The objective is to manage the total dollar value allowed to
be held as inventory. Several alternatives can be considered for changing inventory
levels, some of which require an investment other than in inventory.
The number of items that must be analyzed and the multiple scenarios that are to be
examined can be computationally time consuming. It is strongly suggested that students
use the INPOL module within LOGWARE to aid analysis. The current database has
been prepared and is available in the LOGWARE software.
The Base Case
We begin with the current data optimized as a reorder point design. The optimum order
quantities and associated inventory levels are found. The base case costs are shown as
follows:

Fixed order quantity policy


Purchase cost
Transport costa
Carrying cost
Order processing cost
Out-of-stock cost
Safety stock cost
Total cost
Total investment

$556,912
0
4,425
4,425
0
2,529
$568,291
$27,801

aIncluded in the purchase cost

We note that optimizing the current design shows that investment of $27,801 exceeds
the allowed investment level of $18,000. Ways need to be explored to reduce this.
Transmit Orders More Rapidly
Instead of mailing orders to vendors, Tim O'Hare can buy a facsimile machine and
transmit orders electronically. This scenario can be tested by reducing the lead times in
the base case by 2 days, or (2/5) = 0.40 weeks and increasing order processing costs by
$2, and then optimizing again. INPOL shows that there will be a slight increase in
operating costs from $568,291 to $568,640, an incremental increase of $349. Projecting
this to all 500 items, we have 349(500/30) = $5,817. Since both operating cost and
inventory investment level increase, there is no economic incentive to implement this
change.
Faster Transportation
Suggesting that vendors who are located some distance (>600 miles) from the warehouse
use premium transportation is a possible way of reducing lead times, and therefore safety

121

stock levels. Of course, the increase in transportation cost for those affected vendors is
likely to lead to a price increase to cover these costs. This scenario is tested by reducing
the lead-time in weeks to 2.2 for those vendors over 600 miles from the warehouse. For
these same vendors, a 5% price increase is made.
Compared with the base case, there is little change in the inventory investment
($27,801 vs. $27,746); however, operating costs increase. The total costs now are
$585,490 compared with the base case of $568,291, an increase of $27,199. The major
portion ($17,159) of this comes from the increase in price. We conclude that this is not a
good option for Tim.
Reduce Forecast Error
Reducing the forecast error involves reducing the standard deviation of the forecast error.
Testing this option requires taking 70% of the base-case forecast error standard
deviations and optimizing the design once again.
These changes have a positive impact on operating costs and inventory investment.
Operating cost now is $567,529 and inventory investment is $24,739. This is a saving in
operating costs of $762 per year. For all 500, we can project the savings to be
762(500/30) = $12,700. Based on a simple return on investment, we have:

ROI

12,700
0.25, or 25% / year
50,000

This would appear to be attractive since carrying costs are 25% per year and the
company's return on investment probably makes up about 80% of this value.
Reduce Customer Service
At this point, we have only accepted the idea of reducing the forecast error. However,
inventory investment remains too high. We can now try to reduce it by reducing the
service levels. This is tested by dropping the service index from its current 0.98 level to
a level where inventory investment approximates $18,000. This is done, assuming the
forecast software will be purchased and the forecast error reduced by 30%. By trial and
error, the service index is found to be 0.54, which gives an investment level of $18,028.
The revised service level compared with the base case is summarized below for the 30
items.

122

Item
1
2
3
4
5
6
7
8
9
10
11
12
13
14
15

Base
case
99.88%
99.92
99.96
99.98
99.98
99.96
99.97
99.96
99.92
99.98
99.99
99.99
99.92
99.98
99.96

Revised
96.26%
98.02
98.54
99.15
99.45
98.60
98.84
98.61
97.29
99.26
99.70
99.43
97.30
99.14
98.84

Item
16
17
18
19
20
21
22
23
24
25
26
27
28
29
30

Base
case
99.98%
99.90
99.95
99.89
99.97
99.69
99.97
99.97
99.96
99.92
99.97
99.93
99.89
99.97
99.91

Revised
99.56%
97.57
97.81
95.96
98.15
89.53
98.96
98.96
97.58
99.33
96.68
97.45
98.78
96.92
96.78

Notice how little the service level changes, even with a substantial reduction in the
service index.
Conclusions
Tim can make a good economic argument for purchasing software that will reduce the
forecast error. The only questions here are whether the software can truly produce at
least the error reduction noted and whether a 25% return on investment is adequate for
the risks involved.
Arguing to accept a service reduction in order to lower the investment level is a little
less obvious since we do not know the effect that service levels have on sales. However,
Tim may point out that the service levels need to be changed so little that it is unlikely
that customers will detect the change. He might also raise the question as to whether
customer service levels were too high initially, and suggest that customers be surveyed as
to the service levels that they do need.

123

AMERICAN LIGHTING PRODUCTS


Teaching Note
Strategy
American Lighting Products is a manufacturer of fluorescent lamps in various sizes for
industrial and consumer use. As frequently happens in business, top management has
requested that inventories be reduced across the board, but it does not want to sacrifice
customer service. Sue Smith and Bryan White have been asked to eliminate 20 percent
of the finished goods inventory. Their plan is to reduce the number of stocking locations
and, thereby, eliminate the amount of inventory needed. Of course, they must recognize
that with fewer stocking points, transportation costs are likely to increase and customer
delivery times may increase as well. On the other hand, facility fixed cost may be
reduced.
The purpose of this case is to allow students to examine inventory policy and
planning through aggregate inventory management procedures. They also can see the
connection between location and inventory levels.
Answer to Questions
(1) Evaluate the companys current inventory management procedures.

The companys procedures for controlling inventory levels are at the heart of whether
inventory reductions are likely to be achieved through inventory consolidation. The
company appears to be using some form of reorder point control for the entire system
inventory, but it is modified by the need to produce in production lot sizes. It is not clear
how the reorder point is established. If it is based on economic order quantity principles,
then the effect of the principles becomes distorted by the need to produce to a lot size that
is different from the economic order quantity. Therefore, average inventory levels in a
warehouse will not be related to the square root of the warehouses throughput (demand),
i.e., throughput raised to the 0.5 power.5 Rather, the throughput will be raised to a higher
exponent between 0.5 and 1.0.
The above ideas can be verified by plotting the data given in Table 1 of the case and
then fitting a curve of the form I TP . Note: The curve can be found from standard
linear regression techniques when the equation is converted to a linear form through a
logarithmic transformation, i.e., lnI = ln + lnTP. The results are shown in Figure 1.
The inventory curve is I 2.99TP 0.816 with r = 0.86, where I and TP are in lamps. The
projected inventory reduction can be calculated by using this formula.
From the plot of the inventory data, we can see that there is substantial variation
about the fitted inventory curve. There is not a consistent turnover ratio between the
warehouses. This probably results from the centralized control policy. On the other
hand, improved control may be achieved by using a pull procedure at each MDC. The
data available in the case do not let us explore this issue.

Based on the economic order quantity formula, the average inventory level (AIL) for an item held in
inventory can be estimated as AIL Q / 2 2 DS / IC / 2 . Collecting all constants into K, we have
AIL=K(D)0.5, where D is demand, or throughput.

124

FIGURE 1 Plot of
MDC average
inventory vs. annual
throughput.

(2) Should establishing the LOC be pursued?


One of the ideas proposed in the case is to consolidate all Consumer product line items
into one large order center (LOC). Evaluating the impact of the LOC on inventory
reduction requires that an assumption be made as to how much demand and associated
inventory of the total belongs to Consumer products. Table 2 of the case gives the order
and back order breakdown by sales channel. Using these data, total consumer demand is
312,211 line items, or 33.4% of the total line items. The assumption is that the same
percentage applies to total demand. Hence, Consumer demand is 33.4%169,023,000 =
56,453,682 lamps. From the inventory-throughput curve, we can estimate the amount of
inventory needed at the single LOC. That is, I = 2.997(56,453,682)0.816 = 6,339,684
lamps. If Consumer products account for 33.4% of total inventory, then there are
33.4%23,093,500 = 7,713,229 lamps in Consumer inventory. The reduction that can be
projected is 7,713,229 6,339,684 = 1,373,545 lamps for a reduction of
Reduction

1,373,545
100 17.8%
7,713,229

in Consumer inventory levels, but only a 6% reduction in overall inventory levels. The
20% reduction goal is not achieved. Other alternatives need to be explored.
(3) Does reducing the number of stocking locations have the potential for reducing
system inventories by 20%? Is there enough information available to make a good
inventory reduction decision?
The second alternative proposed in the case is to reduce the number of MDCs from eight
to a smaller number. In order to evaluate this proposal, it needs to be determined which
MDCs will be consolidated and the associated total demand flowing through the
consolidated facilities. The inventory-throughput relationship can then be used to
estimate the resulting inventory levels. For example, if the Seattle and Los Angeles
MDCs are combined, the consolidated demand would be 4,922,000 + 21,470,000 =
26,392,000 lamps. The combined inventory is projected to be I = 2.997(26,392,000)0.816 =

125

3,408,852 lamps, compared with the inventory for the two locations of 4,626,333, as
shown in Table 1. This yields a 26.3% reduction from current levels.
Table 1 shows other possible MDC consolidations and the resulting inventory
reductions that can be projected.
TABLE 1 Inventory Reduction for Selected MDC Combinations, in Lamps
Combined
Combined
Inventory
MDC combination
demand
inventory
reduction
Seattle/Los Angeles
26,392,000
3,408,852
1,217,481
Kansas City/Dallas
29,194,000
3,701,403
50,181
Chicago/Ravenna
49,174,000
5,664,257
-557,590
Atlanta/Dallas
39,314,000
4,718,862
1,224,721
Kansas City/Chicago
39,271,000
4,714,650
-933,900
Ravenna/Hagerstown
64,046,000
7,027,231
1,715,607
K City/Dallas/Chicago
52,515,000
5,976,377
-36,377
Ravenna/Htown/Chicago
87,367,000
7,508,054
3,423,196
Atlanta/Dallas/K City
55,264,000
5,242,351
2,293,566

From the MDC combinations in Table 1, proximity to each other is a primary


consideration in order to not increase transportation costs or jeopardize delivery service
any more than necessary. Several options can be identified that yield a 20% inventory
reduction. These are:

Option
1

MDC combinations
LA/Seattle
Ravenna/Htown/Chicago
Total reduction

Inventory
reduction,
lamps
1,217,481
3,423,196
4,640,677

Total
inventory
reduction

LA/Seattle
Kansas City/Hagerstown
Ravenna/Hagerstown
Total reduction

1,217,481
1,224,721
1,715,602
4,157,804

18.0%

LA/Seattle
Ravenna/Hagerstown
Atlanta/Dallas/K City
Total reduction

1,217,481
1,715,602
2,293,566
5,226,649

22.6%

20.1%

Options 1 and 3 achieve the 20% reduction goal, although other MDC combinations not
evaluated may also do so. The maximum reduction would be achieved with one MDC.
The total inventory would be I = 2.997(169,023,000)0.816 = 15,512,812 lamps, for a system
reduction of 32.8%. However, we must recognize that as the number of warehouses is
decreased, outbound transportation costs will increase. Inbound transportation costs to
the combined MDC will remain about the same, since replenishment shipments are
126

already in truckload quantities. Some difference in cost will result from differences in
the length of the hauls to the warehouses. On the other hand, outbound costs may
substantially increase, since the combined MDC locations are likely to be more removed
from customers then they are at present. Outbound transportation rates will be higher, as
they are likely to be for shipments of less-than-truckload quantities. If the sum of the
inbound and outbound transportation cost increases is greater than the inventory carrying
cost reduction, then the decision to reduce inventories must be questioned.
Calculating all transportation cost changes is not possible, since the case study does
not provide sufficient data on outbound transportation rates. However, they should be
determined before and after consolidation to assess the tradeoff between inventory
reduction and transportation costs increases. On the other hand, inbound transportation
costs can be found, as shown below for option 1, where the consolidation points are Los
Angeles and Hagerstown.
Annual
demand,
Location
lamps
Seattle
4,922,000
Los Angeles
21,470,000
Ravenna
25,853,000
Hagerstown
38,193,000
Chicago
23,321,000
Total
113,759,000
a
(4,922,000/35,000)1800 = 253,131
TL rate,
$/TL
1800
1800
250
475
350

Transport
cost, $
253,131a
1,104,171
184,664
518,334
233,210
2,293,510

Combined
annual
demand, lamps

Transport
cost, $

26,392,000

1,357,302

87,367,000

1,185,695

113,759,000

2,542,997

There will be a net increase in inbound transportation costs of $2,542,997 2,293,510 =


$249,487 for option 1.
In addition, the annual fixed costs for the MDCs will be less, since the total space
needed in the consolidated facilities should be less than that for the existing facilities.
Again, the case study does not estimate the fixed costs for existing or potential locations.
We do know that taking them into account would favor consolidation.
In summary, the costs associated with option 1, that just meets the 20% inventory
reduction goal, would be:
Cost type
Inventory carrying cost reduction
Warehouse cost
Warehouse fixed cost
Outbound transportation cost
Inbound transportation cost

Cost savings
0.200.8824,640,677 = $818,615
0.104,640,677 = $464,068
Unknown, but may be included in warehouse cost
Unknowndata not given
($249,487)

Although Sue and Bryan could report a substantial savings in inventory related costs,
they should be encouraged to include fixed costs and transportation costs so as to report
the true benefits of the inventory reduction plan.
(4) How might customer service be affected by the proposed inventory reduction?

127

The general effect of inventory consolidation is to reduce the number of stocking points
and make them more remote from customers. That is, the delivery distance will be
increased if inventory consolidation is implemented. Therefore, delivery customer
service may be jeopardized and must be considered before deciding to consolidate
inventories.
From Table 3 of the case, it can be seen that customer lead times remain constant for
a variety of locations with the exception of Kansas City. Since consolidation points will
be selected among the existing locations, outbound lead times will remain unaffected.
Customer service due to location should be constant, at least for a moderate degree of
consolidation.
Customer service due to stock availability will be affected if safety stock levels are
reduced after consolidation. Although the inventory-throughput relationship projects
adequate safety stock to maintain the current first-time delivery levels, it does not
account for any increase in lead times that may occur between the current system of
MDCs and the consolidated ones. By comparing the weighted inbound lead times for the
existing distribution system and option 1, as shown in Table 2, the average inbound leadtime is slightly reduced through consolidation. Lead-time variability is usually related to
average lead-time. This should have a favorable affect on inventory levels since
uncertainty is reduced. First-time deliveries should not be adversely affected by
consolidation, according to option 1.
TABLE 2

A Comparison of Inbound Lead Times for the Existing Distribution


System and a Consolidated Distribution System (Option 1)
(a) Current Distribution System
Inbound
Weighted
lead time,
lead time,
Master Distribution Center
Shipments
days
days
Atlanta
26,070,000
2
0.308
Chicago
23,321,000
1
0.138
Dallas
13,244,000
3
0.235
Hagerstown
38,193,000
1
0.226
Kansas City
15,950,000
2
0.094
Los Angeles
21,470,000
5
0.635
Ravenna
25,853,000
1
0.153
6
0.175
Seattle
4,922,000
Total
169,023,000
1.964

128

(b) Consolidation Option 1

Master Distribution Centera


Atlanta
Dallas
Htown/Ravenna/Chicago
Kansas City
Los Angeles/Seattle
Total
a

Shipments
26,070,000
13,244,000
87,367,000
15,950,000
26,392,000
169,023,000

Inbound
lead time,
days
2
3
1
2
5

Weighted
lead time,
days
0.308
0.235
0.517
0.094
0.781
1.935

Consolidation is assumed to take place at the MDC with the largest number of current shipments.

129

AMERICAN RED CROSS: BLOOD SERVICES


Teaching Note
Strategy
The American Red Cross Blood Services has a mission to provide the highest quality
blood components at the lowest possible cost. High quality blood products are provided
to regional hospitals, but managing the inventory to meet demand as it occurs is a
difficult problem. Blood is considered a precious product, especially by those who give
it voluntarily. So, managing this perishable product carefully is a foremost concern.
Blood is a vital product to those in need of it for emergencies and a precious product
to those requiring it for elective surgery and other treatments. The goal is to always have
what is needed but never so much that this perishable product has to outdated. Managing
the blood inventory is quite difficult because (1) forecasting demand is not particularly
accurate, (2) the planning horizon for collections can be up to a year long with uncertain
yields, (3) the life of blood products ranges from 42 days to as short as 5 days, (4) once
scheduled, blood donors are never turned away except for medical reasons, and (5) there
is a limited opportunity to sell blood outside of the local region if too much is on hand.
Overall, this situation has many characteristics of a supply driven inventory
management problem, which requires inventory management techniques different from
those for typical consumer products.
The intended purpose of this case study is for students to examine an inventory
situation where there is limited control over the amount of the product flowing into
inventory. This supply-driven inventory situation is likely to be quite different from that
discussed on the introductory level. Students are encouraged to consider the various
elements that affect inventory levels of individual products and how they interact. These
elements are (1) demand forecasting, (2) collections, (3) decision rules for creating blood
derivatives, (4) product prices, and (5) inventory policy. It is expected that students will
be able to make general suggestions for improvement.
Questions
(1) Describe the inventory management problem facing blood services at the American
Red Cross.

One of the major problems facing the American Red Cross (ARC) is that the availability
of blood is supply-driven, meaning that quantities of blood received for processing to
meet demand in the short term are unknown, yet they must be placed in inventory if
demand is less than the collected quantities. Blood availability is a function of number of
factors that cannot be well-controlled by the regional blood center in the short run,
causing wide variability in supply. The usage of blood at hospital blood banks, which
creates the demand on ARCs blood inventories, is also uncertain and varies from day to
day and between hospital facilities.
The yield of blood at the point of collection is random and does not necessarily give
the product mix needed to meet demand. Different blood types can only be known by a
probability distribution as to the percentage of the blood types that exist in the general
population. In the short term, the demand for blood types may differ from the collected

130

quantities, resulting in a potential for under- and over-stocking, since blood is drawn
from all qualified donors as they arrive at collection sites.
Forecasting demand for blood products will likely be reasonably accurate for a base
load. Surgery loads on hospitals are scheduled in advance so that blood needs will be
known with a fair degree of certainty, although each operation will not typically use the
full amount of blood allocated to it. However, emergency blood needs are not well
predicted, and they can cause spikes in demand and unplanned draws on inventory. A
problem is establishing how much accuracy is needed for good inventory management.
Inventory policy for managing inventory levels is a mixed strategy of product pricing,
derivative product selection for processing at the time of collection, conversion to other
products later in the product life cycle, product sell off, emergency supply (call for
blood), discount pricing, and stocking rules for hospitals. Although there are many
avenues to controlling inventory levels, shortages and outdating cannot always be
avoided. It is not clear that these procedures lead to an optimal control of inventory
levels.
Competition from local independent blood banks that sell selected blood products at
low prices makes it difficult for ARC to cover costs. ARC provides a wider range of
products, but it has difficulty-differentiating price among derivative products so that it
might compete effectively. Given pressures for hospitals to increase efficiency, they will
shop around for the lowest-priced blood products. ARC is having difficulty maintaining
its position as the dominant supplier of blood products in the region, which results in the
greater uncertainty in managing inventory levels.
In summary, blood is a precious product given by volunteers for the benefit of others.
Donors have the right to expect that their contribution will be handled responsibly. To
ARC, this means managing the blood supply so that recipients receive a high-quality
product at the lowest possible price. To achieve this goal, ARC manages the blood
supply through four inter-connected elements: (1) estimating the blood product needs
over time, (2) planning the collection of whole blood, (3) deciding which derivative
products and their amounts should be created from whole blood, and (4) controlling the
inventory levels to avoid outdating. The volunteer nature of the blood giving and donor
attitudes surrounding it, long planning lead times and the associated uncertainties, rising
competition among some products from local blood banks, and the uncertainties of blood
needs all make blood supply management a unique inventory management problem.
(2) Evaluate the current inventory management practices in light of ARCs mission.
Performance of blood management can be evaluated on two levels: customer service and
cost. Tables 8 and 9 of the case show that in March standards were not quite met overall.
Within specific product types, there was up to an 8 percent deficit. Both order fill rate
and item fill rate were less than 100 percent for most products. There would seem to be
some room for improvement, especially in managing the variation among product types.
From a cost standpoint, it is not known how efficiently the blood supply is managed
since no costs are reported. In addition, the revenue that the blood products generate is
not known. We would like to know how prices of the various products are set so that
revenues might be maximized, considering competition among some of the product line.
We do expect that demand is price elastic, since hospitals do shop around for blood

131

products that are available from local, commercial, and community blood banks. On the
other hand, ARC is the sole regional supplier of certain products such as platelets.
Setting product fill-rate standards at various levels can influence costs. We do not
know this effect.
Setting inventory levels by a number of days of inventory rule of thumb is simple
but not as effective as planning inventory levels based on the uncertainties that occur in
demand forecasts and supply lead times. The number-of-days-of-inventory rule does
tend to lead to too much inventory or to too many out-of-stock situations.
The plan for evaluation, if enough data were available, would be to establish a base
case of cost and service. This, then, would provide a basis for evaluating the effect of
change in the supply procedures.
(3) Can you suggest any changes in ARCs inventory planning and control practices that
might lead to cost reduction or service improvement?
Suggestions for improvement in blood supply management stem from a basic
understanding of the nature of the demand-supply relationship. When supply is uncertain
and all supply must be taken that is available, there is the possibility that significant
excess inventory will occur. The goal is to manage the demand in the short run to
reduce inventory levels when overstocking occurs, rather than focusing on managing
supply. Several approaches for doing this are:
Aggressively price selected products that are in excess supply and are nearing their
expiration dates, e.g. run a sale or offer price discounts.
Sell off excess supply to secondary demand sources or other regions of the ARC.
Temporarily adjust return rules for hospitals.
Bring demand more in line with supply by converting products into derivative ones
that have excess demand, e.g., reprocess whole blood into plasma.
Encourage hospitals to buy certain products in excess supply for a more favorable
status in buying other products that are in short supply, such as phersis platelets and
rare whole blood types.
Try to create excess demand for all products, especially those items that are
available from local blood banks, through promotion of ARCs distinct advantages,
such as quality, high service levels, and a wide range of blood derivative products.
Offer two-for-one sales, such that if a hospital buys one blood product, it may
receive another at a favorable price.
Pool the risk of uncertain demand by maintaining a central inventory for all
hospitals, or managing the inventories at all hospitals, as well at ARC, collectively.
Provide quick deliveries or transfers among inventory locations.
ARC should attempt to be the premier provider of blood products and leverage the
advantage. This will allow it to maintain a degree of control over the demand for blood.
Effectively controlling demand in turn allows it to control its costs and avoid product
outdating.

132

(4) Is pricing policy an appropriate mechanism to control inventory levels? If so, how
should price be determined?
From the previous discussion, it can be seen that price plays a role in controlling demand.
Since there appears a relationship between demand and price for some products,
especially among those products offered by local blood banks that compete with ARC
blood products, price may be an effective weapon to meet competition. Rather than
setting price based on the cost of production, ARC might consider raising the price on
products for which it is the sole provider, such as platelets, and then meeting the price of
competitors on whole blood. Although ARC strives to be a nonprofit organization, the
increased volume that an effective pricing strategy promotes would allow more of the
fixed costs to be covered. This may lead to lower overall average prices for ARCs
products.
Blood could also be priced as a function of its freshness at two or more levels.
Although blood that has been donated within 42 days legally can be utilized, the quality
of blood does not remain the same for the entire 42-day period. A chemical compound
found in blood, called 2,3-DPG, decreases with the age of the stored blood, and is
believed to be important in oxygen delivery. For this reason, certain procedures such as
heart transplants and neonatal procedures require that blood be fresh, usually donated
within 10 days or less. Thus, a simple pricing policy could be to charge a higher price for
blood that is less than 10 days old, and a lower price for blood that is between 10 and 42
days old. Price differences here are based on product quality.

133

CHAPTER 10
PURCHASING AND SUPPLY SCHEDULING DECISIONS
1
(a) The following requirements schedules will lead to the proper timing and quantities for
the purchase orders.

Desk style A
Sales forecast
Receipts
Qty on hand 0
Releases to prod.

1
150
200
50
300

2
150
300
200

3
200

1
60

2
60
100
60

3
60

2
120
100
80
100

3
100
100
80

0
300

Week
4
200
300
100
300

5
150
300
250

6
200

7
200
300
150

8
150

6
100
100
40
100

7
80
100
60

8
60

6
60
100
60

7
60

8
80
100
60

50
300

Desk style B
Sales forecast
Receipts
Qty on hand 80
Releases to prod.

20
100

0
100

Week
4
5
80
80
100 100
20
40
100 100

Desk style C
Sales forecast
Receipts
Qty on hand 200
Releases to prod.

1
100
100
100

Week
4
5
80
80
100
0
20
100 100

0
100

Summing the releases for these three desk release schedules gives a production
requirements schedule for desks in general and sheets of plywood in particular. That
is,
Week
1
2
3
4
5
6
7
8
Desk requirements
500 100
400 500 200
400 100
1500 300 1200 1500 600 1200 300 0
Plywood sheetsa
a
Desk requirements times 3

Now, find the purchase order releases for the plywood sheets.

Sales forecast
Receipts
Qty on hand 2400
Releases to prod.

1
2
1500 300
600
900 1200
1000 1000

3
1200
1000
1000
1000

Week
4
5
6
7
1500 600 1200 300
1000 1000 1000
500 900 700 400
1000

8
0
400

Therefore, purchase orders should be placed in weeks 1, 2, 3, and 4 for 1000 sheets
each.

134

(b) Using Equation 10-2 in the text, the probability of not having the plywood sheets at
the time needed would be
Pr 1

Pc
5
1
0.02
01
. 5
Cc Pc

From Appendix A, z@1-.02 = 2.05. Therefore, the lead-time should be


. days
T * LT z sLT 14 2.05( 2) 181
Another week should be added to the current lead-time of 2 weeks.
2
(a) Using Equation 10-2, the probability of not having the item when needed for
production is:
Pr

Pc
150

0.9999
C c Pc (0.2 35 / 365) 150

The time to place an order ahead of need is:


T * LT z s LT 14 3.6( 4) 28 days
where z@0.9999 = 3.6 from Appendix A.
(b) Use part period cost balancing. The unit carrying cost is (0.2/52)35 = 0.134. Then,
(Q=250) Week 4
0.134[500 + 200]/2 = 46.9
(Q=1350) Weeks 4 + 5
0.134[(1350 + 1050)/2 + (1050 + 200)/2] = 244.6
The carrying cost closest to the order cost of $50 is Q = 250. Order this amount.
3
Using the requirements planning procedure, we can develop a schedule of material flows
through the network over the next 10 weeks.
Whse 1
Requirements
Schd receipts
On-hand qty 1700
Releases

1
2
3
4
5
6
7
1200 1200 1200 1200 1200 1200 1200
7500
500 6800 5600 4400 3200 2000 800
7500
7500

8
9
10
1200 1200 1200
7500
7100 5900 4700

135

Whse 2
Requirements
Schd receipts
On-hand qty 3300
Releases

1
2
3
2300 2300 2300
7500
1000 6200 3900
7500

Whse 3
Requirements
Schd receipts
On-hand qty 3400
Releases

1
2
3
4
2700 2700 2700 2700
7500
700 5500 2800 100
7500
7500

Regnl whse A
Requirements
Schd receipts
On-hand qty
52300
Releases to
plant

1
2
22500

3
0

4
5
6
2300 2300 2300
7500
1600 6800 4500
7500
5
6
2700 2700
7500
4900 2200
7500

4
5
0 15000

7
8
9
10
2300 2300 2300 2300
7500
2200 7400 5100 2800
7500
7
8
9
2700 2700 2700
7500
7000 4300 1600
7500

6
7
8
0 7500 15000 0
15000

29800 29800 29800 14800 14800 7300


15000
4
5
4100 4100
7500
4300 200
7500

10
2700
7500
6400

9
7500
15000

7300 7300

10
0

1300

1300

15000

Whse 4
Requirements
Schd receipts
On-hand qty 5700
Releases

1
2
3
4100 4100 4100
7500
1600 5000 900
7500
7500

Whse 5
Requirements
Schd receipts
On-hand qty 2300
Releases

1
2
3
4
1700 1700 1700 1700
7500
600 6400 4700 3000
7500

Whse 6
Requirements
Schd receipts
On-hand qty 1200
Releases

1
2
3
4
5
6
7
8
9
10
900 900 900 900 900 900 900 900 900 900
7500
7500
300 6900 6000 5100 4200 3300 2400 1500 600 7200
7500
7500

Regnl whse B
Requirements
Schd receipts
On-hand qty
31700
Releases to
plant

1
2
3
4
22500
0 7500
15000

Plant
Requirements
Schd receipts
On-hand qty 0
Releases-matls

5
6
7
0 15000 7500
15000

15000
2

3
0 0

0
0 0
20000 20000

7
8
4100 4100
7500
7000 2900
7500

9
10
4100 4100
7500
6300 2200

5
6
7
8
9
10
1700 1700 1700 1700 1700 1700
7500
1300 7100 5400 3700 2000 300
7500

9200 24200 16700 16700

6
4100
7500
3600
7500

4
15000
20000
5000
20000

1700

8
9
10
0 7500 7500
0
15000

9200 1700 9200

9200 9200

15000

5
6
7
15000
0 30000
20000
20000
10000 10000
0

8
0

9
0

10
0

Summing the releases to the plant shows that the plant should place into production
15,000 cases in weeks 4 and 5 and 30,000 cases in week 7. Orders for materials should
be placed in weeks 1, 2, and 4 in an order size to make 20,000 units.
Because demand is shown to be constant, the average inventory must be one-half the
order quantity. For the six field warehouses and a shipping quantity of 7500, the average
long run inventory would be (7500/2)6 = 22,500 cases. For the regional warehouses,
136

the average inventory would be (15,000/2)2 = 15,000 cases. For the plant, the average
inventory would be 20,000/2 = 10,000 cases. The total system average inventory would
be 22,500 + 15,000 + 10,000 = 47,500 cases.
4
(a) The leverage principle shows the relative change that must be made in cost, price, or
sales volume to affect a given change in the profit level. Usually it is used in
reference to the cost of goods sold to show the impact that small changes in the cost
of goods will have on profits and the important role that purchasing plays in the
profitability of the firm. The following simple profit and loss statements will show
how much change is needed in various activities to increase profits to $5,000,000.
Sales Price L&S
OH
COG
Current (+4%) (1%)
(-3%) (-6%) (-2%)
Sales
$55.0
$57.2a $55.5 $55.0 $55.0 $55.0
Cost of goods
27.5
28.6
27.5
27.5
27.5
27.0
Labor & salaries
15.0
15.6
15.0
14.5
15.0
15.0
Overhead
8.0
8.0
8.0
8.0
7.5
8.0
Profit
$ 4.5
$ 5.0 $ 5.0 $ 5.0 $ 5.0 $ 5.0
a
Sales - .7727xSales -8 = 5, where L&S is 0.2727 of Sales and COG is
0.5 of Sales. So, Sales = (5 + 8)/(1 0.7727) = 57.2

Due to the magnitude of cost of goods sold, it requires less than a 2 percent
change in COG to increase profits to $5,000,000.
(b) The current ROA as:
Profit margin = (4.5/55)100 = 8.2%
Investment turnover = 55/20 = 2.75
ROA = 2.758.2 = 22.6%

Or, ROA =
Profit/Assets

Reducing cost of goods by 7% will increase profits to 55 27.50.93 15 8 =


$6.43 and the profit margin now is 6.43100/55 = 11.7%. Inventory at 20% of total
assets is $4 million. If the cost of goods is reduced by 7%, inventory value will
decline to $40.93 = $3.72. Total assets will be 3.72 + 16 = $19.72 million. The
investment turnover is 55/19.72 = 2.789. The ROA now will be 11.72.789 = 32.63%
5
(a) A mixed purchasing strategy will generally be beneficial when prices show a definite
seasonality, they are predictable, and inventory costs associated with forward buying
are not excessive. In the problem, we should consider forward buying in the first half
of the year and hand-to-mouth buying in the last half. To test the various strategies,
compare (1) hand-to-mouth buying, (2) forward buying every 2 months, (3) forward
buying every 3 months, and (4) forward buying for the first 6 months. The results are
summarized in Table 10-1.

137

The inventory for the hand-to-mouth buying strategy can be approximated as


50,000/2 = 25,000. The carrying cost would be 0.304.9825,000 = $37,350 per
year.
The carrying cost for the 2-mouth forward buying strategy is:
0.304.88[(0.5100,000/2) + (0.550,000/2)] = $54,900
For the 3-month forward buying strategy:

For 2nd half of year

0.34.56[(0.5300,000/2) + (0.550,000/2)] = $119,700


From the total costs in Table 10-1, the best strategy is to forward buy the first sixmonth's requirements in January and hand-to-mouth buy for the last six months.
(b) Some possible disadvantages are:
Prices may fall rather than rise in the first six months
There may not be adequate storage space to accommodate such a large purchase.
The materials may be perishable and not easily stored.
Uncertainties in the requirements and carrying costs may void the strategy.

138

TABLE 10-1 A Comparison of Various Forward Buying Strategies with Hand-to-Mouth Buying
Hand-to-mouth buy

Jan
Feb
Mar
Apr
May
Jun
Jly
Aug
Sep
Oct
Nov
Dec

Price, Quantity,
$/unit
units
4.00
50,000
4.30
50,000
4.70
50,000
5.00
50,000
5.25
50,000
5.75
50,000
6.00
50,000
5.60
50,000
5.40
50,000
5.00
50,000
4.50
50,000
4.25
50,000
Subtotals
Inventory costs
Totals
Average price/unit

Total
$200,000
215,000
235,000
250,000
262,000
287,500
300,000
280,000
270,000
250,000
225,000
212,000
$2,987,500
37,350
$3,024,850
$4.98

2-month forward buy

Price,
$/unit
4.00

Quantity,
units
100,000

Total
$400,000

4.70

100,000

470,000

5.25

100,000

525,000

6.00
5.60
5.40
5.00
4.50
4.25

50,000
50,000
50,000
50,000
50,000
50,000

300,000
280,000
270,000
250,000
225,000
212,000
$2,932,500
54,900
$2,987,400
$4.88

3-month forward buy

6-month forward buy

Price,
$/unit
4.00

Quantity,
units
150,000

Total
$600,000

5.00

150,000

750,000

6.00
5.60
5.40
5.00
4.50
4.25

50,000
50,000
50,000
50,000
50,000
50,000

300,000
280,000
270,000
250,000
225,000
212,500
$2,887,500
72,150
$2,959,650
$4.81

Price,
$/unit
4.00

Quantity,
units
300,000

6.00
5.60
5.40
5.00
4.50
4.25

50,000
50,000
50,000
50,000
50,000
50,000

Total
$1,200,000

300,000
280,000
270,000
250,000
225,000
212,500
$2,737,500
119,700
$2,857,200
$4.56

139

6
(a) On the average, a total expenditure of 1.1025,000 = $27,500 should be made for
copper each month.

(b) For the next 4 months, the dollar averaging purchases would be:
(1)
(2)
Price, No. of
Month $/lb. lb.
1
1.32
20,833
2
1.05
26,190
3
1.10
25,000
4
0.95
28,947
100,970
a
50,486/4 = 12,622

(3)=(1)(2)
Total
cost,$
27,500
27,500
27,500
27,500
$110,000

(4)=(2)/2
Average
inventory, lb.
10,417
13,095
12,500
14,474
12,622a

The average per-lb. cost would be $110,000/100,970 = $1.089. The inventory


carrying cost over 4 months would be 0.201.089(4/12) 12,622 = $916.
If hand-to-mouth were used, we would have:
(1)
(2)
Price, No. of
Month $/lb. lb.
1
1.32
25,000
2
1.05
25,000
3
1.10
25,000
4
0.95
25,000
100,000
a
50,000/4 = 12,500

(3)=(1)(2)
Total
cost,$
33,000
26,250
27,500
23,750
$110,500

(4)=(2)/2
Average
inventory, lb.
12,500
12,500
12,500
12,500
12,500a

The average per-lb. cost would be $110,500/100,000 = $1.105. The inventory


carrying cost over 4 months would be 0.201.105(4/12) 12,500 = $921.
If 100,000 lb. of copper is purchased, the two strategies can be compared as
follows.
Purchase Inventory Total
Strategy
cost
cost
cost
Dollar averaging $108,900 + 916
= $109,816
Hand-to-mouth 110,500 + 921
= 111,421
Dollar averaging buying would be preferred.

7
For an inclusive quantity discount price incentive plan, we first compute the economic
order quantities for each range of price. Using

Q * 2 DS / IC
we compute

140

Q1* 2(500 )(15) / ( 0.20 )( 49.95) 38.75 cases


Q2* 2(500 )(15) / ( 0.20 )( 44.95) 40.85 cases

Since Q2* is outside of the second price bracket, Q1* is the only relevant quantity. Now
we check the total cost at Q1* and at the minimum quantities within the price break. We
solve:

TCi Pi D DS / Qi ICi Qi / 2
At Q = 38.75
TC = 49.95500 + 50015/38.75 + 0.249.9538.75/2
= $25,362
At Q = 50
TC = 44.95500 + 50015/50 + 0.244.9550/2
= $22,850
At Q = 80
TC = 39.95500 + 50015/80 + 0.239.9580/2
= $20,388
Floor polish should be purchased in quantities of 80 cases.
8
This noninclusive price discount problem requires solving the following relevant total
cost equation for various order quantities until the minimum cost is found.

TCi Pi D DS / Qi ICi Qi / 2
The computations can be shown in the table below given that D = 1,400, S = 75, and I =
0.25.

141

Q
20
50
100
200
300
400
500

550

600

Price
795
795
795
795
200795+100750
300
200795+200750
400
200795+200750
+100725
500
200795+200750
+150725
550
200795+200750
+200725
600

P D
+DS/Q
+ICQ/2
= Total cost
1,113,000.00 5,250.00
1,987.50 $1,120,237.50
1,113,000.00 2,100.00
4,968.75
1,120,068.75
1,113,000.00 1,050.00
9,937.50
1,123,987.50
1,113,000.00
525.00 19,875.00
1,133,400.00
1,092,000.00
350.00 29,250.00
1,121,600.00
1,081,500.00

262.50

38,625.00

1,120,387.50

1,068,200.00

210.00

47,687.50

1,116,097.50

1,063,363.64

190.91

52,218.75

1,115,773.27

1,059,333.33

175.00

56,750.00

1,116,258.33

The optimal purchase quantity is 550 motors.


9
(a) This problem is a good application of the transportation method of linear
programming. We begin by determining the costs for the current sourcing
arrangement.
Source
Dayton
Dayton
Kansas City
Minneapolis

Destination
Cincinnati
Baltimore
Dallas
Los Angeles

Price
3.40
3.40
3.45
3.25

Transport
0.05
0.15
0.08
0.24

Volume Cost
5,000 $17,250
1,000
3,550
2,500
8,825
1,200
4,188
Total $33,813

To optimize, we establish the following transportation cost matrix and solve it using
any appropriate method, such as the TRANLP module in LOGWARE.
Cincinnati
3.40

Dallas
3.44

Minneapolis
3.55

3.53

Los
Angeles
3.49
1200
3.65

Baltimore
3.46

1200
3.63

Kansas City

4800
3.52

3.45
Dayton
Requirements

Capacity

5000
5000

3.67
2500
2500

3.55
1200

1000
1000

9999

The total cost for this solution is $33,788 or a savings of $25 over the current sourcing.

142

(b) Because Minneapolis is at capacity, this supplier should be examined further. If


unlimited capacity were available at Minneapolis, all requirements would be met by
this supplier for a total cost of $33,248, or a savings of $565 for this material.
(c) The above analysis does indicate that too many suppliers are being used. Only two
are needed if Minneapolis continues to supply at the current level. If Minneapolis
can be expanded, it becomes the only supplier. Of course, whether the company
would risk a single supplier for this material must be left unanswered.
10
(a) The deal-buying equation (Equation 10-5) can be applied to this problem. First, find
the optimal order quantity before the discount.

Q*

2 DS

IC

2(120,000)( 40)
566 units
0.30(100)

Next, find the adjusted order quantity after the discount has been applied.
Q

dD
pQ *
5(120,000)
100(566)

21,648 units
(p d)I p d (100 5)(0.30) (100 5)

A large order size of 21,648 units should placed.


(b) The time that an order of this size will be held before it is depleted is given by
Q
21,648

0.18 years, or 9.4 weeks


D 120,000

143

INDUSTRIAL DISTRIBUTORS, INC.


Teaching Note
Strategy
The purpose of the Industrial Distributors case study is to illustrate the computation of
purchase quantities under inclusive and noninclusive price discounts and transport rateweight breaks. The INPOL module of LOGWARE is helpful in conducting the analysis.
As a teaching strategy, it may be worthwhile to begin any class discussion with the cost
tradeoffs that are present in such a problem as this. This will help to establish the nature
of the total cost equation that needs to be solved in this problem.
Answers to Questions
(1) What size of replenishment orders, to the nearest 50 units, should Walter place, given
the manufacturer's noninclusive price policy?

When price discounts are offered, purchase quantities are not simply determined by a
single formula. Due to discontinuities in the total cost curve as a function of order
quantity, the optimal order quantity is found by computing total costs for different
quantity values. In this case of both price and transport rate breaks plus warehousing
costs that can be affected by the order size, the following annual total cost formula is to
be solved.
TC = PD + RD +

SD
ICQ
+
+ W ( Q - 300)
Q
2

where
TC = total cost for quantity Q, $
PD = purchase cost for price P, $
RD = transport costs at rate R, $
SD/Q = ordering cost at quantity Q, $
ICQ/2 = carrying cost at quantity Q, $
W(Q-300) = public warehousing cost if Q is greater than 300 units, $
W = public warehousing rate, $ per unit per year
D = annual demand, units
P = price for orders of size Q, $ per unit
R = transport per unit for shipments of size Q, $ per unit
S = order processing cost, $ per order
I = annual carrying cost, %
C = product value, $ per unit
Q = size of purchase order, units
Under noninclusive price discounts, price is an average, determined by the number of
units in each break. For example, if 250 units are to be ordered, the average price per
unit would be computed as:

144

P250 =

(100 $700) + (100 $680) + (50 $670)


$686.00
250

A table of annual costs can now be developed, as shown in Table 1. To the nearest 50
units, the optimal purchase quantity should be 250 units.
(2) If the manufacturer's pricing policy were one where the prices in each quantity break
included all units purchased, should Walter change his replenishment order size?
The average price per unit is more easily determined in this case than the previous one.
Since all units are included in the price break back to the first unit, the average price is
simply the price associated with a given purchase quantity.
Finding the optimal purchase quantity is simply a matter of determining the total cost
for the quantities, found by the economic order quantity formula, assuming these
quantities are feasible, and for the quantities at the transport rate-weight break. The
comparison is made among the total costs of these alternatives. These costs are shown in
Table 2.
The order quantities, as determined by the economic order quantity formula for the
base price of $700, would be
Q* =

2 DS
2(1500)( 25)
=
= 18.3, or 18 units
0.3(700 + 45)
IC

where C is the $700 price per unit at Baltimore plus the $45 transport cost from
Baltimore, as determined by an LTL shipment (18 units 250 lb. = 4,500 lb.) at $18
2.5 cwt. = $45 per unit. The Q values for the other prices in the schedule lie outside the
feasible range of the price used to compute Q.
The optimal strategy is to purchase 201 units per order, which is one unit into the last
price break. Yes, Walter should alter his buying strategy.
TABLE 1 Annual Costs by Quantity Purchased for Noninclusive Price Discounts
Average
Purchase
Transport
Quantity
price
cost
cost
18a
$700.00
$1,050,000
$67,500
50
700.00
1,050,000
67,500
100
700.00
1,050,000
67,500
693.33
1,039,995
67,500
150
692.50
1,038,750
45,000
160
200
690.00
1,035,000
45,000
686.00
1,029,000
45,000
250
300
683.33
1,024,995
45,000
400
680.00
1,020,000
45,000
a
EOQ at a price of ($700 + 45+ 0.625) per unit.
b
First price break.
c
Transport rate break.
d
Second price break.

Ordering
cost
$2,083
750
375
250
234
188
150
125
94

Carrying
cost
$2,013
5,592
11,184
16,619
17,355
21,619
26,873
32,128
42,638

Warehouse
cost
$0
0
0
0
0
0
0
0
1,000

Total cost
$1,121,596
1,123,842
1,129,059
1,124,364
1,101,339
1,101,807
1,101,023Opt.
1,102,248
1,108,732

145

TABLE 2 Annual Costs by Quantity Purchased for Inclusive Price Discounts


Average
Purchase
Transport Ordering
Quantity
price
cost
cost
cost
$700.00
$1,050,000
$67,500
$2,083
18
680.00
19
670.00
19
680.00
1,020,000
67,500
371
101
680.00
1,020,000
45,000
234
160
670.00
1,005,000
45,000
187
201
a
Feasible EOQ at a price of ($700 + 45 + 0.625) per unit.
b
Infeasible EOQ at a price of ($680 + 45 + 0.625) per unit.
c
Infeasible EOQ at a price of ($670 + 30 + 0.625) per unit.
d
First price break.
e
Transport rate break.
f
Second price break.

Carrying
cost
$2,013

10,993
17,175
21,124

Warehouse
cost
$0

0
0
0

Total cost
$1,121596
Infeasible
Infeasible
1,098,864
1,082,409
1,071,311Opt.

146

CHAPTER 11
THE STORAGE AND HANDLING SYSTEM

All questions in this chapter require individual judgment and response. No answers are
offered.

147

CHAPTER 12
STORAGE AND HANDLING DECISIONS
2
Various alternatives are evaluated in Tables 12-1 to 12-4. The annual costs of each
alternative are plotted in Figure 12-1. The best economic choice is to use all public
warehousing.

148

or a Pure Public Warehouse Strategy

Privately-operated
Rented
Space
equirePrivate
Monthly
Monthly
Rented
Monthly
Monthly
ments,
allofixed cost variable
allostorage
handling
Monthly
sq. ft.b
cation
cost
cation
cost
cost
total cost
62,500
0%
$0
$0
100%
$30,000c
$50,000d
$80,000
50,000
0
0
0
100
24,000
40,000
64,000
37,500
0
0
0
100
18,000
30,000
48,000
25,000
0
0
0
100
12,000
20,000
32,000
12,500
0
0
0
100
6,000
10,000
16,000
3,125
0
0
0
100
1,500
2,500
4,000
15,625
0
0
0
100
7,500
12,500
20,000
28,125
0
0
0
100
13,500
22,500
36,000
37,500
0
0
0
100
18,000
30,000
48,000
43,750
0
0
0
100
21,000
35,000
56,000
50,000
0
0
0
100
24,000
40,000
64,000
56,250
0
0
0
100
27,000
45,000
72,000
21,875
$0
$0
$202,500
$337,500
$540,000
($5/lb.)
t.) = Thruput (lb.) (1/ 2 turns) (1/0.40 storage space ratio) (0.1 cu. ft./$) (1/10 ft.) (5 $/lb.)

2 and 100% of the demand through the rented warehouse, then 1,000,000 (1.00/2) 0.06 = $30,000
= $50,000

TABLE 12-2 Costs for a Mixed Warehouse Strategy Using a 10,000 Square Foot PrivatelyOperated Warehouse
Privately-operated
Rented
Space
WarerequirePrivate
Monthly
house
Monthly
Rented
Monthly
Monthly
ments,
allofixed cost variable
thruput,
allostorage
handling
Monthly
sq. ft.b
cation
Month
lb.a
cost
cation
cost
cost
total cost
84%
$25,200f
$42,000g
$80,192
Jan.
1,000,000
62,500
16%
$9,792d $3,200e
Feb.
800,000
50,000
20
9,792
3,200
80
19,200
32,000
64,192
Mar.
600,000
37,500
27
9,792
3,200
73
13,140
21,900
48,032
Apr.
400,000
25,000
40
9,792
3,200
60
7,200
12,000
32,192
May
200,000
12,500
80
9,792
3,200
20
1,200
2,000
16,192
June
50,000
3,125
100
9,792
1,000
0
0
0
10,792
July
250,000
15,625
64
9,792
3,200
36
2,700
4,500
20,192
Aug.
450,000
28,125
36
9,792
3,200
64
8,640
14,400
36,032
Sept.
600,000
37,500
27
9,792
3,200
73
13,140
21,900
48,032
Oct.
700,000
43,750
23
9,792
3,200
77
16,170
26,950
56,112
Nov.
800,000
50,000
20
9,792
3,200
80
19,200
32,000
64,192
56,250
18
9,792
3,200
82
22,140
36,900
72,032
Dec.
900,000
Totals
6,750,000
421,875
$117,504 $36,200
$147,930
$246,550
$548,184
a
Thruput (lb.) = Sales ($)/($5/lb.)
b
Space requirements (sq. ft.) = Thruput (lb.) (1/0.40) (0.1/10) 5 = Thruput (lb.) 0.0625
c
10,000/62,500 = 0.16
d
(3510,000/20) + 1010,000/12 = $9,792 per month
e
1,000,0000.160.02 = $3,200
f
Given a turnover ratio of 2 and 84% of the demand through the rented warehouse, then 1,000,000 (0.84/2) 0.06 = $25,200
d
1,000,000 0.84 0.05 = $42,000

150

TABLE 12-3 Costs for a Mixed Warehouse Size Strategy Using a 30,000 Square Foot PrivatelyOperated Warehouse
Privately-operated
Rented
Space
WarerequirePrivate
Monthly
house
Monthly
Rented
Monthly
Monthly
ments,
allofixed cost variable
thruput,
allostorage
handling
Monthly
sq. ft.b
cation
Month
lb.a
cost
cation
cost
cost
total cost
$29,375d $9,600e
52%
$15,600f
$26,900g
$80,575
Jan.
1,000,000
62,500
48%c
Feb.
800,000
50,000
60
29,375
9,600
40
9,600
16,000
64,575
Mar.
600,000
37,500
80
29,375
9,600
20
3,600
6,000
48,575
Apr.
400,000
25,000
100
29,375
8,000
0
0
0
37,375
May
200,000
12,500
100
29,375
4,000
0
0
0
33,375
June
50,000
3,125
100
29,375
1,000
0
0
0
30,375
July
250,000
15,625
100
29,375
5,000
0
0
0
34,375
Aug.
450,000
28,125
100
29,375
9,000
0
0
0
38,375
Sept.
600,000
37,500
80
29,375
9,600
20
3,600
6,000
48,575
Oct.
700,000
43,750
69
29,375
9,600
31
6,510
13,020
58,505
Nov.
800,000
50,000
60
29,375
9,600
40
9,600
16,000
64,575
56,250
53
29,375
9,600
47
12,690
21,150
72,815
Dec.
900,000
Totals
6,750,000
421,875
$352,500 $94,200
$61,200
$104,170
$612,070
a
Thruput (lb.) = Sales ($)/($5/lb.)
b
Space requirements (sq. ft.) = Thruput (lb.) 1/0.40 (0.1/10) 5 = Thruput (lb.) 0.0625
c
30,000/62,500 = 0.48
d
(3530,000/20) + 1030,000/12 = $29,375 per month
e
1,000,0000.480.02 = $9,600
f
Given a turnover ratio of 2 and 52% of the demand through the rented warehouse, then 1,000,000 (0.52/2) 0.06 = $15,600
d
1,000,000 0.52 0.05 = $26,000

151

TABLE 12-3 Costs for a Mixed Warehouse Size Strategy Using a 40,000 Square Foot PrivatelyOperated Warehouse
Privately-operated
Rented
Space
WarerequirePrivate
Monthly
house
Monthly
Rented
Monthly
Monthly
ments,
allofixed cost variable
thruput,
allostorage
handling
Monthly
sq. ft.b
cation
Month
lb.a
cost
cation
cost
cost
total cost
$39,167 $12,800e
36%
$10,800f
$18,000g
$80,767
Jan.
1,000,000
62,500
64%c
Feb.
800,000
50,000
80
39,167
12,800
20
4,800
8,000
64,767
Mar.
600,000
37,500
100
39,167
12,800
0
0
0
51,167
Apr.
400,000
25,000
100
39,167
8,000
0
0
0
47,167
May
200,000
12,500
100
39,167
4,000
0
0
0
43,167
June
50,000
3,125
100
39,167
1,000
0
0
0
40,167
July
250,000
15,625
100
39,167
5,000
0
0
0
44,167
Aug.
450,000
28,125
100
39,167
9,000
0
0
0
48,167
Sept.
600,000
37,500
100
39,167
12,000
0
0
0
51,167
Oct.
700,000
43,750
91
39,167
12,800
0
1,890
3,150
57,007
Nov.
800,000
50,000
80
39,167
12,800
20
4,800
8,000
64,767
56,250
71
39,167
12,800
29
7,830
13,050
72,847
Dec.
900,000
Totals
6,750,000
421,875
$470,004 $115,000
$30,120
$50,200
$665,324
a
Thruput (lb.) = Sales ($)/($5/lb.)
b
Space requirements (sq. ft.) = Thruput (lb.) 1/0.40 ( 0.1/10) 5 = Thruput (lb.) 0.0625
c
40,000/62,500 = 0.64
d
(3540,000/20) + 1040,000/12 = $39,167 per month
e
1,000,0000.640.02 = $12,800
f
Given a turnover ratio of 2 and 52% of the demand through the rented warehouse, then 1,000,000 (0.36/2) 0.06 = $10,800
d
1,000,000 0.36 0.05 = $18,000

152

FIGURE 12-1
Total Annual
Costs for a
Combined
Warehouse
Size Using
Private and
Public
Warehouse
Space

670

Total cost, $000s

650
630
610
590
570
550
530
0

10,000

30,000

40,000

Private warehouse space, sq. ft.

3
The annual cost of public warehousing is:

Handling
Storage
Total

$ 600,000
300,000
$ 900,000

The costs of private warehousing are:


Annual operating
$ 250,000
Annual lease payment 3150,000 =
450,000
Other fixed (one time)
400,000
The savings in operating costs of lease vs. public warehousing is:
Savings = $900,000 250,000 = $650,000/yr.

153

TABLE 12-5 Ten-Year Cash Flow Stream for Public vs. Leased Warehouse Comparison
Savings
DepreSavings
less
Savings: Pre-tax ciation
less
depreAfter-tax
Lease vs. net cash schedepreTaxes
ciation Savings
net cash
Year public
flow
dule
ciation
(35%)
& tax
less tax
flow
0
$0
(3,050)a
$0
0
0
0
0
($3,050)
593
208
385
442c
442
1
650
650
57b
2
650
650
57
593
208
385
442
442
3
650
650
57
593
208
385
442
442
4
650
650
57
593
208
385
442
442
5
650
650
57
593
208
385
442
442
6
650
650
57
593
208
385
442
442
7
650
650
58
592
207
385
443
443
8
650
650
0
650
228
422
442
442
9
650
650
0
650
228
422
442
442
650
0
650
228
422
442
442
10
650
$6,500
$3,450
$400
$6,100
$2,139 $3,961
$4,361
$1,311
a
Capitalization lease plus initial cash outlay, i.e., $2,650,154 + 400,000 = $3,050,154
b
Depreciation charge for each of seven years is 1/7 = 0.1429 such that 400,0000.1429 = $57,143
c
Add back depreciation, i.e., 385 + 57 = $442

Discount
factor
1/(1+i)j
0.9009
0.8116
0.7312
0.6587
0.5935
0.5346
0.4817
0.4339
0.3909
0.3522
NPV =

Discounted
cash flow
($3,050)
398
359
323
291
262
236
213
183
165
149
($471)

154

Capitalizing the lease over ten years, we have:


(1 011
. )10 1
PV 450,000
$2,650,154
011
. (1 011
. )10
The initial investment in $000s then is:
Initial investment = $2,650 + 400 = $3,050
The ten-year cash flow stream is shown in Table 12-5. Since the savings are expressed to
favor leasing and the net present value is negative, choose public warehousing.
4
Given:

k = $210/sq ft.
S = 100,000 sq. ft.
C = $0.01/ft.10,000 = $100/ft.
The width is:
C 8k
S
2C 8k

W*

100 8( 210)
100,000
2(100) 8( 210)

308 ft.
The length is:
L* S / W * 100,000 / 308 325 ft.

5
Space layout according to text Fig. 12-4(a) can be determined by the application of
equations 12-8 and 12-9. These equations specify the best number of shelf spaces and
the best number of double racks, respectively. Equations 12-10 and 12-11 give the length
and width of the building.
The optimal number of shelf spaces would be:

m1*

1
L

dCh 2aCs 2C p K ( w a ) L

2h
2( dCh C p )

1 400,000( 0.001) 2(10)( 0.50) 2(3.00) 50,000(8 10)( 4)

4
2( 400,000)( 0.001) 3.00
2( 4)

120.48, or 121

155

The optimal number of double racks would be:


n1*

1
wa

2( dCh C p ) K ( w a ) L

2h
dCh 2aCs 2C p

50,000(8 10 )( 4)
1
2[( 400,000)( 0.001) 3.00]

8 10 400,000( 0.001) 2(10)( 0.50) 2(3.00)


2( 4 )

52
The warehouse length would be:
u1 n1* ( w a ) 52(8 10) 936 ft.
and the width would be:
v1 2a m1* L 2(10) 121( 4) 504 ft.
6
According to Equation 12-17, the number of truck doors can be estimated by:

DH
CS

Therefore,
N = (7512,000) 3/(312,000) 8 = 9.37, or 10 doors
7
Summarizing the given information as follows:

Initial investment
Useful life
Salvage value @15%
of initial cost
Annual operating
expenses
Return on investment
before tax

Three
Five
Seven
type 1
type 2
type 3
units
units
units
$60,000
$50,000
$35,000
10 yr.
10 yr.
10 yr.
$ 9,000

$ 7,500

$ 5,250

$ 6,000

$12,500

$21,000

20%

20%

20%

An initial solution to this problem can be found through a discounted cash flow
analysis. Three alternatives are to be evaluated.

156

(1 0.2)10 1

1
9,000
PV1 60,000 6,000
10
10
(1 0.2)
0.2(1 0.2)
. )
60,000 6,000( 4.2) 9,000( 016
$83,760
PV2 50,000 12,500( 4.2 ) 7,500( 016
. )
50,000 52,500 1,200
$101,300
. )
PV3 35,000 21,000( 4.2) 5,250( 016
35,000 88,200 840
$122,360
The low present value of the Type 1 truck indicates that from among these three
alternatives, this would be the best buy.
8
Given:

Initial cost of equipment = $4,000


Operating costs 500 + 40(t - 1)2 - 30(t - 1)
Salvage value Sn = I(1 t/7)
Rate of return on investment = 20%
Replacement is expected to be with equipment of like kind
The best replacement year can be found by comparing the equivalent annual cost of a
sequence of similar equipment replaced every n years. The equivalent annual cost is:

AC n I j1 ( C j /(1 i ) j ) ( S n /(1 i ) n ) i (1 i ) n /(1 i ) n 1


n

Solving this equation for different years is facilitated if the equation is set up in tabular
form, as shown in Table 12-6.
The equipment should be replaced at the end of the third year of service although a 5year replacement cycle is also attractive.

157

TABLE 12-6 Equivalent Annual Cost Computations for Problem 8


(4)
(5)=(1+2-3)(4)
(3)
(1)
(2)
Factor
Salvage
Operating
Equivalent annual
Year,
value,
Initial
costs,
i(1 i ) n
n
j
n
S n / (1 i)
cost, ACn
n
investment, I C j / (1 i)
(1 i ) 1
1
$4,000
$416
$2,857
1.20
$1,871
a
1,984
0.65
1,821
2
4,000
770
1,323
0.47
3
4,000
1,117b
1,783
4
4,000
1,488
827
0.39
1,818
5
4,000
1,898
459
0.33
1,795
6
4,000
2,350
191
0.30
1,848
7
4,000
2,841
0
0.28
1,915
a
b

$416 + [500 + 40(2-1)2 30(2-1)]/(1+0.20)2 = $770


$770 + [500 + 40(3-1)2 30 (3-1)]/1+0.20)3 = $1,117

9
(a1) Layout by popularity involves locating the more frequently ordered items closest to
the outbound dock. Based on the average number of daily orders on which the item
appears, the items closest to the outbound dock would be ranked as follows:
B,I,E,A,F,H,J,C,G,D
The storage space might then be used as follows.

Inbound

H, F, A

A, E

J, C, G

H, J

B, I, E

Outbound
(a2) Layout by cube places the smallest items nearest the outbound dock. Using the
individual item size, the ranking would be as follows: A,E,I,C,J,H,G,B,F
The layout of items in the storage bays would be:

158

Inbound

F, D

H, D

H, G, B

D, J, C

E, A

E, I, C

Outbound
(a3) The cube-per-order index is created by ratioing the average required cubic footage
of a product to the average number of daily orders on which the item is requested.
Hence, this index is found as follows:
(1)
(2)
Space
required Daily
Product
cu. ft.
orders
A
5,000a
56
B
30,000
103
C
15,000
27
D
17,000
15
E
55,000
84
F
11,000
55
G
7,000
26
H
28,000
45
I
13,000
94
J
9,000
35
a
500 sq. ft. stacked 10 ft. high

(3)=(1)/(2)
CPO
index
89
291
556
1,133
655
200
269
622
138
257

Locating the products with the lowest index values nearest to the outbound dock
results in the following ranking and layout: A,I,F,J,G,B,C,H,E,D

159

Inbound

E, F

H, C

F, I, A

F, J, G, B

Outbound
(b) All of the above methods assume (1) that the product is moved to the storage
locations in large unit loads but retrieved from the storage locations in relatively
small quantities and (2) that only one product is retrieved during an out-and-back trip.
Therefore, these methods do not truly apply to the situation of multiple picks on the
same trip. However, they may be used with some degree of approximation if the
products can be aggregated as one and grouped together or zoned in the same section
of the warehouse.
10
This is an extra challenging problem that requires some knowledge of linear
programming. It may be formulated as follows.

Let Xij represent the amount per 1,000 units of product j stored in location i. Let Cij
be the handling time associated with storage bay i and product j. Gj is the capacity of
a bay for product j and Rj is the number of units of product j required to be stored.
The linear programming statement is:
Objective function
Zmin = .90X11 + .75X12 + .90X13 + .80X21 + .65X22 + .95X23
+ .60X31 + .70X32 + .65X33 + .70X41 + .55X42 + .45X43
+ .50X51 + .50X52 + .45X53 + .40X61 + .45X62 + .35X63

Subject to:
Capacity restrictions on bays

160

20X11 + 33.3X12 + 16.7X13 100


20X21 + 33.3X22 + 16.7X23 100
20X31 + 33.3X32 + 16.7X33 100
20X41 + 33.3X42 + 16.7X43 100
20X51 + 33.3X52 + 16.7X53 100
20X61 + 33.3X62 + 16.7X63 100

and storage requirements restrictions on products


X11 + X21 + X31 + X41 + X51 + X61 11
X12 + X22 + X32 + X42 + X52 + X62

X13 + X23 + X33 + X43 + X53 + X63 12

Solving the linear programming problem by means of any standard transportation code of
linear programming, such as LNPROG in LOGWARE, yields:
X12
X21
X22
X31
X43
X51
X53
X63

=
=
=
=
=
=
=
=

The total minimum


handling time is
138.68 hours

1.610
1.020
2.390
5.000
5.988
4.980
0.024
5.988

where Xs are in thousands of units. That is, product 1 should be stored in bays 3, 4, and 5
in quantities of 1,020, 5,000, and 4,980, respectively. Product 2 should be stored in bays
1 and 2 in quantities of 1,610 and 2,390, respectively. Product 3 should be stored in bays
4, 5, and 6 in quantities of 5,988, 24, and 5,988, respectively. Graphically, this is:

Bay
Product
1
2
3
% of bay
capacity

3
5,000

1,610

1,020
2,390

53.7

100.0

100.0

4,980
5,988

24

5,988

100.0

100.0

100.0

Require
-ments
11,000
4,000
12,000

161

CHAPTER 13
FACILITY LOCATION DECISIONS
1
(a) The center-of-gravity method involves finding the X,Y coordinates according to the
formulas:

V R X
X
V R
i

and

V R Y
Y
V R
i

i i

These formulas can be solved for in tabular form as follows.


Point
P1
P2
M1
M2
M3

X Y
3 8
8 2
2 5
6 4
8 8
Totals

Vi
5,000
7,000
3,500
3,000
5,500

Ri
.040
.040
.095
.095
.095

ViRi
200.0
280.0
332.5
285.0
522.5
1620.0

ViRiXi
600.0
2240.0
665.0
1710.0
4180.0
9395.0

ViRiYi
1600.0
560.0
1662.5
1140.0
4180.0
9142.5

Now,
X

9,395.0
5.8
1,620.0

and
Y

9,142.5
5.64
1,620.0

This solution has a total cost for transportation of $53,614.91. This problem may also
be solved using the COG module in LOGWARE.
(b) Solving for the exact center-of-gravity method requires numerous computations. We
now use the COG module of LOGWARE to assist us. A table of partial results is
shown below.

162

Iteration
number
0
1
2
3
.
.
.
50

X coord
5.799383
5.901199
5.933341
5.941554
.
.
.
5.939314

Y coord
5.643518
5.518863
5.446919
5.402429
.
.
.
5.317043

Total
cost
53,614.91 COG
53,510.85
53,483.97
53,474.60
.
.
.
53,467.71

After 50 iterations, there is no further change in total cost. The revised coordinates
are X = 5.94 and Y = 5.32 for a total cost of $53,467.71.
(c) The center-of-gravity solution can be one that is close to optimum when there are
many points in the problem and no one point has a dominant volume, that is, has a
larger volume relative to the others. Otherwise, the best single location can be at a
dominant location. The exact center-of-gravity approach has the capability to find
the minimal cost location.
Although the COG model only considers transportation costs that are constant per
mile, the transportation cost can be the major consideration in single facility location.
However, other costs such as labor, real estate, and taxes can also be important in
selecting one location over another. These are not directly considered by the model.
Although the COG model may seem of limited capability, it is a useful tool for
locating facilities where transportation costs are dominant. Location of oil wells in
the Gulf, truck terminals, and single warehouses are examples of application. It also
can be quite useful to provide a starting solution to more complex location models.
(d) Finding multiple locations by means of the center-of-gravity approach requires
assigned supply and demand volumes to specific facilities and then solving for the
center of gravity for each. In this problem, there are 3 market combinations that need
to be considered. This creates 3 scenarios that need to be evaluated. They can be
summarized as follows. Point volumes appear in the body of the table.
Scenario

Whse
1

P1
P2
1458a 2042

3542

4958

2708

3792

2292

3208

3750

5250

M1
3500

M2

M3

I
3000
3500

5500

3000

II
5500
3500

5500

III
2
1250 1750
3000
Allocated as a proportion of the volume to be served through the
warehouse. That is, 50003500/(3500 + 3000 + 5500) = 1458. The
volumes associated with other supply points are computed similarly.
a

163

The COG module in LOGWARE was used to find the exact centers of gravity for
each warehouse in each scenario. The computational results are:
Scenario
I
II
III

Warehouse 1
X
Y
2.00 5.00
5.84 4.04
7.06 7.28

Warehouse 2
X
Y
7.88 7.80
8.00 8.00
6.00 4.00

Total
cost
$39,050
35,699
46,568

Scenario II appears to be the best.


2
(a) The center-of-gravity formulas (Eqs. 13-5 and 13-6) can be solved using the COG
module of LOGWARE, or they can be solved in tabular form as shown below.
Point
A
B
C
D
E
F
G
H
I
J

X Y
50 0
10 10
30 15
40 20
10 25
40 30
0 35
5 45
40 45
20 50
Totals

Vi
9,000
1,600
3,000
700
2,000
400
500
8,000
1,500
4,000

Ri
.75
.75
.75
.75
.75
.75
.75
.75
.75
.75

ViRi
6,750
1,200
2,250
525
1,500
300
375
6,000
1,125
3,000
23,025

ViRiXi
337,500
12,000
67,500
21,000
15,000
12,000
0
30,000
45,000
60,000
600,000

ViRiYi
0
12,000
33,750
10,500
37,500
9,000
13,125
270,000
50,625
150,000
586,500

Now,
X

600,000
.
261
23,025

and
Y

586,500
25.5
23,025

The total cost of this location is $609,765. The exact center-of-gravity coordinates
are:
X 2351
. , Y 26.98
with a total cost of $608,478.
(b) The number of points, even in this small problem, requires us to apply some
heuristics to find which patient clusters should be assigned to which warehouses. We
will use a clustering technique whereby patient clusters are grouped by proximity
until two clusters are found. The procedure works as follows.

164

There are as many clusters as there are points, which is 10 in this case.
The closest points are found and replaced with a single point with the combined
volume located at the center of gravity point. There is now one less cluster.
The next closest two points/clusters are found, and they are further combined and
located at their center of gravity.
The process continues until only two clusters remain. The centers of gravity for
these two clusters will be the desired clinic locations.
Applying the clustering technique, we start by combining points D and F into
cluster DF.
X

40( 700)( 0.75) 40( 400)( 0.75)


40.00
700( 0.75) 400( 0.75)

and
Y

20( 700 )( 0.75) 30( 400)( 0.75)


23.64
700( 0.75) 400( 0.75)

Continuing this process, we would form two clusters containing A, C, D, and F, and
B, E, G, H, I, and J. The centers of gravity would be:
Cluster 1 - ACDF
X 50.00, X 0.00
Cluster 2 - BEGHIJ
X 5.00, Y 45.00
for a total cost of $241,828.
These are the same results obtained from the MULTICOG module in
LOGWARE.
(c) The second clinic can save $608,278 241,828 = $366,458 in direct costs annually.
This savings does not exceed the annual fixed costs of $500,000 required to maintain
a second clinic. On economic grounds, it should not be built.
3
(a) The center-of-gravity location can be determined by forming the following table or
by using the COG module in LOGWARE. The coordinates for each location must be
approximated.

165

Point
A
B
C
D
E
F
G
H
I

X
Y
1.5 6.6
4.7 7.3
8.0 7.1
1.5 4.0
5.0 4.9
8.5 5.1
1.5 1.3
4.4 1.8
7.8 1.8
Totals

Vi
10,000
5,000
70,000
30,000
40,000
12,000
90,000
7,000
10,000

Ri
.10
.10
.10
.10
.10
.10
.10
.10
.10

ViRi
1,000
500
7,000
3,000
4,000
1,200
9,000
700
1,000
27,400

ViRiXi
1,500
2,350
56,000
4,500
20,000
10,200
13,500
3,080
7,800
118,930

ViRiYi
6,600
3,650
49,700
12,000
19,600
6,120
11,700
1,260
1,800
112,430

The center-of-gravity coordinates are:


X

118,930
4.34
27,400

and
Y

112,430
.
410
27,400

with a total cost of $195,966.


(b) After 100 iterations in the COG module, the exact center of gravity was found to be:
X 4.75, Y 4.62
with a total cost of $195,367. In this case, using the exact center of gravity
coordinates as compared with the approximate ones reduced costs by only:
195,966 195,367
100 0.3%
195,966
(c) Additional costs can be included in the analysis, although not necessarily in the
model. The COG model can evaluate the variable costs of location. Other costs are
compared with these.
4
We begin by developing a 3-dimensional transportation problem. The cost matrix is
developed in the same manner as that in text Fig. 13-11. The initial throughput of W1 and
W2 is found by assuming that an equal amount of the customer demand flows through
each warehouse. The cell cost for W1-C1 would be:

[100(200,000/2)0.7]/(200,000/2 )+ 2 + 4 + 0 = 3.2 + 2 + 4 = 9.2

166

In fact, the cell costs are identical to those in text Fig. 13-11, except that there is no fixed
cost element.
Using the transportation method of linear programming (e.g., the TRANLP module in
LOGWARE), the cell cost and solution matrix for iteration 1 is shown in Fig. 13-1. The
solution shows that only W2 remains, and the solution process can be terminated.
A summary of the costs is shown in Table 13-1. The total cost is $2,213,714, and the
product is produced in plant P2 and stocked in warehouse W2. No further iterations are
needed since only one warehouse is used and no further dropping of warehouses is
possible.
TABLE 13-1 Summary Information for
Solution to Problem 4
Whse 1
Whse throughput
Costs:
Transportation
Inbound
Outbound
Inventory
Warehousing
Fixed
Production
Total

Whse 2

200,000

$0
0
0
0
0
0

$400,000
300,000
513,714
200,000
0
800,000
$2,213,714

Iteration 2
A repeat of the iteration 1 solution.

Plants

P1
P2

Whses

W1

W2
Capacity/
Reqmts
a

Warehouses
W1
W2
9
4
0
0
6
8
0
200,000
99a
0
60,000
99
0
799,999
60,000

999,999

Stop iterating.

Customers
C2
C3
99
99

C1
a

99

Capacity
60,000

99

99

99
999,999

9.2

8.2

10.2

60,000

6.2
50,000

5.2
100,000

6.2
50,000

999,999

50,000

100,000

50,000

High rate of $99/unit for an inadmissible cell.

FIGURE 13-1 Cell Cost and Solution Matrix for Iteration 1 of Problem 4
5
We begin by forming the cell cost matrix of a 3-dimensional transportation problem, as
shown in Figure 13-2. It is similar to the text Figure 13-1 except that the capacity for
warehouse 1 is set at 75,000. Solving the problem by means of the transportation method
shows the solution given in Figure 13-2.

167

Plants

P1
P2

Whses

W1

Warehouses
W1
W2
9
4
60,000
6
8
40,000
100,000
100
0
899,999
100
0

100

Customers
C2
C3
100
100

100

100

100

9.7

8.7
100,000
7.2

10.7

C1

Capacity
60,000
999,999

8.2
50,000

999,999
8.2
50,000

W2
100,000
Capacity/
Reqmts
999,999
100,000
50,000
100,000
50,000
FIGURE 13-2 Cell Cost and Solution Matrix for Iteration 1 of Problem 5
Given the solution from iteration 1, the per-unit inventory and fixed costs are revised.
Inventory
W1

$100(100,000) 0.7
$3.16 / unit
100,000 units

W2

$100(100,000) 0.7
$3.16 / unit
100,000 units

Fixed
W1
W2

$100,000/100,000 = $1.00/unit
$400,000/100,000 = $4.00/unit

Adding outbound transportation and warehouse handling to per-unit inventory and fixed
costs gives the following cell costs.

W1
W2

C1
10.2
10.2

C2
9.2
9.2

C3
11.2
10.2

Revising the warehouse-customer cell costs and solving gives the same warehouse
throughputs, so cell costs will no longer change. A stopping points is reached. The
solution is the same as that in Figure 13-2.
A summary of the costs is:

168

Cost type
Production
Inbound transportation
Outbound transportation
Fixed
Inventory carrying
Handling
Subtotal
Total

Warehouse 1
100,000 cwt.
60,0004 =
$240,000
40,0004 =
160,000
60,0000 =
0
40,0004 =
160,000
100,0003 =
300,000

Warehouse 2
100,000 cwt.
100,0004 =
$400,000
100,0002 =

200,000

50,0002 =
50,0002 =

100,000
100,000
100,000
400,000
100(100,000)0.7 = 316,228 100(100,000)0.7 = 316,228
100,000
100,0002 =
200,000 100,0001 =
$1,476,228
$1,616,228
$3,092,456

Compared with the costs from the text example, the cost difference is $3,092,456
2,613,714 = $478,742. This is the penalty for restricting a warehouse with economic
benefit to the network.
6
Prepare a matrix for a 3-dimensional transportation problem like that in text Figure 1311, except that the per-unit cell costs for warehouse 2 to customer are reduced by $1/unit
to reflect the reduction in that warehouses fixed costs. That is, $200,000/200,000 =
$1/unit instead of $400,000/200,000 = $2/unit. The matrix setup and first iteration
solution are shown in Figure 13-3.
Customers

Warehouses

W1
4a
Plants

C1
99b

C2
99

C3
99

P1

60,000
8

P2
Warehouses

W2
9

Plant &
warehouse
capacities

W1

W2
Warehouse
capacity &
customer
demand

0
60,000
99b

60,000

6
200,000
99

99
9.7

0
799,999

999,999c

99

99
999,999c

8.7

10.7

7.2e
50,000

6.2
100,000

7.2
50,000

50,000

100,000

50,000

60,000
999,999c

Production plus inbound transport rates, that is, 4 + 0 = 4.


Used to represent an infinitely high cost.
c
Used to represent unlimited capacity.
d
Inventory carrying, warehousing, outbound transportation, and fixed rates, that is,
3.2 + 2 + 4 + 0.5 = 9.7.
e
3.2 + 1 + 2 + 1 = 7.2.
b

FIGURE 13-2

Cell Cost and Solution Matrix for Iteration 1 of Problem 6

169

The results show that one warehouse is to be used. Further computations are not
needed, as further warehouse consolidation is not possible. The total network costs are
the same as those in the text example minus the $200,000 reduction in fixed costs for a
total coat of $2,413,714.
8
This problem requires us to rework the dynamic programming solution to the example
problem given in the text. The only change is that the cost of moving from one location
to another is now $300,000 instead of $100,000. We begin with the last year and
determine the best action based on the highest net profits. The action will be to move
(M) or to stay (S). For example, given the discounted moving cost of 300,000/(1 +
0.20)(4) = $144,676, we evaluate each course of action, assuming that we are in location
alternative A at the end of year 4. From the location profits of text Table 13-6, we
generate the following table for location A.
Alternative (x)
A
B
P5(A) = max C
D
E

Location
profit
$1,336,000
1,398,200
1,457,600
1,486,600
1,526,000

Moving
cost
-

0
144,676
144,676
144,676
144,676

Net
profit
= $1,336,000
= 1,253,524
= 1,312,924
= 1,341,924
= 1,381,324

The best action in the beginning of the 5th year, if we are already in location A, is to
move to location E. This is an entry in Table 13-2.
Once each of the five alternatives is evaluated for the 5th year, then the 4th year
alternatives are evaluated. The moving cost is 300,000/(1 + 0.2)(3) = $173,611. We now
include the profits for the subsequent years in our calculations.
After Table 13-2 is completed, we search the first column for the highest cumulative
profit. This is initially to locate in location D and remain there throughout the subsequent
years.
9
(a) Using PMED software in LOGWARE and the PMED02.DAT database, solve for the
number of locations from 1 to 9. The best locations for each number of sites are
given in the table below.

170

TABLE 13-2 Location-Relocation Strategies Over a Five-Year Planning Horizon with Cumulative Profits Shown from Year j
to Year 5 for Problem 8
Year from present date j
Warehouse
5th
1st
4th
3rd
2nd
location
StraStraStraStraaltertegya
P2(x)
tegya
P3(x)
tegya
P4(x)
tegya
P5(x)
P1(x)
natives (x)
A
$3,557,767
SA
$3,363,767
SA
$3,007,667
MD
$2,268,289
MD
$1,381,324
3,379,667
SB
3,007,667
MD
2,268,289
MD
1,398,000
B
3,556,167
SB
3,500,900
SC
3,156,200
SC
2,319,800
SC
1,457,600
C
3,673,200
SC
3,553,600
SD
3,216,000
SD
2,459,900
SD
1,486,600
D b
3,720,300
SD
3,374,700
SE
3,071,300
SE
2,355,800
SE
1,526,000
E
3,534,100
SE
a
Strategy symbol refers to staying (S) in the designated location or moving (M) to a new location as indicated.
b
Arrows indicate maximum profit location plan when warehouse is initially located at D.

Strategya
ME
SB
SC
SD
SE

171

Number
of sites
1
2
3
4
5
6
7
8
9

Total cost
$11,694,821
7,634,242
7,596,604
8,381,775
9,455,339
11,536,669
13,909,997
16,581,348
20,000,338

The optimal number of sites is 3 and they are to be located at Cincinnati, Phoenix,
and Denver.

(b) The optimal cost for four sites is $8,381,775, as found in part a. The company
operates the same sites as found in the optimal solution for four sites. Therefore, the
cost savings comes from a reassignment of customers to the sites. The savings is
$35,000,000 8,381,775 = $26,618,225 without any major investment. All of this
may not be recovered since there may be other operating costs included that are not
directly associated with location.
The savings between the optimal 4 sites and the optimal 3 sites is $8,381,775
7,596,604 = $785,171. There is also the reclamation of the salvage value of two
incinerators (close Chicago and Atlanta) and the construction of a new one
(Cincinnati). If the real estate recovery cost exceeds the new construction cost, that
would add to the savings, otherwise a ROI estimation is needed to see there is an
adequate return from the savings on the net investment.
Reallocation of customers among the existing incinerators is certainly attractive
and the reduction of the number of incinerators from 4 to 3 is also attractive as long
as there is no net investment required.

(c) Increase the annual volume for Los Angeles and Seattle markets by a factor of 10 and
re-solve the problem as in part a. Selected results are as follows:
Number
of sites
2
3
4
5

Total cost
$10,506,286
9,831,846
9,810,216
10,595,387

The optimal number of sites is four. An additional site at Seattle is needed, compared
with the three locations found in part a.

172

10
(a) We can apply Huff's model of retail gravitation to this problem. The solution table
(Table 13-3) can be developed. Summarizing, branch A can be expected to attract
11,735/(11,735 + 11,765) = 49.9% of the customers, and branch B should attract the
remaining 50.1%.
TABLE 13-3 Estimate of the Number of Customers Attracted to Each Branch
Bank
Pij
S j / Tij2

Time to ja
Customer i
1
2
3
4
5
6
7
8
9
a

A
0.28
0.10
0.28
0.40
0.20
0.50
0.45
0.57
0.67

B
0.72
0.50
0.45
0.20
0.40
0.50
0.28
0.20
0.54

Tij2
A
0.08
0.01
0.08
0.16
0.04
0.25
0.20
0.32
0.45

S j / Tij2
B
0.52
0.25
0.20
0.04
0.16
0.25
0.08
0.04
0.29

A
12.5
100.0
12.5
6.3
25.0
4.0
5.0
3.1
2.2

B
1.4
2.8
3.5
17.5
4.4
2.8
8.8
17.5
2.4

S
j

A
0.90
0.97
0.78
0.26
0.85
0.59
0.36
0.15
0.48

E ij Pij Ci

/ Tij2
B
0.10
0.03
0.22
0.74
0.15
0.41
0.64
0.85
0.52

A
900
1,940
3,120
1,820
850
885
1,440
300
480
11,735

B
100
60
880
5,180
150
625
2,560
1,700
520
11,765

Time = ( X i X )2 (Yi Y )2 / 50

(b) The economic analysis of site A would be:


Revenue (100No. of customers) $1,173,500
Operating expenses
300,000
Profit
$ 873,000

Return on investment 873,000/750,000 = 116.4%


The ROI seems sufficiently high so that the branch should be constructed.
(c) The size of a branch and its proximity to customers may be too simple to explain the
market share of each. The nature of the services offered, the accessibility of the site,
and the reputation of the bank may be just as important in estimating patronage.
What will the countermoves be of the competing branch? Adding another branch
and locating near branch A could substantially reduce its market share. How likely is
this to happen?
Are the customer numbers stable? Will a third or fourth bank be locating
branches in the region?
Can we expect that customers will drive such long distances to seek banking
services?

173

11
This problem can be solved as an integer linear programming problem similar to the Ohio
Trust Company example in the text. First, we create a table showing the counties that are
adjacent to each county. That is,
Counties under
consideration
1. Williams
2. Fulton
3. Lucas
4. Ottawa
5. Defiance
6. Henry
7. Wood
8. Sandusky
9. Paulding
10. Putnam
11. Hancock
12. Seneca
13. Van Wert
14. Allen
15. Hardin
16. Wyandot
17. Mercer
18. Auglaize
19. Marion
20. Shelby
21. Logan

Adjacent counties
by number
2,5,6
1,3,6
2,4,6,7
3,7,8
1,6,9,10
1,2,3,5,7,10,11
3,4,6,8,10,11,12
4,7,12
5,10,13
5,6,7,9,11,13,14
6,7,10,12,14,15,16
7,8,11,16
9,10,14,17,18
10,11,13,15,18
11,14,16,18,19,21
11,12,15,19
13,18
13,14,15,17,20,21
15,16
18,21
15,18,20

Next, according to the problem formulation given in the Ohio Trust Company example,
we can build the matrix as given in the prepared database called ILP03.DAT. The
problem formulation is shown in Table 13-4. This matrix can be solved by the integerprogramming module (MIPROG) in LOGWARE for solution. Note that all coefficients
are 1s or 0s. A coefficient of 1 is given to each county and its adjacent counties. The
sum of all constraints must be 1 or greater. The Xs take on the values of 0 or 1. An X of
1 means that the branch is located in the county.
Solving this problem using the integer-programming module in LOGWARE shows
that a minimum of 5 principal places of business are needed. They should be located in
Henry, Wood, Putnam, Hardin, and Auglaize counties.

174

TABLE 13-4 Coefficient Matrix Setup for Problem 11


Obj Fun
Constraints
Williams
Fulton
Lucas
Ottawa
Defiance
Henry
Wood
Sandusky
Paulding
Putnam
Hancock
Seneca
Van Wert
Allen
Hardin
Wyandot
Mercer
Auglaize
Marion
Shelby
Logan

X1
1

X2
1

1
1

1
1
1

1
1

X3
1

1
1
1
1
1

X4
1

X5
1

X6
1

1
1
1

1
1
1
1
1
1
1
1

1 1

1
1
1

1
1
1 1

X7
1

1
1

X8
1

X9
1

1
1

1
1
1

X10 X11 X12 X13 X14 X15 X16 X17 X18 X19 X20 X21
1
1
1
1
1
1
1
1
1
1
1
1

1
1

1
1
1
1
1 1

1
1

1
1
1
1
1
1
1
1

1
1

1
1
1
1
1
1

1
1
1
1
1
1
1

1
1

1
1

1
1
1

1
1

1
1
1

1
1
1
1

1
1
1
1

1
1

1
1

1
1

1
1

1
1
1

RHS

1
1
1
1
1
1
1
1
1
1
1
1

1
1
1
1
1
1
1
1

175

12
This problem can be solved with the aid of the PMED program in LOGWARE. A
database has been prepared for it called PMED04.DAT. The database shows the fixed
costs for a single site. When other numbers are to be evaluated, the FOC must be
recalculated and entered into the database. The scaling factor is set at 1 for this problem.
The fixed operating cost must be calculated for each possible number of locations. Using
PMED in LOGWARE gives the following results based on recalculated FOC values, an
estimation of vendor to laboratories transportation cost, and an enumerative search.
No. of
locations
1
2

Volume,
Sites
lb.
Chicago
680,000
Cleveland
515,000
L. Angeles
165,000
Total
680,000
3
New York
235,000
Chicago
280,000
L. Angeles
165,000
Total
680,000
4
New York
200,000
Atlanta
145,000
Chicago
170,000
L. Angeles
165,000
Total
680,000
5
New York
200,000
Atlanta
100,000
Chicago
170,000
Dallas
60,000
L. Angeles
150,000
Total
680,000
6
New York
200,000
Atlanta
65,000
Miami
35,000
Chicago
170,000
Dallas
60,000
L. Angeles
150,000
Total
680,000
1
515,000x310x0.02=3,193,000
2

Outbound
cost, $
28,350,000

Inbound
distance,
mi.
0
310
1,750

15,083,500
713
0
1,750
10,641,999
713
585
0
1,750
7,515,250
713
585
0
790
1,750
6,079,249
713
585
1180
0
790
1,750
5,029,250

Inbound
cost, $
0
3,193,0001
5,775,000
8,968,000
3,351,100
0
5,775,000
9,126,100
2,852,000
1,696,500
0
5,775,000
10,313,500
2,852,000
1,170,000
0
948,000
5,250,000
10,220,000
2,852,000
760,500
826,000
0
948,000
5,250,000
10,636,500

FOC, $
5,000,000
3,535,5342
3,535,534
7,071,068
2,886,751
2,886,751
2,886,751
8,660,253
2,500,000
2,500,000
2,500,000
2,500,000
10,000,000
2,236,067
2,236,067
2,236,067
2,236,067
2,236,067
11,180,335
2,041,241
2,041,241
2,041,241
2,041,241
2,041,241
2,041,241
12,247,446

Total cost,
$
33,350,000

31,122,568

28,428,352

27,828,750

27,479,584

27,913,196

5,000,000 2 / 2 3,535,534

The PMED program is used to find the best combination of sites for a particular
number of sites to be found. The fixed cost must be adjusted for the number of sites
being evaluated. It should be recognized that the model handles only the outbound leg of
the network (sites to serve laboratories). The vendor to site transportation cost is
included externally, as shown in the previous table. Calculating the distances between
vendor and the selected sites easily can be done by using the MILES module in
LOGWARE with a scaling factor of 1. Then, inbound transport costs are a product of
site volume, distance, and the inbound rate.

176

Searching from 1 to N sites shows that outbound transportation costs decrease while
inbound and fixed costs increase with increasing numbers of sites. Initially, total cost
declines until 5 sites are reached after which total cost increases. We select 5 sites as
economically the best number. Their customer assignments are
Location
number
1
2
3
4
5

Assignments
New York
Atlanta
Chicago
Dallas
Los Angeles

Volume
200,000
100,000
170,000
60,000
150,000

Customers
1, 2, 3, and 12
4 and 5
6, 7, 8, 9, 10, and 11
13, 14, and 16
15, 17, 18, 19, and 20

A map of the solution is as follows.

The total cost for 5 sites is $27,479,584.


13
After changing the fixed costs in the problem setup matrix, the following solution is
found.

177

OPTIMAL SOLUTION
Variable
X(1)
=
X(2)
=
X(3)
=
X(4)
=
X(5)
=
X(6)
=
X(7)
=
X(8)
=
X(9)
=
X(10) =
X(11) =
X(12) =
X(13) =
X(14) =
X(15) =
X(16) =
X(17) =
X(18) =
X(19) =
X(20) =
X(21) =
X(22) =
X(23) =
X(24) =
X(25) =
X(26) =
X(27) =
X(28) =
X(29) =
X(30) =
X(31) =
X(32) =

Value
.0000
10000.0000
50000.0000
.0000
.0000
.0000
50000.0000
90000.0000
.0000
.0000
.0000
.0000
.0000
30000.0000
20000.0000
.0000
.0000
.0000
20000.0000
.0000
40000.0000
.0000
.0000
.0000
1.0000
.0000
1.0000
1.0000
1.0000
.0000
.0000
.0000

Rate
8.0000
7.0000
9.0000
11.0000
10.0000
11.0000
12.0000
11.0000
13.0000
8.0000
7.0000
8.0000
6.0000
5.0000
7.0000
11.0000
10.0000
11.0000
9.0000
8.0000
10.0000
7.0000
6.0000
7.0000
100000.0000
500000.0000
140000.0000
260000.0000
220000.0000
70000.0000
130000.0000
110000.0000

Objective function value =

Cost
.0000
70000.0000
450000.0000
.0000
.0000
.0000
600000.0000
990000.0000
.0000
.0000
.0000
.0000
.0000
150000.0000
140000.0000
.0000
.0000
.0000
180000.0000
.0000
400000.0000
.0000
.0000
.0000
100000.0000
.0000
140000.0000
260000.0000
220000.0000
.0000
.0000
.0000

Variable label
P1S1W1C1
P1S1W1C2
P1S1W1C3
P1S1W2C1
P1S1W2C2
P1S1W2C3
P1S2W1C1
P1S2W1C2
P1S2W1C3
P1S2W2C1
P1S2W2C2
P1S2W2C3
P2S1W1C1
P2S1W1C2
P2S1W1C3
P2S1W2C1
P2S1W2C2
P2S1W2C3
P2S2W1C1
P2S2W1C2
P2S2W1C3
P2S2W2C1
P2S2W2C2
P2S2W2C3
zW1
zW2
yW1C1
yW1C2
yW1C3
yW2C1
yW2C2
yW2C3

3700000.00

Note that warehouse 2 is no longer used in favor of all products flowing through
warehouse 1.
14
(a) The demand of customer 1 for product 1 increases to 100,000 cwt. In the problem
matrix of ILP02.DAT the following cell values are changed.

Cell
Dem P1W1C1, yW1C1
Dem P1W2C1, yW2C1
Cap-W1, yW1C1
Cap-W2, yW2C1
Obj. coef., yW2C1
Obj. coef., yW2C1

From
-50000
-50000
70000
70000
140000
70000

To
-100000
-100000
120000
120000
240000
120000

The result shows that warehouse 2 is still the only warehouse used, and the
products are sourced from plant 2. However, the costs have increased to $3,500,000.

178

(b) Using the ILP02.DAT file in MIPROG of LOGWARE, the following changes are
made to the following cells.
Cell

Obj. Coef., P2S2W1C1


Obj. Coef., P2S2W1C2
Obj. Coef., P2S2W1C3
Obj. Coef., P2S2W2C1
Obj. Coef., P2S2W2C2
Obj. Coef., P2S2W2C1

From
9
8
10
7
6
7

To
12
11
13
10
9
10

The result shows that warehouse 2 is still the only warehouse used, and the products
are sourced from plant 2. However, the costs have increased to $3,380,000.
(c) Using the ILP02.DAT file in MIPROG of LOGWARE, the changes are made to the
following cells.
Cell

Obj. Coef., yW2C1


Obj. Coef., yW2C2
Obj. Coef., yW2C3

From
70,000a
130,000
110,000

To
280,000
520,000
440,000

The sum of demand for the same customer for all


products multiplied by the handling rate, i.e.,
(50,000 + 20,000) $4/cwt. = 280,000.

The solution for product 1 shows that 50,000 cwt. flows from plant 1 through
warehouse 1 and on to customer 1. The remainder flows from plant 2 through
warehouse 2 and on to customers 2 and 3.
For product 2, plant 1 supplies warehouse 1 and customer 1 with 20,000 cwt. The
remaining 90,000 cwt. flows from plant 2 through warehouse 2 to customers 2 and 3.
The total cost is $3,920,000.
(d) Making some slight revisions in file ILP02.DAT can adjust the capacities on plant 1.
The cell changes to make are:
Cell
Cap-P1S1, RHS
Cap-P1S2, RHS

From
60,000
999,999

To
150,000
90,000

Both warehouses are now used for both products.


summarized as:

The solution can be

179

Product
1
1
1
1
2
2
2

Plant
1 50,000 cwt.
1 60,000
2 40,000
2 50,000
1 20,000
2 30,000
2 60,000

Warehouse
1 50,000
2 60,000
2 40,000
2 50,000
1 20,000
2 30,000
2 60,000

Customer
1 50,000 cwt.
2 60,000
2 40,000
3 50,000
1 20,000
2 30,000
3 60,000

The total cost is $3,270,000


(e) Again revising the ILP02.DAT file by changing cells Obj. coef., P2S1W2C3 and Obj.
Coef., P2S2W2C3 to have a very high cost (999), these cells are locked out of
consideration. The solution is the same as the text example except that the customers
both products are serve from warehouse 1. The total cost is $3,340,000.
15
This problem follows the form of the Ohio Trust Company example in the text. First,
identify the zones that are within 30 minutes of any particular zone. That is,
Zone
no.
1
2
3
4
5
6
7
8
9
10

Zones within 30 minutes


1,2,4,7,8,9,10
1,2,3,4,7,9,10
2,3,4,5,8,9,10
1,2,3,4,5,6,10
3,4,5,6,7,8
4,5,6,7,8
1,2,5,6,7,9,10
1,3,5,6,8,9,10
1,2,3,7,8,9,10
1,2,3,4,7,8,9,10

Using the MIPROG module in LOGWARE, the following matrix can be defined.

180

The solution from MIPROG is:


OPTIMAL SOLUTION
Variable
Value
Rate
X(1)
=
1.0000
1.0000
X(2)
=
.0000
1.0000
X(3)
=
.0000
1.0000
X(4)
=
1.0000
1.0000
X(5)
=
.0000
1.0000
X(6)
=
.0000
1.0000
X(7)
=
.0000
1.0000
X(8)
=
.0000
1.0000
X(9)
=
.0000
1.0000
X(10) =
.0000
1.0000
Objective function value =

Cost
1.0000
.0000
.0000
1.0000
.0000
.0000
.0000
.0000
.0000
.0000

Variable label
1
2
3
4
5
6
7
8
9
10

2.00

The optimal solution is to place claims adjuster stations in zones 1 and 4.


16
This is a location problem where the dominant location factor is transportation cost is and
this cost is determined from optimizing the multi-stop routes originating at the material
yard. The ROUTER module in LOGWARE can be used to generate these routes for each
yard location. A database file for this problem (RTR13.DAT) has been prepared.
Solving this problem requires balancing the cost of transporting the merchandise to
the customers with the operating cost of the material yards at various locations.
Optimizing the routing from the current material yard gives the route design shown in
Figure 13-3. A minimum of 9 trucks are required to meet all constraints on the problem.
The total daily cost for this location is the route cost + vehicle costs + yard operating
cost, or P4,500.87 + 9 x P200 + P350 = P6,650.87.

FIGURE 13-3 Optimized Routing from Current Material Yard Location


*** SUMMARY REPORT ***

181

TIME/DISTANCE/COST INFORMATION

Route
no
1
2
3
4
5
6
7
8
9
Total

Route
Run Stop
Brk Stem
time, time, time, time, time,
Start Return No of
Route
hr
hr
hr
hr
hr
time
time stops dist,Mi
9.9
6.4
2.5
1.0
5.0 08:00AM 05:56PM
4
205
3.4
1.6
1.8
.0
.7 08:00AM 11:25AM
3
52
10.0
6.5
2.5
1.0
3.1 08:00AM 05:58PM
4
207
7.4
4.1
2.3
1.0
2.2 08:00AM 03:24PM
3
130
9.6
6.4
2.3
1.0
5.3 08:00AM 05:37PM
3
204
9.4
5.8
2.6
1.0
4.3 08:00AM 05:22PM
4
185
9.6
6.7
1.9
1.0
6.2 08:00AM 05:34PM
2
214
9.6
5.9
2.7
1.0
2.8 08:00AM 05:35PM
4
189
5.9
2.8
2.2
1.0
1.5 08:00AM 01:56PM
3
89
74.8 46.1 20.7
8.0 31.1
30
1476

Route
cost,$
602.45
220.25
608.59
416.05
599.64
552.96
625.79
563.48
311.65
4500.87

VEHICLE INFORMATION
Route Veh Weight Delvry Pickup Weight
Cube Delvry Pickup
no typ capcty weight weight
util capcty
cube
cube
1
1
1000
925
0 92.5%
9999
0
0
2
1
1000
625
0 62.5%
9999
0
0
3
1
1000
900
0 90.0%
9999
0
0
4
1
1000
950
0 95.0%
9999
0
0
5
1
1000
900
0 90.0%
9999
0
0
6
1
1000
950
0 95.0%
9999
0
0
7
1
1000
825
0 82.5%
9999
0
0
8
1
1000
1000
0 100.0%
9999
0
0
9
1
1000
850
0 85.0%
9999
0
0
Total
9000
7925
0 88.1% 89991
0
0

Cube
util
.0%
.0%
.0%
.0%
.0%
.0%
.0%
.0%
.0%
.0%

Vehicle
description
Truck #1
Truck #1
Truck #1
Truck #1
Truck #1
Truck #1
Truck #1
Truck #1
Truck #1

Substituting yard location A for the current yard location and solving for the route
design in ROUTER yields Figure 13-4. No routes can be put end-to-end so that one
truck can be used instead of two, so the minimum number of trucks remains at nine. The
total daily cost for this location is 3872.02 +9 x 200 + 480 = P6,152.092.
Yard Location A

FIGURE 13-4 Route Design for Yard Location A

182

*** SUMMARY REPORT ***


TIME/DISTANCE/COST INFORMATION

Route
no
1
2
3
4
5
6
7
8
9
Total

Route
Run Stop
Brk Stem
time, time, time, time, time,
Start Return No of
Route
hr
hr
hr
hr
hr
time
time stops dist,Mi
9.8
7.0
1.8
1.0
5.4 08:00AM 05:47PM
4
224
8.4
5.1
2.3
1.0
3.8 08:00AM 04:26PM
3
163
8.4
5.0
2.4
1.0
3.9 08:00AM 04:26PM
3
161
8.6
4.8
2.9
1.0
2.2 08:00AM 04:37PM
5
152
7.2
4.0
2.2
1.0
2.6 08:00AM 03:12PM
3
128
6.3
2.9
2.4
1.0
1.6 08:00AM 02:18PM
3
93
5.3
2.1
2.2
1.0
1.6 08:00AM 01:15PM
3
66
8.9
5.7
2.1
1.0
3.7 08:00AM 04:51PM
3
183
5.1
1.7
2.3
1.0
.6 08:00AM 01:03PM
3
55
68.0 38.3 20.7
9.0 25.5
30
1225

Route
cost,$
649.88
498.40
491.57
470.04
410.15
321.27
254.22
548.68
227.82
3872.02

VEHICLE INFORMATION
Route Veh Weight Delvry Pickup Weight
Cube Delvry Pickup
no typ capcty weight weight
util capcty
cube
cube
1
1
1000
475
0 47.5%
9999
0
0
2
1
1000
950
0 95.0%
9999
0
0
3
1
1000
1000
0 100.0%
9999
0
0
4
1
1000
975
0 97.5%
9999
0
0
5
1
1000
875
0 87.5%
9999
0
0
6
1
1000
1000
0 100.0%
9999
0
0
7
1
1000
875
0 87.5%
9999
0
0
8
1
1000
825
0 82.5%
9999
0
0
9
1
1000
950
0 95.0%
9999
0
0
Total
9000
7925
0 88.1% 89991
0
0

Cube
util
.0%
.0%
.0%
.0%
.0%
.0%
.0%
.0%
.0%
.0%

Vehicle
description
Truck #1
Truck #1
Truck #1
Truck #1
Truck #1
Truck #1
Truck #1
Truck #1
Truck #1

Continuing with location B, nine trucks are required and the total daily cost for the
route design in Figure 3 is 3370.42 + 9 x 200 + 450= P5,620.42.
Yard Location B

FIGURE 3 Design for Yard Location B

*** SUMMARY REPORT ***

183

TIME/DISTANCE/COST INFORMATION

Route
no
1
2
3
4
5
6
7
8
9
Total

Route
Run Stop
Brk Stem
time, time, time, time, time,
Start Return No of
Route
hr
hr
hr
hr
hr
time
time stops dist,Mi
7.1
3.7
2.4
1.0
2.3 08:00AM 03:06PM
4
120
6.5
3.1
2.3
1.0
1.8 08:00AM 02:27PM
3
100
7.9
4.6
2.4
1.0
3.0 08:00AM 03:56PM
4
146
5.9
2.8
2.2
1.0
1.4 08:00AM 01:56PM
3
89
5.9
2.6
2.3
1.0
1.1 08:00AM 01:55PM
3
84
6.7
3.6
2.1
1.0
2.3 08:00AM 02:41PM
3
114
5.8
2.5
2.3
1.0
2.1 08:00AM 01:50PM
3
80
6.1
3.3
1.9
1.0
2.8 08:00AM 02:07PM
2
104
9.7
5.8
2.8
1.0
3.0 08:00AM 05:40PM
5
187
61.7 32.0 20.7
9.0 19.8
30
1024

Route
cost,$
389.30
340.53
454.97
311.84
300.41
375.39
290.22
350.46
557.31
3370.42

VEHICLE INFORMATION
Route Veh Weight Delvry Pickup Weight
Cube Delvry Pickup
no typ capcty weight weight
util capcty
cube
cube
1
1
1000
825
0 82.5%
9999
0
0
2
1
1000
950
0 95.0%
9999
0
0
3
1
1000
825
0 82.5%
9999
0
0
4
1
1000
850
0 85.0%
9999
0
0
5
1
1000
925
0 92.5%
9999
0
0
6
1
1000
825
0 82.5%
9999
0
0
7
1
1000
950
0 95.0%
9999
0
0
8
1
1000
825
0 82.5%
9999
0
0
9
1
1000
950
0 95.0%
9999
0
0
Total
9000
7925
0 88.1% 89991
0
0

Cube
util
.0%
.0%
.0%
.0%
.0%
.0%
.0%
.0%
.0%
.0%

Vehicle
description
Truck #1
Truck #1
Truck #1
Truck #1
Truck #1
Truck #1
Truck #1
Truck #1
Truck #1

Finally, the route design from location C is shown in Figure 4. Although 10 routes
are in the design, two of these can be dovetailed so that only nine trucks are needed. The
total daily cost is 4479.99 + 9 x 200 + 420 = P6,699.99.
Yard Location C

FIGURE 4 Design for Yard Location C


*** SUMMARY REPORT ***

184

TIME/DISTANCE/COST INFORMATION

Route
no
1
2
3
4
5
6
7
8
9
10
Total

Route
Run Stop
Brk Stem
time, time, time, time, time,
Start Return No of
Route
hr
hr
hr
hr
hr
time
time stops dist,Mi
1.5
.6
.9
.0
.6 08:00AM 09:30AM
1
20
6.7
3.7
2.0
1.0
1.9 08:00AM 02:41PM
2
120
8.0
4.1
2.9
1.0
1.5 08:00AM 04:00PM
5
132
9.4
5.6
2.8
1.0
3.4 08:00AM 05:25PM
5
180
7.5
4.2
2.3
1.0
2.7 08:00AM 03:29PM
3
134
7.7
4.2
2.4
1.0
1.9 08:00AM 03:39PM
3
136
7.0
3.7
2.3
1.0
2.8 08:00AM 02:59PM
3
117
7.3
4.0
2.3
1.0
3.5 08:00AM 03:18PM
3
127
9.6
7.2
1.4
1.0
5.6 08:00AM 05:35PM
2
229
9.8
7.4
1.4
1.0
6.3 08:00AM 05:47PM
3
236
74.5 44.7 20.7
9.0 30.3
30
1432

Route
cost,$
140.91
389.21
420.20
540.48
425.69
429.10
383.04
408.53
663.40
679.42
4479.99

VEHICLE INFORMATION
Route Veh Weight Delvry Pickup Weight
Cube Delvry Pickup
no typ capcty weight weight
util capcty
cube
cube
1
1
1000
375
0 37.5%
9999
0
0
2
1
1000
875
0 87.5%
9999
0
0
3
1
1000
975
0 97.5%
9999
0
0
4
1
1000
925
0 92.5%
9999
0
0
5
1
1000
925
0 92.5%
9999
0
0
6
1
1000
1000
0 100.0%
9999
0
0
7
1
1000
950
0 95.0%
9999
0
0
8
1
1000
950
0 95.0%
9999
0
0
9
1
1000
550
0 55.0%
9999
0
0
10
1
1000
400
0 40.0%
9999
0
0
Total
10000
7925
0 79.3% 99990
0
0

Cube
util
.0%
.0%
.0%
.0%
.0%
.0%
.0%
.0%
.0%
.0%
.0%

Vehicle
description
Truck #1
Truck #1
Truck #1
Truck #1
Truck #1
Truck #1
Truck #1
Truck #1
Truck #1
Truck #1

From an economic analysis, it appears that yard location B is the best choice.

185

SUPERIOR MEDICAL EQUIPMENT COMPANY


Teaching Note
Strategy
The purpose of the Superior Medical Equipment Company case study is to encourage
students to apply the center-of-gravity methodology to a problem involving a single
warehouse location. Although the methodology is somewhat elementary, it can be useful
in providing some first approximations to good warehouse locations. It allows students
to evaluate the financial implications of alternative network designs, and it is intended to
be solved with the aid of the COG module provided in the LOGWARE software.
Answers to Questions
(1) Based on information for the current year, is Kansas City the best location for a
warehouse? If not, what are the coordinates for a better location? What cost
improvement can be expected from the new location?

When a single warehouse is to be located, the primary location costs are transportation,
both inbound to the warehouse and outbound from it, and the warehouse lease, which
varies with the location. The current location serves as a benchmark against which the
costs for other locations can be compared. That is, based on information given in the
case, the total relevant cost for the Kansas City location is:
Inbound transportation
Outbound transportation
Lease $2.75/sq. ft. 200,000 sq. ft.
Total relevant cost

2,162,535
4,819,569
550,000
7,532,104

Using the COG module with inbound transport rates from Phoenix set at $16.73/1163 =
$0.014/cwt./mile and from Monterrey set at $9.40/1188 = $0.008/cwt./mile, and the
outbound transport rate from the unknown warehouse location set at $0.0235/cwt./mile,
the coordinates for the best location are X = 7.61 and Y = 4.51, or approximately
Oklahoma City. Total transportation cost for this location would be $6,754,082. The
total relevant cost would be:
Transportation
Lease $3.25/sq. ft. 200,000 sq. ft.
Total

$ 6,754,082
650,000
$ 7,404,082

The annual cost savings can be projected as $7,532,104 7,404,082 = $128,022.


(2) In five years, management expects that Seattle, Los Angeles, and Denver markets to
grow by 5%, but the remaining markets to decline by 10%. Transport costs are expected
to be unchanged. Phoenix output will increase by 5%, and Monterrey's output will
decrease by 10%. Would your decision about the location of the warehouse change? If
so, how?

186

A new benchmark for the 5th year can be computed from the data given in Tables 1
and 2. After adjusting the plant and market volumes according to the changes indicated,
the 5th-year benchmark costs can be computed as follows.
Volume,

Rate,

Transport

Point
cwt.
1
64,575
2
108,540
3
17,850
4
33,600
5
13,125
6
8,550
7
26,550
8
18,900
9
37,170
10
7,740
11
9,630
Totals 346,230

$/cwt.
$16.73
9.40
33.69
30.43
25.75
18.32
25.24
19.66
26.52
26.17
27.98

cost, $
$1,080,340
1,020,276
601,367
1,022,448
337,969
156,636
670,122
371,574
985,748
202,556
269,447
$6,718,483

The total 5th-year benchmark relevant cost would be


Transportation
Lease $2.75/sq. ft. 200,000 sq. ft.
Total

$ 6,718,483
550,000
$ 7,268,483

Optimizing the location with the 5th-year data gives a location at X = 7.05 and Y =
4.52. The relevant costs for this location are
Transportation
Lease $3.25/sq. ft. 200,000 sq. ft.
Total

$ 6,464,206
650,000
$ 7,114,206

The annualized cost savings would be $7,268,483 7,114,206 = $154,277.


The average annual cost savings are (128,022 + 154,277)/2 = $141,150. The simple
annual return on investment (moving cost) can be computed as:
ROI

$141,150
100 471%
.
$300,000

Management must now judge whether 47.1% annual return is worth the risk of changing
warehouse locations.
(3) If by year 5 increases are expected of 25% in warehouse outbound transport rates
and 15% in warehouse inbound rates, would your decision change about the warehouse
location?

187

It is assumed here that the 5th-year demand level applies. A revised 5th-year benchmark
can be recomputed by applying the cost growth factors to the overall 5th-year transport
costs. That is,
Inbound 1.15 2,100,616 =

$2,415,708
Outbound 1.254,617,867 =

Subtotal

$8,188,042

Total

$8,738,042

Lease

5,772,334
550,000

Running COG shows that the minimum transport cost location would be at coordinates
X = 7.20 and Y = 4.62, which is near the previous location in question 2. The shift in
location is minimal. The cost for this location is:
Transportation
$
7,939,545
Lease $3.25/sq. ft. 200,000 sq. ft.
650,000
Total
$ 8,589,545
The annualized cost savings would be $8,738,042 8,589,545 = $148,497.
It can be concluded that:
1. Location is similar to the optimized 5th-year location.
2. The increase in possible cost savings further encourages relocation from Kansas City
and toward a site near Oklahoma City, OK.
(4) If the center-of-gravity method is used to analyze the data, what are its benefits and
limitations for locating a warehouse?
The center-of-gravity method locates a facility based on transportation costs alone. This
is reasonable when only one facility is being located and the general location for it is
being sought. Such costs as inventory carrying, production, and warehouse fixed are not
included, but they are not particularly relevant to the problem. However, costs such as
warehouse storage and handling, and other costs that vary by the particular site are not
included but may be relevant in a given situation. Transportation costs are assumed
linear with distance. This may not be strictly true, although distance may be nonlinear.
The obvious benefits of the method are (1) it is a fast solution methodology; (2) it
considers all possible locations (continuous); (3) it is simple to use; (4) its data are
readily available; and (5) it gives precise locations through a coordinate system. Some
potential limitations are (1) coordinates need to be linear; (2) transportation rates on a
per-mile basis are constant; (3) volumes are known and constant for given demand and
source points; and (4) locations may be suggested that are not feasible such as in lakes,
central cities, or restricted lands.
Concluding Comments

188

The analysis in the case seems to suggest a move from Kansas City to a region around
Oklahoma City would be advantageous. A return on investment of 47 percent or higher
is possible, however management must now seek a particular site in the area whose
choice may add or detract from this savings potential. In any case, the COG method has
assisted in the selection of good potential locations and testing their sensitivity to changes
in costs and volumes.
APPENDIX 1

LOGWARE COG Module Input Data for the Current Year

Title: SUPERIOR MEDICAL EQUIPMENT COMPANY


Power factor: .5
Scale factor: 230
Point X-coordinate Y-coordinate
Volume
1
3.60
3.90
61,500
2
6.90
1.00
120,600
3
0.90
9.10
17,000
4
1.95
4.20
32,000
5
5.60
6.10
12,500
6
7.80
3.60
9,500
7
10.20
6.90
29,500
8
11.30
3.95
21,000
9
14.00
6.55
41,300
10
12.70
7.80
8,600
11
14.30
8.25
10,700

Rate
0.0140
0.0080
0.0235
0.0235
0.0235
0.0235
0.0235
0.0235
0.0235
0.0235
0.0235

189

OHIO AUTO & DRIVER'S LICENSE BUREAUS


Teaching Note
Strategy
The purpose of this case study is to introduce students to a logistics problem in a service
area. It also provides an application for the multiple center of gravity location
methodology. The MULTICOG module in the LOGWARE software can effectively be
applied. The module allows students to quickly evaluate alternatives as to the number of
bureaus to use, the bureau locations, and the size of the territory that each bureau should
serve.
The case may be assigned as a homework problem, a short case study project, or as a
case for class discussion. The later would be appropriate especially if adequate attention
is given to the issues of how Dan should go about solving a problem such as this, what
data he needs and where to obtain it, and what concerns he should have about changing
the existing network design. This would encourage students to think beyond the
computational aspects of the problem.
Answers to Questions
(1) Do you think there is any benefit to changing the network of license bureaus in the
Cleveland area? If so, how should the network be configured?

The nature of the costs and the number of possible alternative network designs make it
impractical to seek an optimal solution. Therefore, a possible approach to the analysis is
outlined as follows.
First, establish a benchmark against which changes to the network can be compared.
Much of the data for this is given in the case write up. The costs can simply be applied to
the size of each bureau and its associated staff. The cost for residents traveling to the
bureaus is not known because the bureau territories are not known. However, an estimate
can be made of travel costs by solving the problem in MULTICOG for eight bureaus.
Since MULTICOG attempts to optimize bureau location, this travel cost is probably
understated. The benchmark costs are summarized in Table 1. The location costs for the
current operation are estimated to be $1,355,706.
Second, what improvements can be made on the existing eight locations? Besides
moving the locations of the bureaus, which results in resizing the facilities and adjusting
the staff numbers, there are no obvious improvements to be made. Therefore, the
benchmark remains the base for comparison.
Third, it is now necessary to estimate the approximate number of bureaus that are
needed to serve the area. Since the costs for a particular network design depend on the
size of each bureau, which cannot be known until the problem is solved, an initial
assumption must be made. It will be assumed that all bureaus are of the same size.
Hence, for 5 bureaus, the average number of residents in each bureaus territory would be
the total number of residents divided by the number of bureaus, or 691,700/5 = 138,340.
Rent, staff salaries, and utility expenses can be derived from this estimate. Table 2 is
developed to show the bureau size and the number of staff for 1 to 10 bureaus. Table 3
extends the average costs from these estimates. A reduced number of bureaus, in the
range of two, is about right.

190

TABLE 1

Bureau
1
2
3
4
5
6
7
8
Totals

Benchmark Costs for the Current Network of License Bureaus


Customer
Size,
Rent,
Travel,
Salaries, Utilities,
sq. ft.
Staff
$
$
$
$
1,700
4
37,400
84,000
6,800
1,200
4
26,400
84,000
4,800
2,000
5
44,000
105,000
8,000
1,800
4
39,600
84,000
7,200
1,500
4
33,000
84,000
6,000
2,200
5
48,400
105,000
8,800
2,700
5
59,400
105,000
10,800
5
33,000
105,000
6,000
1,500
14,600
36
321,200
756,000
58,400
220,106*

*Approximated by running MULTICOG at 8 bureaus.

TABLE 2 Average Size and Staff for Various Numbers of Bureaus


(1)
691,700/(1)
Avg.
Avg. no. of
bureau
No. of
residents per
size,
Staff per
bureaus
bureau
sq. ft.
bureau
1
691,700
4,500
10
2
345,850
3,000
7
3
230,567
2,500
6
4
172,925
2,000
5
5
138,340
2,000
5
6
115,283
2,000
5
7
98,814
1,500
4
8
86,463
1,500
4
9
78,855
1,500
4
10
69,170
1,500
4

191

TABLE 3 Average Costs by Number of Bureaus

No. of
bureaus
1
2
3
4
5
6
7
8
9
10

Rent,
$
99,000
132,000
165,000
176,000
220,000
264,000
231,000
264,000
297,000
330,000

Staff salaries,
$
210,000
294,000
378,000
420,000
525,000
630,000
588,000
672,000
756,000
840,000

Utilities,
$
18,000
24,000
30,000
32,000
40,000
48,000
42,000
48,000
54,000
60,000

Resident
travel, $
662,319
430,922
354,239
298,000
278,181
249,287
237,635
220,106
206,496
198,600

Annual total
cost, $
989,319
880,922
927,239
926,965
1,063,181
1,191,287
1,098,636
1,204,106
1,313,496
1,428,600

The cost estimates can now be refined around two bureaus. A comparison with the
benchmark costs and a return of the initial investment (costs related to changing the
network design) are sought. A sample analysis for two bureaus, based on a design
provided by MULTICOG, is shown below.

Bureau
1
2
Totals

Residents
290,200
401,500
691,700

Size,
sq. ft.
2,500
3,500
6,000

Staff
6
8
14

Rent,
$
55,000
77,000
132,000

Staff
salaries, Utilities,
$
$
126,000
10,000
168,000
14,000
294,000
24,000

Resident
travel cost,
$
166,332
264,590
430,922

The total annual variable cost is $132,000 + 294,000 + 24,000 + 430,922 = $880,922.
There are one-time costs due to staff separation and equipment moves. Compared
with the benchmark, 36 14 = 22 staff members will be separated for a cost of 22
$8,000 = $176,000. Equipment movement costs to two bureaus would be 2 10,000 =
$20,000. Total movement costs would be $176,000 + 20,000 = $196,000.
Annual variable cost savings compared with the benchmark would be $1,355,706
880,922 = $474,784. A simple return on investment would be:
ROI

$474,784
100 242%
$196,000

Similar calculations are carried out for various numbers of bureaus. These results are
tabulated in Table 4.

192

TABLE 4 Cost Savings and Return of Investment for Alternate Network Designs
as Produced by MULTICOG
Total
Annual
Return on
No. of
size, sq.
Total
variable cost,
Moving
investSavings,
bureaus
ft.
staff
$
cost, $
ment, %
$
1
4,500
10
989,319
218,000
366,387
168
2
6,000
14
880,922
196,000
242
474,784
3
7,500
18
927,239
176,000
428,467
243
4
8,500
21
960,965
160,000
394,741
246
5
9,500
24
1,029,181
146,000
326,525
223
6
11,500
29
1,157,287
106,000
198,419
187
7
12,000
31
1,200,635
110,000
155,071
141
8
13,000
34
1,272,106
96,000
83,600
87
Benchmark 14,600
36
1,355,706
---------

The maximal annual savings occurs with a network containing two bureaus.
However, the maximal return on investment occurs with four bureaus. ROI is selected as
the appropriate measure on which to base this economic decision. The details for a
design with four bureaus are given in Table 5. The design is shown pictorially in Figure
1 of this note.
TABLE 5 Design Details for a Network with Four Bureaus
Column
grid
Row grid
coordicoordiBureau
nate
nate
Residents Grid box number assignment
1
3.36
2.74
218,200
1,2,3,4,5,6,7,8,9,10,11,12,13,
14,15,16,17,18,19,20,21,22,23,
24,25,26,27,28,29,30,31,32,33
2
7.00
3.00
168,700
34,35,36,37,38,39,40,41,43,44,
45,46,47,50,51,52,53
3
9.74
5.66
195,000
42,48,49,54,55,56,60,61,62,63,
67,68,69,70,74,75,76,77,81,82, 83,84,
4
10.00
2.00
109,800
57,58,59,64,65,66,71,72,73,78, 79,80

(2) Do you think Dan Roger's study approach is sound?


Overall Dan can be praised for the simplicity of the methodology that he has chosen. The
center-of-gravity approach is appropriate in this problem since there are no capacity
limitations on the facilities, locations across a continuous space are desired, and
transportation cost (except for some fixed costs) is the primary location variable. The
data requirements are relatively straightforward, although they are not always easy to
fulfill. This would likely be a problem with any other solution approach as well.
The results of this methodology can only be used as a first approximation at best.
Several criticisms of this particular approach can be offered as follows:

193

The effect of bureau location on the resident's perception of service is not as well
known as portrayed in the case. In addition, service may need to be represented by
more than location.
Travel to the bureaus is assumed straight line. However, location in the area is
likely to be influenced by a road network. Time may be more important than
distance to residents.
The fixed costs associated with location are not handled directly by the center-ofgravity approach.
Residents are assumed to travel to the locations within their assigned territories.
They may not strictly do this.
Good facilities may not be available at the indicated location coordinates.
The analysis is particularly weak around the estimate of the resident travel cost.
While an exact cost is not likely to be known, Dan should conduct a sensitivity analysis
around this cost. He may find that the design does not change a great deal over a wide
range of assumed values. If this is the case, he can feel comfortable that his
recommendation is fundamentally sound. If not, he should seek to find a more precise
value.
(3) What concerns besides economic ones should Dan have before suggesting that any
changes be made to the network?
A quantitative approach to location will rarely give the precise locations to be
implemented. Rather, it provides a starting point for further analysis. There are a
number of other factors to be considered before the revised network design can be
implemented. First, there are site selection factors to be taken into account such as the
availability of adequate space near the location coordinates, proximity to good highway
linkages, and reasonable neighborhood reactions to this type of operation.
Second, there are political concerns. Reducing the number of locations will result in
a releasing some of the staff. Dan may experience some political resistance to this.
Since staff is a large expense in the operation, currently about 2/3 of the costs, retention
of a larger number of bureaus may be required. Of course, transferring staff to other
governmental operations may be a way of dealing with this issue. This assumes their
willingness to be relocated, although this is not likely to be a strong issue if relocation
were to occur in the same area.
Third, there may be difficulty in demonstrating the economics of network redesign.
Although others may appreciate the costs of rent, salaries, and utilities, the cost of
resident travel is subject to much interpretation. Those favoring many bureaus may argue
the high cost while those wanting to reduce the number of bureaus may perceive it as not
very significant.

194

Grid row number

Current bureau locations

Grid column number

Revised bureau locations

FIGURE 1 Four Bureau Locations and Their Territories

SUPPLEMENT Sample Input Data File for MULTICOG in LOGWARE for the
Ohio Auto & Driver's License Bureau Case Study
Title: LICENSE BUREAU
Number of sources: 4
Number of demand points: 84
Scaling factor: 2.5
POINT
X-COORDINATE
Y-COORDINATE
1
1
1
2
1
2
3
1
3
4
1
4
5
1
5
6
1
6
7
1
7
8
2
1
9
2
2
10
2
3
11
2
4
12
2
5
13
2
6
14
2
7
15
3
1
16
3
2
17
3
3
18
3
4
19
3
5
20
3
6

VOLUME
4100
6200
7200
10300
200
0
0
7800
8700
9400
11800
100
0
0
8100
10500
15600
10500
200
0

RATE
.12
.12
.12
.12
.12
.12
.12
.12
.12
.12
.12
.12
.12
.12
.12
.12
.12
.12
.12
.12

195

21

.12

SUPPLEMENT (Continued)
POINT
22
23
24
25
26
27
28
29
30
31
32
33
34
35
36
37
38
39
40
41
42
43
44
45
46
47
48
49
50
51
52
53
54
55
56
57
58
59
60
61
62
63
64
65
66
67
68
69
70
71
72
73
74
75
76
77
78

X-COORDINATE
4
4
4
4
4
4
4
5
5
5
5
5
5
5
6
6
6
6
6
6
6
7
7
7
7
7
7
7
8
8
8
8
8
8
8
9
9
9
9
9
9
9
10
10
10
10
10
10
10
11
11
11
11
11
11
11
12

Y-COORDINATE
1
2
3
4
5
6
7
1
2
3
4
5
6
7
1
2
3
4
5
6
7
1
2
3
4
5
6
7
1
2
3
4
5
6
7
1
2
3
4
5
6
7
1
2
3
4
5
6
7
1
2
3
4
5
6
7
1

VOLUME
10700
12800
13800
15600
400
0
0
11500
13900
14500
13700
600
0
0
9300
14900
13700
10200
1200
0
0
10100
12600
16700
15800
12400
2600
0
8800
13700
15200
14100
10800
17200
500
5300
16700
13800
11900
13500
18600
12000
5100
17400
10300
9800
10300
15500
11700
7700
9200
7500
8500
7800
9900
8700
4300

RATE
.12
.12
.12
.12
.12
.12
.12
.12
.12
.12
.12
.12
.12
.12
.12
.12
.12
.12
.12
.12
.12
.12
.12
.12
.12
.12
.12
.12
.12
.12
.12
.12
.12
.12
.12
.12
.12
.12
.12
.12
.12
.12
.12
.12
.12
.12
.12
.12
.12
.12
.12
.12
.12
.12
.12
.12
.12

196

79
80

12
12

2
3

6700
5800

.12
.12

Y-COORDINATE
4
5
6
7

VOLUME
6800
5400
7100
6400

RATE
.12
.12
.12
.12

SUPPLEMENT (Continued)
POINT
81
82
83
84

X-COORDINATE
12
12
12
12

197

SOUTHERN BREWERY
Teaching Note
Strategy
The purpose of this case study is to provide students with the opportunity to design a
distribution network where plant location is at issue. They first should identify the major
costs and alternatives that are important to such a design problem. Second, they should
be encouraged to apply the transportation method of linear programming to assist in the
analysis of alternatives using the TRANLP module in LOGWARE. Finally, they should
consider factors other than those in the analysis that might alter the course of their
recommendation and be sensitive to the limitations and benefits of linear programming as
a solution methodology.
Answers to Questions
(1) If you were Carolyn Carter, would you agree with the proposal to build the new
brewery? If you do, what plan for distribution would you suggest?

If growth is uniform over the next five years, Southern can expect that demand for its
products will exceed the currently available plant capacity. That is, demand is increasing
at the rate of (0 + (595,000 403,000)) 5 = 38,400 barrels per year. Thus, the current
annual capacity of 500,000 barrels will be used up in (500,000 403,000) 38,400 = 2.5
years. A major concern is whether it would be profitable to construct the new plant.
Rough estimates of its profitability can be made, projecting profits with and without the
new plant. We know that 595,000 500,000 = 95,000 barrels of beer would not be sold
annually in the 5th year if additional capacity is not constructed. This represents a
potential average lost revenue of $280/barrel 0.20 [(0 + 95,000) 2.5] = $2,128,000.
(The figure of 2.5 years assumes that the new brewery can be brought on stream at
approximately the time when capacity will be used up in the existing plants.) From the
benchmark1 costs for the current system, as shown in Table 1, Southern is currently
producing and distributing 403,000 barrels at a total cost of $60,015,000. This is an
average cost per barrel of $60,015,000 403,000 = $148.92. The overhead and sales
expense is 27 percent, or $280 0.27 = $75.60 per barrel. Total costs per barrel are
$148.92 + 75.60 = $224.52, which is about 80 percent of the sales dollar. The 20 percent
profit margin seems valid. Therefore, the benefit of serving the potentially lost demand
with a new brewery can be estimated using on a simple return on investment:
ROI

$2,128,000
0.21, or 21%
$10,000,000

If management feels that this is an adequate return for such a project, Carolyn should
proceed with her analysis. Let's assume that she has this approval.
Next, she may wish to explore the opportunities available by improving upon the
existing distribution system without the presence of the new plant. This is an improved
1

A benchmark refers to the costs of producing and distributing demand as currently allocated throughout
the network.

198

benchmark2 and it can be found by solving a transportation-type linear programming


problem of the type shown in Table 2. The results given in the TRANLP module of
LOGWARE can be summarized in Table 3. This shows that with some slight
reallocation of demand among the plants, production and distribution costs can be
reduced by $60,015,000 59,804,000 = $211,000 annually. A comparison of the
benchmark results in Table 1 and the improved benchmark results in Table 3 shows that
Knoxville's demand should be shifted from Columbia to Montgomery and 28,000 barrels
of Columbia's market should be shifted from the Columbia plant to the Montgomery
plant. The Columbia plant is a high-cost producer and cost savings are achieved by
trading production costs for transportation costs. That is, production costs will be
reduced by $400,000 while transportation costs will be increased by $189,000. Note that
this $211,000 savings can be realized without any capital investment.
TABLE 1 Benchmark of Production and Transportation Costs ($000s) for Current
Demand
Demand
TransBrewery of
in 000s
Producport
Market area
origin
barrels tion costs
costs
Total costs
1 Richmond
Richmond
56
$7,840
$475
$8,315
2 Raleigh
Richmond
31
4,340
332
4,672
3 Knoxville
Columbia
22
3,190
282
3,472
4 Columbia
Columbia
44
6,380
306
6,686
5 Atlanta
Montgomery
94
12,878
959
13,837
6 Savannah
Montgomery
13
1,781
179
1,960
7 Montgomery
Montgomery
79
10,823
550
11,373
8 Tallahassee
Montgomery
26
3,562
355
3,917
5,206
577
5,783
9 Jacksonville
Montgomery
38
Total
403
$56,000 $4,015
$60,015
TABLE 2 TRANLP Problem Setup
RICH-

RA-

KNOX-

COL-

AT-

SAVAN-

MONT-

TALL

JACK-

From\To

MOND

LEIGH

VILLE

UMBIA

LANTA

NAH

GOMRY

AHAS

SONVL

RICHMD

148.49

150.70

156.38

152.54

155.48

154.64

159.98

164.30

158.84

100

COLMBA

157.54

154.78

157.81

151.96

156.85

154.54

157.93

160.18

157.27

100

MONTGM

156.98

153.35

150.80

149.93

147.20

150.80

143.69

150.65

152.18

300

JACKVL

152.13

149.25

150.48

146.16

148.80

144.54

148.80

144.72

142.68

Demand

56

31

22

44

94

13

79

26

38

Supply

An improved benchmark refers to a reallocation of current demand in an optimal way, respecting plant
capacity restrictions.

199

TABLE 3 Improved Benchmark of Production and Transportation Costs ($000s)


for Current Demand
Demand
TransBrewery of
in 000s
Producport
Market area
origin
barrels tion costs
costs
Total costs
1 Richmond
Richmond
56
$7,840
$475
$8,315
2 Raleigh
Richmond
31
4,340
332
4,672
3 Knoxville
Montgomery
22
3,014
304
3,318
4 Columbia
Columbia
16
2,320
111
2,431
4 Columbia
Montgomery
28
3,836
362
4,198
5 Atlanta
Montgomery
94
12,878
959
13,837
6 Savannah
Montgomery
13
1,781
179
1,960
7 Montgomery
Montgomery
79
10,823
550
11,373
8 Tallahassee
Montgomery
26
3,562
355
3,917
5,206
577
5,783
9 Jacksonville
Montgomery
38
Total
403
$55,600 $4,204
$59,804

Adding a plant at Jacksonville with a capacity of 100,000 barrels per year provides
enough capacity to satisfy demand out to the 5th year. If the new plant were constructed
and producing immediately, total costs could be reduced from the improved benchmark
by $59,804,000 59,090,000 = $714,000 per year. (Compare the total costs in Tables 3
and 4.) The Columbia plant would not be needed if the lower-cost Jacksonville plant
were on line.
We do not know the savings for the 5th-year demand level since not all demand can
be served without the presence of the new plant. Therefore, a future-year benchmark
cannot be determined. However, we do know how the new plant should be utilized
within the system (see Table 5) and how demand allocation should be adjusted to
accommodate it. Also, note that the Columbia plant is needed once again although its
capacity is not required until the last one-half year of the 5-year planning horizon. This
suggests that the Columbia plant should not be sold, but perhaps some alternate use could
be made of the facility in the interim, such as subcontracting beer production to a noncompeting company.
The new plant is not likely to be brought on stream immediately nor is it needed for
2.5 years, so Carolyn might suggest a distribution plan similar to that in Table 5. A
careful inspection of this plan shows that only 1 barrel of demand in the Knoxville region
is assigned to Richmond. Splitting demand to this extent is probably not practical and
can be assigned to Columbia where there is excess capacity. Costs will rise only slightly.
An interesting question is whether the Columbia plant, through modernization, could
be made as efficient as the new brewery, and what the implications for distribution might
be. We know that this could potentially save $145 135 = $10 per barrel in production
costs. At a 100,000-barrel capacity, this is $1,000,000 in cost savings. If the
modernization were to cost no more than $5,000,000, this option might be attractive. Of
course, we would need to resolve the linear programming problem with Columbia's per
barrel costs at $135 plus transportation costs. This would tell us how and to what extent
demand would be allocated to Columbia and give a more accurate basis for determining

200

the cost savings. Similarly, it would be interesting to explore what it means to expand
the capacity of an existing brewery at a lower investment cost per barrel than the
construction of a new facility.
TABLE 4 Production and Transportation Costs ($000s) for Current Demand with
the Jacksonville Plant
Demand
TransBrewery of
in 000s
Producport
Market area
origin
barrels tion costs
costs
Total costs
1 Richmond
Richmond
56
$7,840
$475
$8,315
2 Raleigh
Richmond
31
4,340
332
4,672
3 Knoxville
Montgomery
22
3,014
304
3,318
4 Columbia
Montgomery
21
2,877
272
3,149
4 Columbia
Jacksonville
23
3,105
257
3,362
5 Atlanta
Montgomery
94
12,878
959
13,837
6 Savannah
Jacksonville
13
1,755
124
1,879
7 Montgomery
Montgomery
79
10,823
550
11,373
8 Tallahassee
Jacksonville
26
3,510
253
3,763
5,130
292
5,422
9 Jacksonville
Jacksonville
38
Total
403
$55,272
$3,818
$59,090

(2) If the new brewery is not to be constructed, what distribution plan would you propose
to top management?
Table 6 shows a linear programming solution where the new plant is not brought on
stream and the demand in the markets is set at the 5-year level. An interesting solution
occurs when demand exceeds capacity. The most costly demand region to serve is not
assigned to any plant. As can be seen in Table 6, portions of the demand in Tallahassee
and Jacksonville should not be served, and essentially the entire Knoxville market should
not be served at all. Top management may wish to adjust this plan for reasons other than
economic ones.

201

TABLE 5 Production and Transportation Costs ($000s) for Projected 5th-Year


Demand With the Jacksonville Plant
Demand
TransBrewery of
in 000s
Producport
Market area
origin
barrels tion costs
costs
Total costs
1 Richmond
Richmond
64
$8,960
$543
$9,503
2 Raleigh
Richmond
35
4,900
375
5,275
3 Knoxville
Richmond
1
140
16
156
3 Knoxville
Columbia
20
2,900
256
3,156
3 Knoxville
Montgomery
12
1,644
166
1,810
4 Columbia
Columbia
55
7,975
383
8,358
5 Atlanta
Montgomery
141
19,317
1,438
20,755
6 Savannah
Columbia
20
2,900
191
3,091
7 Montgomery
Montgomery
119
16,303
828
17,131
8 Tallahassee
Montgomery
28
3,836
382
4,218
8 Tallahassee
Jacksonville
24
3,240
233
3,473
10,260
584
10,844
9 Jacksonville
Jacksonville
76
Total
595
$82,375
$5,395
$87,770

TABLE 6 Production and Transportation Costs ($000s) for Projected 5thYear Demand Without the Jacksonville Plant
Expected
Served
demand in demand in
Brewery of
000s
000s
Market area
origin
barrels
barrels
Total costs
1 Richmond
Richmond
64
64
$9,503
2 Raleigh
Richmond
35
35
5,275
3 Knoxville
Richmond
33
1*
156
4 Columbia
Columbia
55
55
8,358
5 Atlanta
Montgomery
141
141
20,755
6 Savannah
Columbia
20
20
3,091
7 Montgomery
Montgomery
119
119
17,131
8 Tallahassee
Montgomery
52
40*
6,026
25*
3,931
9 Jacksonville
Columbia
76
Total
595
500
$74,226
*Indicates market demand is not fully served due to inadequate plant capacity.

(3) What additional considerations should be taken into account before reaching a final
decision?
A number of assumptions have been implied in the analysis shown above. For example,
Demand has been assumed to grow at a constant rate in the markets.
Production is assumed limited to exactly the values given without the possibility for
expansion through overtime, additional shifts, or subcontracting.
202

Per-unit production and transportation costs are assumed to remain unchanged with
the reallocation of demand throughout the network.
Customer service effects are not considered in reallocation of demand.
There is no change in per-unit costs throughout the 5-year planning horizon.
This case might end with a discussion of the appropriateness of using linear
programming as a vehicle for analysis in a problem such as this. Mentioning that linear
programming does not consider such factors as fixed costs, return on investment, or the
many subjective factors (top management's intuition about location, vested interests, etc.)
that are typically a part of such problems means that linear programming, at best, is a
facilitating vehicle for analysis. It does not provide the final answer.

203

CHAPTER 14
THE LOGISTICS PLANNING PROCESS
3
The MILES module within the LOGWARE software is used to solve this problem. It
computes distance based on the great circle distance formula using longitude and latitude.

(a) The estimated road distance is 1,380 miles.


(b) The estimated road distance is 830 miles.
(c) Since both latitudes are in the same hemisphere, no adjustments need to be made.
The estimated distance is 244 miles, or 2441.61 = 393 km.
(d) In this case, one point is east and the other west of the Greenwich line. Therefore, we
need to set a sign convention. Let's set west longitudes as + and east longitudes as .
Thus, 2.20o E longitude is entered into MILES as 2.20 o. The estimated distance is
250 miles, or 2501.61 = 402.5 km.
4
Suppose that a certain linear grid coordinate system has been overlaid on a map of the
United States. The grid numbers are calibrated in miles, and there is a road circuity
factor of 1.21. Find the expected road distances between the following pairs of points:
Equation 14-1 in the text is used to approximate distances from linear coordinates.
The K factor in the equation is set at 1.21.

(a) Lansing, MI to Lubbock, TX


Location
a. From
To
b. From
To
c. From
To
d. From
To

X Coordinate
Lansing, MI
Lubbock, TX
El Paso, TX
Atlanta, GA
Boston, MA
Los Angeles, CA
Seattle, WA
Portland, OR

Y Coordinate
924.3
1488.6
1696.3
624.9
374.7
2365.4
2668.8
2674.2

1675.2
2579.4
2769.3
2318.7
1326.6
2763.9
1900.8
2039.7

D 121
.
(924.3 1,488.6) 2 (1,675.2 2,579.4) 2 1,290 miles
(b) El Paso, TX to Atlanta, GA
D 121
.
(1,696.3 624.9) 2 ( 2,769.3 2,318.7) 2 1,406 miles
(c) Boston, MA to Los Angeles, CA

204

D 121
.
(374.7 2,365.4) 2 (1,326.6 2,7639
. )2 2,971 miles
(d) Seattle, WA to Portland, OR
D 121
.
( 2,668.8 2,674.2) 2 (1,900.8 2,039.7) 2 168 miles
5
The plot of the truck class rates is shown in Figure 14-1. The rates show a high degree of
linearity. A linear regression was found with aid of the MULREG module in
LOGWARE. The rate equation was determined to be:

R = 5.1745 + 0.0041D
The standard error of the estimate SE is 0.9766
The coefficient of determination r2 is 0.928
The best single estimate of the rate at 500 miles is
R = 5.1745 + 0.0041500
= $7.23/cwt.
Assuming the error around the regression line is normally distributed, a 95% confidence
band would give a range for the actual rate. That is,
Y = R 1.96SE
= 7.23 1.914
where 1.96 is the normal deviate for the normal distribution representing 95% of the area
in a two-tailed distribution. The range of the estimate is:
$5.32/cwt. Y $9.14/cwt.
The r2 value of 0.928 indicates that a linear rate equation explains about 93% of the
variation in the data with distance. Such a simple relationship seems to represent the
rates quite well.

205

20
18

Class rate, $/cwt.

16
14
12
10

Estimating line

8
6
4
2
0
0

500

1000

1500

2000

2500

3000

3500

Distance, miles

FIGURE 14-1 Plot of Truck Class Rates


6
A plot of the average inventory level versus warehouse throughput is shown in Figure 142. The multiple regression software in LOGWARE was used to test two equation forms.
The first was of the form

I aTP b
and the other was of the form

I a bTP
Both forms showed high r2 values, with the exponential form being slightly higher at
0.9406. It was selected as the equation form to use. This equation was:

I 0.704 TP 0.83
where TP and I are both expressed in thousands of dollars. We can now estimate that for
an annual warehouse throughput of $50,000,000, the average inventory would be:
I 0.704 50,000 0.83
5,593939
. , or $5,593,939

Warehouse 22 has a much higher inventory turnover ratio than the average of the
other warehouses. This would suggest that the inventory control procedures might be
different from the others. One reason might be that person in control of the inventory in
this warehouse attempts to keep inventories at low level, demand may be high such that
the inventory level has been restored to a normal level, or lead times have been extended
to the point where replenishment has been delayed. The reason should be investigated.
206

Average inventory level, $(Millions)

This type of inventory-throughput relationship is very useful in network planning,


especially warehouse location, to estimate how inventory levels will change when sales
are reallocated to a varying number of warehouses.

12
10
8
6
4

Estimating line

2
0
0

20

40

60

80

100

Annual warehouse thruput, $(Millions)

FIGURE 14-2 Plot of Inventory Levels and Warehouse Throughput for


California Fruit Growers Association

207

USEMORE SOAP COMPANY


Teaching Note

The purpose of this case study is to provide students with the opportunity to evaluate and
design a large-scale production-distribution network using real data and cost
relationships. To assist in the substantial amount of computational effort in this problem,
an interactive computer program (WARELOCA) is available in the LOGWARE
collection of software modules.
Major Issues
The text of the case suggests a number of questions that are critical to productiondistribution network design. These reduce to three major issues, namely:

(1) Should plant capacity be added and, if so, when and where?
(2) How many warehouses are optimal and where should they be located?
(3) Should the current customer service level be retained?
Although no change can be made in the network without potentially affecting other
variables, the attempt here will be to treat these questions sequentially to converge on a
good network design.
Numerous computer runs were made to provide the basic information needed in the
analysis. The more meaningful runs are summarized in Appendix A to this note. Tables
1 and 2 compare selected runs for both the current-year and the future-year time periods.
This information is used throughout the analysis of the major issues.
The Plant Expansion Issue
An attempt to meet 5-year growth goals using current plant capacity will cause the
system having a total capacity of 1,630,000 cwt. to be out of capacity in 1.7 years. That
is,

5th-year demand
Current demand
Net increase

1,908,606 cwt.
1,477,026
431,580 cwt.

Therefore, the average annual growth rate is 431,580/5 = 86,316 cwt. So, in (1,630,000
1,477,026)/86,316 = 1.7 years all available capacity will be depleted.
If no expansion of plant capacity occurs, then 1,908,606 1,630,000 = 278,606 cwt.
will potentially be lost by the 5th year. Sales are $100 million on 1.477 million cwt. in
volume for a product value of $67.7/cwt. With a profit margin of 20%, the profit per cwt.
would be 20%$67.7/cwt, or $20/1.477, = $13. Thus, 278,60613 = $3.16 million in lost
sales. The weighted profit loss over the five-year period would be:
2/5 (0) + (3/5) ([0 + 3.6])/2) = $1.08m/yr.

208

TABLE 1 Current-Year Comparison of Network Alternatives ($000s)


Cost type

Benchmark

Production
$30,762
Warehouse operations
1,578
Order processing
369
Inventory carrying
457
Transportation
Inbound
2,050
Outbound
6,896
Total costs
$42,112

Improved
benchmark

Optimum
number
of whses

Optimum
number
of whses

Relaxed
service (1)

Relaxed
service (2)

Maximum
opportunity

$30,678
1,468
354
431

$30,673
1,608
370
508

$30,675
1,572
358
490

$30,678
1,296
349
390

$30,673
1,420
354
445

$30,386
1,529
358
500

1,802
6,991
$41,725

1,976
6,310
$41,447

1,860
6,365
$41,321

1,249
7,238
$41,201

1,178
6,698
$41,043

1,178
6,458
$40,409

Customer
service:
300 mi.
600 mi.

93%
98%

93%
98%

98%
100%

92%
100%

75%
98%

88%
100%

81%
94%

No. of stocking
points

22

21

31

30

19

26

40

No. of plants

Savings vs.
benchmark

$0

$387

$665

$791

$911

$1,069

$1,703

$0

$278

$404

$524

$1,316

Service
to match
benchmark

600 mi constraint on
current
warehouses

600 mi constraint on
opt no. of
warehouses

Savings vs. improved


benchmark
$0
Comments:

682

Unlimited
service,
whses, and
plant cap.

209

TABLE 2 Future-Year Comparison of Alternatives ($000s)


No plant
expansion

Add plant
@ Memphis

Add plant
@ Memphis
& Chicago

Memphis
and opt no.
of whses

Memphis
and opt no.
of whses

$33,965
1,496
393
431

$39,517
1,842
462
505

$39,548
1,847
454
497

$39,524
2,028
470
591

$39,522
1,976
460
573

1,647
7,230
$45,164

2,350
9,030
$53,705

2,000
9,036
$53,382

2,614
8,117
$53,342

2,426
8,222
$53,179

Customer
service:
300 mi
600 mi

98%
99%

94%
98%

95%
98%

98%
100%

92%
100%

No. of stocking
points

20

21

20

31

30

No. of plants

Comments:

Not all
demand
met

Cost type
Production
Warehouse operations
Order processing
Inventory carrying
Transportation
Inbound
Outbound
Total costs

Service
at benchmark

210

Based on a simple rate of return on investment, capturing this profit potential would yield
1.08/4 = 27% annually on a $4,000,000 investment for expansion. The return would
increase to 90% per year with the full loss in the 5th year. The potential seems great
enough to justify one unit of expansion (1,000,000 cwt.). Two units of expansion
probably cannot be justified, since adequate capacity would be available from the first
capacity unit to meet demand requirements. The only benefit would be from the network
design improvement. The savings would be about $323,000 per year in the 5th year (see
Table 2) comparing one additional plant with two additional plants and keeping the
current number of warehouses. The simple return on investment using 5th-year savings
would only amount to about 8% (323,000100/4,000,000 = 8.1%).
The next question is: Where should the expansion take place at an existing plant
or at one of the two proposed locations? From a test of expanding any of the four
existing plants or the two proposed plant locations (runs 10 through 16 in Appendix A of
this note), it would appear that Memphis would be the lowest cost site in the 5th year
with Chicago next at only an additional cost of $76,000 per year (compare runs 14 and 15
in Appendix A). Adding a plant at a new location rather than expanding an existing plant
site saves a minimum of $281,000 annually (compare runs 11 and 14 in Appendix A of
this note), which results from placing plant capacity closer to warehouses.
Selecting Warehouses
A simple test on the number of warehouses in the network shows that transportation costs
are dropping more rapidly than inventory related costs are increasing (see Figure 1). This
means that 40 active warehouses will have the lowest total cost. However, some of these
warehouses will have low throughput. In order to maintain a minimum replenishment
frequency and shipment size, a minimum throughput needs to be met. Approximately a
truckload every two weeks, or 10,400 cwt. of throughput per year, is the minimum
activity needed to open a warehouse. Therefore, any warehouse showing less than this
throughput will be eliminated from consideration.
Under various assumptions about plants and their capacities, demand growth, and
service levels, 30 to 31 warehouses seem most economical with no deterioration on
service over the benchmark network. The following table shows selected results.

211

Type of
run
Benchmark
Improved
benchmark
Improved
benchmark
Current
yr. whses
5th yr.
whses

Percent
of demand
300 mi.

Year

Plant
capacities

Total
cost

No. of
whses

Current

Current

93

$42,112

22

Current

93

41,725

21

5th yr.

Current
Current
+ Memphis

94

53,705

31

Current
Current
+ Memphis

92

41,321

30

5th
year

92

53,179

30

Note that this conclusion about the number of warehouses depends on the previous
conclusion that a Memphis plant should be added by the 5th year. The number of
warehouses should be increased from the present 22 in both the current year and the 5th
year.
42

100
99

41.8

98
97

41.6

% of demand < 300 mi.

Total cost, $(000,000s)

Service (right scale)

96
95

41.4

94
Cost (left scale)

93
41.2

92

Practical design

91
41

90
22

26

30

31

36

40

Number of warehouses

FIGURE 1 Cost and Customer Service Profiles for Alternative Network Designs

More detailed economic analysis shows that if the plants are held at current
throughput levels, a savings realized from 30 warehouses would be $41,725,000
41,321,000 = $404,000 (see previous table). If current plant capacities are used and the
Memphis plant is on-stream in year 5, the savings of the added warehouse would be:
$53,705,000 53,179,000 = $526,000

212

On the average, there can be savings of approximately ($404,000 + 526,000)/2 =


$465,000 per year by increasing the number of warehouses to 30 from the current 22.
Since these are public warehouses, little or no investment would be required to
implement the change.
Although the number of warehouses remains relatively unchanged from the current
year to the 5th year, there is some shifting among the particular warehouses in the mix.
The 30 warehouses in the current year should be numbers:
1,2,3,4,5,7,8,11,13,14,15,16,17,18,19,20,21,25,28,31,32,33,34,35,36,37,38,40,44,45
providing that the loading on the current plants is allowed up to the limits of their current
capacity. When the Memphis plant is brought on-stream by the end of the second year,
the warehouse mix should begin to evolve to numbers:
1,2,3,4,5,7,8,11,13,14,15,17,18,19,20,21,25,28,29,31,32,34,35,36,37,38,40,44,45,47
As the Memphis plant is bought on stream, the Memphis public warehouse is closed
and the volume is shifted to the Memphis plant as a warehouse. In addition, the
Richmond, VA warehouse is closed and the Las Vegas, NV warehouse is opened. The
number of warehouses remains at 30.
Both in the base year and in the future year, the throughputs in the plants serving as
warehouses are within acceptable limits as the following summary shows.
Plant
as a
warehouse

Thruput
limits

Current
year
solution

Future
year
solution

Covington
New York
Arlington
Long Beach

450,000 cwt.
380,000
140,000
180,000

254,471 cwt.
302,043
66,592
95,943

306,478 cwt.
380,523
66,161
117,288

Customer Service
Currently, a high proportion of demand (93%) is located within 300 miles of a stocking
point. Since the service distance may be up to 600 miles and still meet the company's
service policy, should the service level be reduced somewhat to effect a cost saving? For
example, using the improved benchmark as the base case (run 2), 93% of the demand is
within 300 miles and 97.5% is within 600 miles. If a 600-mile constraint is applied to the
current network configuration (run 23), 75% of the demand is within 300 miles and 98%
is still within 600 miles. The total costs are reduced from $41,725,000 to $41,201,000, or
a savings of $524,000 per year. In addition, if the number of warehouses in the network
is optimized, the costs can be reduced by another $158,000 per year (run 23 vs. run 22).
However, $278,000 of the total $524,000 + 158,000 = $628,000 can be realized without a
service change. This leaves approximately $404,000 that can be saved by a relaxed
service restriction.
The question now becomes one of whether the higher costs associated with the more
restrictive service level are justified. Since there is no sales-service relationship for this

213

problem, we can only estimate the worth of the service. That is, can enough sales be
generated to cover the higher service level? If physical distribution costs for the company are 15 percent of sales, which is probably a conservative estimate, then 1/0.15 =
$6.70 in sales must be generated for each dollar that is added to distribution costs.
Therefore, to cover $404,000 in cost would require
$404,000 $6.70
38,124 cwt.
$0.71 / lb.100lb./cwt.
increase in sales. In terms of overall demand, this would be 38,124100/1,477,026 =
2.5%.
But not all customers would experience a higher service level. Comparing the
demand centers for 299,818 cwt. of demand shows a reduction in warehouse to customer
miles. Thus, moving from a minimum cost network to one with a high service level,
where the percent of demand less than 300 miles increases from 75 percent to 93 percent,
requires that the 38,124 cwt. increase in demand occur in the 299,818 cwt. of demand
affected by the change. This would be a 13 percent increase.
The products are not highly differentiated from others in the marketplace so that
service plays an important role in selling these products. Whether a 93 75 = 18
percentage points increase in service can result in a 2.5 percent increase in overall sales
cannot be judged by the distribution department alone. The sales department must play
an important part in indicating whether the additional sales are possible. If they are not
likely to be realized, there is no incentive for a network other than the minimum cost one.
If this information is not available from sales, the conclusion is likely to be to
maintain the status quo as represented by the benchmark. That is, one-day service is
most likely to guide the design.
Overall Analysis and Summary
The recommended design would involve an immediate increase in the number of
warehouses from 22 to 30. In addition, there should be an immediate reallocation of
demand among the existing plants. No reduction in the customer service level seems
justified at this time. Therefore, a total cost reduction of $42,112,000 41,321,000 =
$791,000 per year seems immediately achievable (run 1 vs. run 18). By the end of the
2nd year, the Memphis plant should be brought on stream and the network should begin
to evolve from the current design (run 24) to that for the 5th year (run 25). The addition
of a plant is justified from the high rate of return realized from the profit potential of
being able to continue meeting the growth in demand.
For the current year, a breakdown of the service and the cost changes show the
following.

214

Cost type
Production
Whse operations
Order processing
Inventory carrying
Transportation
Inbound
Outbound
Total costs ($000s)

Benchmark

Currentyear
design

Change from
benchmark

$30,762
1,578
369
457

$30,675
1,572
358
490

$ -87
- 6
-11
+33

-0.3%
-0.4
-3.0
+7.2

2,050
6,896
$42,112

1,860
6,365
$41,320

-190
-531
$-792

-9.3
-7.7
-1.9%

By the 5th year, total distribution costs should be $53,179,000, or


$53,179,000/1,908,606 = $27.86, compared with the current-year cost of
42,112,463/1,477,026 = $28.51 per cwt. If current year costs are projected to the 5thyear demand level, the 5th-year production/distribution costs might be 28.511,908,606
= $54,414,357, or a savings of $54,414,357 53,179,000 = $1,235,357 per year. Of
course, these savings can only be realized through the addition of capacity at Memphis
for $4,000,000. If this capacity is useful for at least 15 years, the amortization of
$4,000,000/15 = $267,000 per year would yield a net savings of $532,000 per year.
Overall, the design change appears to be justified.

215

APPENDIX A Listing of Selected Computer Runs


Service
constraint

Run Run
no. description

No of
plants

Plant
capacity

Demand
level

1
2
3
4
5
6
7
8
9
10
11
12
13
14
15
16
17
18
19
20
21
22
23
24
25

4
4
4
6
4
4
4
4
4
4
4
4
4
5
5
6
4
4
4
5
4
4
4
4
5

Current
Current
Current
Crnt+1m
Current
Current
Current
Current
Current
Current
Current
Current
Current
Current
Current
Current
Current
See cmt
Current
Current
Current
Current
Current
See cmt

Current
- mi
Current 300
Current 9000
Current 9000
5th yr. 300
Current 300
Current 300
Current 300
Current 300
5th yr. 300
5th yr. 300
5th yr. 300
5th yr. 300
5th yr. 300
5th yr. 300
5th yr. 300
5th yr. 300
Current 300
Current 300
5th yr. 300
Current 600
Current 600
Current 600
Current 375
5th yr. 375

Benchmark
Improved benchmark
No serv constraint
Max opportunity
Future yr-imp bmk
Test 27 whses
Test 32 whses
Test 37 whses
Test 42 whses
Exp Covington
Exp New York
Exp Arlington
Exp Long Beach
Add Memphis
Add Chicago
Add Mem & Chi
No plant expansion
Optimum whses
Optimum whses
Optimum whses
Test cust service
Test cust service
Test cust service
Optimum whses
Optimum whses

No of
whses

Total
costs

Percent of
demand within
300 600 Comments

22
21
18
40
21
26
31
36
40
21
21
21
21
21
20
20
20
31
30
31
31
26
19
30
30

$42,112
41,725
40,896
40,409
53,777
41,744
41,615
41,501
41,486
54,145
53,986
54,709
55,251
53,705
53,781
53,382
45,164
41,447
41,563
53,342
40,996
41,043
41,201
41,321
53,179

93%
93
71
81
93
95
98
99
99
94
93
94
94
94
94
95
98
98
97
98
80
88
75
92
92

98%
98
89
94
98
100
100
100
100
98
98
98
98
98
98
98
100
100
100
100
100
100
98
100
100

Current network design


No investment required
Added plants at 1m cwt
Plant cap + 1m cwt

Covington cap + 1m cwt


New York + 1m cwt
Arlington cap + 1m cwt
Long Beach cap + 1m cwt
Add Memphis at 1m cwt
Add Chicago at 1m cwt
Add Chi & Mem at 1m cwt ea
Only 85.4% of demnd served
Plants at current capacity
Plants at current thruput
Memphis at 1m cwt
Whses at opt no = 31
Whses from opt no = 31
Whses from current 22
Service level at bmk
Serv at bmk/Mem @ 1m cwt

216

ESSEN USA
Teaching Note
Strategy

Essen USA is concerned with entire supply channel performance. The supply channel
consists of four echelons ranging from factory to customers. The purpose of this case
study is for the student to manipulate the supply channel variables through the use of a
channel simulator in order to improve individual member and system-wide performance.
The channel variables include forecasting methods, inventory policies, transportation
services, production lot sizes, order processing costs, and stock availability levels.
Students should seek to optimize channel performance, although it is not expected that
the optimum actually can be found or verified. However, improving performance over
existing levels is achievable.
The SCSIM module of LOGWARE is used to simulate the demand and product flows
throughout the multi-echelon supply chain. SCSIM is an ordinary Monte Carlo day-today type of simulator. Using a simulator for performance improvement requires thinking
of it in terms of as an experimental methodology. That is, a single run of the simulator is
a particular event sequence generated from random numbers. Changing the seed number
in the simulator causes a different set of random numbers to be generated and possibly
another outcome from the same input data. A simulation run with a specified seed
number should be viewed as a single statistical observation and multiple outcomes from
various seed numbers should be treated as a statistical sample and analyzed accordingly,
i.e., comparing means and standard deviations.
Each simulation is run for a period of 11 years with results taken from years 2
through 11. The first year is not used since it can show unstable results due to startup
conditions. The results appear to reach steady state by the second year, and the results
for the 10 years thereafter are averaged to give a reasonable representation of channel
performance for a given run. The database used to represent the current performance of
the channel, as derived from the case study, is summarized in the Appendix A of this note
and a typical run report is shown in Appendix B.
This case provides students with the opportunity to observe the operation of a multiechelon supply channel and to assess the impact of changing key operating variables on
individual members as well as on channel-wide performance. The effect on cost and
customer service as well as sales, inventory, and back order levels of demand patterns,
demand forecasting methods, inventory control methods, transportation performance,
production lot sizing, order processing procedures, and item fill rates can be observed in
both graphical and report forms. Most importantly, students can see the effects of supply
chain decisions rather than project the results statistically.
Questions

1. What can you say about the logistics performance throughout the supply channel for
Essen and its customers?

217

General observations

It is recognized that Essen must deal with demand that has significant seasonal peaks at
gift giving times of the year as shown in Figure 1. Compared with a smooth demand
pattern, this can cause increasing demand variability upstream from the customers, as
illustrated in Figure 2. This bull whip effect is partly a result of the demand for an
upstream member being derived from the order size and pattern of its immediate
downstream channel member. Forecast accuracy, lead-time uncertainty, and inventory
control method also affect demand variability and the resulting cost of that variability.

Figure 1 Typical Demand Pattern for Essen Over the Period of One Year

Retailer
Distri
warehouse

Essen
Warewarehouse
house

Factory
Factory

-butor

Retailer
Retailer

Figure 2 Increasing Demand Variability of Upstream Channel Members for a


Four-Year Period

218

Benchmark

Running the simulator (SCSIM) with a seed number of 123456 and simulated period of
11 years with results taken from the last 10 years, the channel generates average annual
sales of $109.5 million for a net average annual system profit contribution of $24.4
million, as shown in Table 1. The question arises as to whether channel performance can
be improved and profits increased. At least two observations can be made that suggest
there is room for improvement. First, the inventory levels for both the retailers
warehouse and Essens warehouse are quite high compared with the Retailer (see Figure
3). It is possible that Retailer inventories are too low. However, the inventory turnover
ratio is about 7 for the Essens warehouse (see Table 1). This is not particularly high for
a food product that might have a turnover at least in the range of 10 to 12. The turnover
for the retailers warehouse appears more in line with industry norms of about 13 (see
Table 1).

Retailer
warehouse

Essen
warehouse

Retailer

Figure 3 Inventory Levels for Four Years Using Benchmark Data

Second, the backorders at the Retailer level do not seem to recover well from the
seasonal spike in demand. Correspondingly, the Retailer inventory turnover is 81 (see
Table 1), which is quite high. The low percentage of demand filled on request (<50%)
suggests that inadequate inventory is being maintained to meet reasonable fill rates.
Third, customer service levels are also low for the retailers warehouse and Essens
warehouse. Backorder occurrences are high for both channel members. Although
inventory levels are adequate most of the time, seasonal demand rippling through the
supply chain causes a significant number of back orders before inventory can be
replenished.
The observation is that there is an opportunity to improve channel performance,
especially in terms of customer service. A major concern is how to mange the seasonal
demand pattern that is causing the cyclical behavior throughout the echelons of the
channel. Current performance of the channel members is summarized in Table 1 for 4
simulation runs using different seed numbers.

219

Table 1

Average Annual Performance of Channel Members and the System at


Benchmark

Channel member

Run 1

Run 2

Run 3

Run 4

Average

$73,105,904
37,918
$1,928

$72,967,088
37,846
$1,928

$72,616,192
37,664
$1,928

$72,477,376
37,592
$1,928

$72,791,640
37,755
$1,928

6.52
<50%
$5,578,291
$147.02

6.59
<50%
$5,549,202
$146.50

6.61
<50%
$5,521,236
$146.32

6.58
<50%
$5,540,447
$146.65

6.58
<50%
$5,547,294
$146.62

12.95
<50%
$3,873,236
$101.94

13.01
<50%
$3,895,406
$102.15

12.96
<50%
$3,853,890
$102.14

12.89
<50%
$3,912,699
$103.07

12.95
<50%
$3,883,808
$102.33

81.02
<50%
38,017
$2,884,527
$76.19

80.38
53.02%
37,983
$2,768,697
$72.89

80.45
54.09%
37,774
$2,939,464
$77.82

80.60
<50%
37,804
$2,981,607
$78.87

80.61
<50%
37,895
$2,893,574
$76.44

$24,426,593
22.23%
123456

$24,591,053
22.40%
444444

$24,235,498
22.20%
555555

$24,340,563
22.28%
666666

$24,398,426
22.28%

Essens factory
Total cost

Units produced
Cost per unit
Essens
warehouse
TO ratio
Fill rate
Cost

Cost per unit


Retailers
warehouse
TO ratio
Fill rate
Cost

Cost per unit


Retailer
TO ratio
Fill rate
Units sold
Cost

Cost per unit


System
Profit
Profit as % of sales
Seed Number

2. What steps would you suggest taking to improve logistics performance throughout
the channel? Do any of the changes involve Essen? If so, does the company directly
realize any cost and/or operating performance improvements?
There are a number of actions that generally can be taken to lower costs and improve
customer service. Improving the forecast, shortening the lead times, changing the
inventory control policy, and changing production lot sizes are all variables that can be
altered for possible performance improvement. The interactions among these variables
and the large number of variable combinations preclude finding the optimal set.
However, they can be explored in a systematic way to find improvement. The primary
focus of this analysis will be to increase the fill rates at the risk of increasing costs.
Ultimately, revenues, through improved customer service, may be preserved or increased
to more than compensate for reduced profits.
Retailer Level

Start with the retailer because of the proximity to the customer. Fill rates need to be
improved, probably in the 95-99% range as specified in the database. Inventory turns can

220

be guided by the industry average of 12 turns per year. Where the two cannot be jointly
met, service will prevail.
Clearly, putting additional inventory at the retail point will improve customer service.
Using the companys current inventory policy of stocking to demand, the target level can
be raised without changing the review time. Exploring different target levels shows 14
days to offer about 35 turns and a 99+% fill rate. Because of the high cost of a back
order, total costs at the retail level drop significantly.
Altering the forecasting method and the settings associated with the method yield
little opportunity for improvement. Using an exponential smoothing model with a high
smoothing constant to better follow the seasonal changes in demand results in increased
costs. Lowering the smoothing constant to 0.1 did not offer improvement either.
Altering the number of periods in the moving average model did not improve costs and
only degraded performance. Shortening the review time in the stock-to-demand reorder
policy did have a positive effect on fill rate, but resulted in high costs and lower
inventory turns. The tradeoff did not seem beneficial, given the fill rate and turnover
targets.
Retail Warehouse Level

Determining an improved policy at this level is difficult because a 95% fill rate and 10 to
12 inventory turns is an illusive goal. Using service as the primary target, a stock-todemand control policy is used with a review period of 7 days and a target of 25 days of
inventory. The forecasting method is moving average with a period of 7 days. The
performance achieved at this channel level is about 9 inventory turns per year and a 97%
fill rate.
Essens Warehouse Level

The performance at Essens warehouse level seems to mimic that at the retail warehouse
level except that there is more demand variability. Again, an inventory turnover ratio in
the target range cannot be achieved while maintaining a high fill rate level. Trying to
achieve high service levels with high levels of inventory is difficult, probably due to the
extensive demand variability that filters back to this member of the channel. Multiple
simulation runs show that a high fill rate cannot consistently be achieved even when on
the average inventory levels are high. However, average performance shows an 82% fill
rate and 1.5 inventory turns per year based on a 7-day moving average forecasting model
and a stock-to-demand inventory control policy with a review time of 7 days and an
inventory target of 25 days.
Essens Factory

The concern with the factory level in the channel is whether product should be
manufactured in a larger lot size, but with slightly higher production time variability.
The reduced costs seem to out weigh the negative effects of increased variability.
Producing in the larger lot size is favored.
Overall

Using the objective of improving customer service, it is not surprising that supply
channel costs increase as shown from the reduced profit in Table 2 compared with Table
1. The average fill rate has increased for all members of the channel, but the cost effects
221

are spread disproportionately among the members. Even with a higher fill rate, the
retailer benefits from a substantial reduction in the cost per unit sold. On the other hand,
the cost for handling a unit of the product at Essens warehouse is substantially increased.
Essen should take advantage of the cost reduction from producing in the larger batch
size, but this does not offset the higher cost at the companys warehouse. As an upstream
member of the supply channel, Essen undoubtedly suffers from the variability in demand,
which cannot entirely be controlled.
The retailer benefits from the action to increase fill rates across the channel.
However, Essen is put at a disadvantage and may take a counter action to improve its
cost position. Essen may simply lower its inventory level by reducing the reorder target
quantity from 25 to 10 days. This reduces Essens per-unit warehouse cost, but it also
increases the costs for the retailer. The reduced inventory level at the Essen warehouse
causes lower fill rates for the downstream retailer. Unless the retailer can find an
incentive to reward Essen for its good service, it will be difficult for Essen to provide the
level of service that the retailer would like and that is economically beneficial to Essen.
Table 2

Average Annual Performance of Channel Members and the System as


Revised

Channel member

Run 1

Run 2

Run 3

Run 4

Average

$68,638,738
35,950
$1,909

$74,761,258
39,286
$1,903

$78,418,188
41,268
$1,900

$75,400,666
39,622
$1,903

$74,304,713
39,032
$1,904

1.47
90.65%
$12,060,444
$317.51

1.42
100%
$12,449,170
$326.28

1.51
63.16%
$11,895,743
$311.87

1.46
72.92%
$12,178,581
$319.61

1.47
81.68%
$12,145,984
$318.82

9.24
96.64%
$4,018,143
$105.70

9.02
96.29%
$4,080,160
$107.09

9.14
97.60%
$4,043,973
$106.55

9.28
98.09%
$3,992,791
$105.51

9.17
97.16%
$4,033,767
$106.21

35.44
99.52%
38,017
$727,378
$19.13

35.36
99.53%
37,936
$717,363
$18.91

34.99
99.70%
37,979
$709,772
$18.69

35.14
99.40%
37,730
$748,198
$19.75

35.23
99.54%
37,916
$725,677
$19.12

$24,423,849
22.23%
123456

$17,627,666
16.08%
111111

$14,690,765
13.38%
222222

$17,184,464
15.69%
333333

$18,481,686
16.85%

Essens factory
Total cost

Units produced
Cost per unit
Essens
warehouse
TO ratio
Fill rate
Cost

Cost per unit


Retailers
warehouse
TO ratio
Fill rate
Cost

Cost per unit


Retailer
TO ratio
Fill rate
Units sold
Cost

Cost per unit


System
Profit
Profit as % of sales
Seed Number

222

3. Would shipping by airfreight from Germany be a benefit to channel performance? To


Essen?
No. Selling candies to end customers at $2,890 per thousand lb. using airfreight shipping
results in obvious loss to Essen. The cost of shipping by air is $1,833 per thousand lb.,
plus $1,000 material cost and $850 production cost results in a total cost much higher
than selling price. There is no point to using airfreight. Running the simulation with the
higher freight rate but lower variability confirms that channel profits would be negative.
4. Is there a benefit to producing in the larger 20,000-pound batch size?
Yes. This was tested in question 2. From Tables 1 and 2, it can be seen that production
costs drop from $1,928 per unit to $1,904 per unit. The overall channel cost reduction
overshadows the negative effects of greater length and variability in production time.

Appendix A Simulation Database for Essen USA Under Current Conditions


Title: ESSEN USA
Initialization
123456
11
2890

Seed value
Length of simulation, years
Annual price, $/unit

Customer demand pattern


Generate daily demand
100
Average daily demand, units
15
Standard deviation of daily demand, units
1
Annual demand growth increment, %
Monthly seasonal indices
Month Index Month Index Month Index Month
1
0.25
4
0.75
7
0.75
10
2
1.25
5
0.75
8
0.75
11
3
1.25
6
0.75
9
0.75
12
Retailer/Level 1
Product item data
2220
1
35
25
1
0
98
670

Index
0.75
1.50
2.50

Item value in inventory, $/unit


Customer order filling cost, $/unit
Purchase order processing cost, $/order
Inventory carrying cost, %/year
Average customer order fill time, days
Customer order fill time standard deviation, days
In-stock probability, %
Back order cost, $/unit

223

Forecasting method
Moving average
7
Number of periods
Reorder policy
Stock-to-demand control method
10
Target days of inventory
7
Review time in days
Distributor/Level 2
Product item data
2220
Item value in inventory, $/unit
20
Retailer order filling cost, $/unit
75
Purchase order processing cost, $/order
25
Inventory carrying cost, %/year
2
Average retailer order fill time, days
0.2
Retailer order fill time standard deviation, days
95
In-stock probability, %
100
Back order cost, $/unit
Forecasting method
Moving average
30
Number of periods
Reorder policy
Stock-to-demand control method
45
Target days of inventory
30
Review time in days
Warehouse/Level 3
Product item data
1710
Item value in inventory, $/unit
15
Distributor order filling cost, $/unit
75
Purchase order processing cost, $/order
20
Inventory carrying cost, %/year
3
Average distributor order filling time, days
0.3
Distributor order fill time, days
95
In-stock probability, %
25
Back order cost, $/unit
Forecasting method
Moving average
360
Number of periods
Reorder policy
Stock-to-demand control method
Target days of inventory
90
Review time in days
30

224

Factory/Source
Product item data

850
10
10
8
2
1000

Production cost, $/unit


Minimum production lot size, units
Warehouse order filling cost, $/unit
Average production time, days
Production time standard deviation, days
Purchase cost, $/unit
Transportation

Transport between Distributor and Retailer

25
1
0

Transport cost, $/unit


Average time in-transit, days
Transit time standard deviation, days

Transport between Warehouse and Distributor

70
5
1

Transport cost, $/unit


Average time in-transit, days
Transit time standard deviation, days

Transport between Factory and Warehouse

78
9
3

Transport cost, $/unit


Average time in-transit, days
Transit time standard deviation, days

Appendix B

Benchmark Simulation Results with Seed Number 123456 and Simulation


Length of 11 Years

SUPPLY CHANNEL REPORT FOR SIMULATED YEARS 2 TO 11


Yearly
Simulated
average
period
FINANCIAL PERFORMANCE
$109,868,552 $1,098,685,520
Revenue
37,918,000
379,180,000
Cost of purchased goods
71,950,552
719,505,520
Gross margin
32,230,300

322,303,000

949,025
2,656,010
2,957,604

9,490,250
26,560,100
29,576,040

38,017
759,890
569,145

380,168
7,598,900
5,691,450

1,638
683
675

16,380
6,825
6,750

Production cost
Transportation costs:
Distributor to retailer
Warehouse to distributor
Factory to warehouse
Sales order handling cost for:
Customer orders
Retailer orders
Distributor orders
Order processing cost for:
Orders to distributor
Orders to warehouses
Orders to factory
Inventory costs

225

260,414
1,627,909
1,988,984

2,604,145
16,279,092
19,889,837

Retailer
Distributor
Warehouse

2,584,458
535,730
363,478

25,844,580
5,357,300
3,634,775

Back order costs


Retailer
Distributor
Warehouse

$24,426,593

$244,265,928

Net profit contribution

Appendix C Simulation Database for Essen USA as Revised for Serivce


Improvement
Title: ESSEN USA
Initialization
123456
11
2890

Seed value
Length of simulation, years
Annual price, $/unit

Customer demand pattern


Generate daily demand
100
Average daily demand, units
15
Standard deviation of daily demand, units
1
Annual demand growth increment, %
Monthly seasonal indices
Month Index Month Index Month Index Month
1
0.25
4
0.75
7
0.75
10
2
1.25
5
0.75
8
0.75
11
3
1.25
6
0.75
9
0.75
12

Index
0.75
1.50
2.50

Retailer/Level 1
Product item data
2220
Item value in inventory, $/unit
1
Customer order filling cost, $/unit
35
Purchase order processing cost, $/order
25
Inventory carrying cost, %/year
1
Average customer order fill time, days
0
Customer order fill time standard deviation, days
98
In-stock probability, %
670
Back order cost, $/unit
Forecasting method
Moving average
7
Number of periods
Reorder policy
Stock-to-demand control method
14
Target days of inventory

226

Review time in days

Distributor/Level 2
Product item data
2220
Item value in inventory, $/unit
20
Retailer order filling cost, $/unit
75
Purchase order processing cost, $/order
25
Inventory carrying cost, %/year
2
Average retailer order fill time, days
0.2
Retailer order fill time standard deviation, days
95
In-stock probability, %
100
Back order cost, $/unit
Forecasting method
Moving average
7
Number of periods
Reorder policy
Stock-to-demand control method
35
Target days of inventory
7
Review time in days
Warehouse/Level 3
Product item data
1710
Item value in inventory, $/unit
15
Distributor order filling cost, $/unit
75
Purchase order processing cost, $/order
20
Inventory carrying cost, %/year
3
Average distributor order filling time, days
0.3
Distributor order fill time standard deviation, days
95
In-stock probability, %
25
Back order cost, $/unit
Forecasting method
Moving average
7
Number of periods
Reorder policy
Stock-to-demand control method
25
Target days of inventory
7
Review time in days
Factory/Source
Product item data

825
20
10
10
2.1
1000

Production cost, $/unit


Minimum production lot size, units
Warehouse order filling cost, $/unit
Average production time, days
Production time standard deviation, days
Purchase cost, $/unit

227

Transportation
Transport between Distributor and Retailer

25
1
0

Transport cost, $/unit


Average time in-transit, days
Transit time standard deviation, days

Transport between Warehouse and Distributor

70
5
1

Transport cost, $/unit


Average time in-transit, days
Transit time standard deviation, days

Transport between Factory and Warehouse

78
9
3

Transport cost, $/unit


Average time in-transit, days
Transit time standard deviation, days

228

CHAPTER 15
LOGISTICS/SUPPLY CHAIN ORGANIZATION

All questions in this chapter require individual judgment and response. No answers are
offered.

229

CHAPTER 16
LOGISTICS/SUPPLY CHAIN AND CONTROL

All questions in this chapter require individual judgment and response. No answers are
offered.

230

Das könnte Ihnen auch gefallen